Sunteți pe pagina 1din 160

Exam Title : Test 21-Revision: Polity & ...

Email
Contact :

QUESTION 1.
The nine judge constitution bench has declared that ‘Right to Privacy’ is a Fundamental Right in
Constitution. According to SC, Right to Privacy is an intrinsic part of

a) Right to freedom of Speech and Expression


b) Right to Life and Personal Liberty
c) Right to Equality
d) Right against exploitation
Correct Answer: B
Your Answer: B
Explanation

Solution (b)

The nine-judge Constitution Bench of the Supreme Court headed by Chief Justice of India JS
Khehar in a landmark unanimous decision has declared right to privacy a fundamental right
under the constitution.

Supreme Court Judgment

The apex court ruled that right to privacy is an intrinsic part of Right to Life and Personal
Liberty under Article 21 and entire Part III of the Constitution.

It overruled the apex court’s earlier two judgments that right to privacy is not protected under
the Constitution.

Think

· Right to Privacy

QUESTION 2.
Which of the following statements are correct regarding Pitt’s India Act of 1784?

1. It distinguished between the commercial and political functions of the company.

2. It created Board of Control to manage Political Affairs.

3. It made governors of Bombay and Madras Presidencies subordinate to Governor General of


Bengal.

Select the code from below:

a) 1 and 2
b) 2 and 3
c) 1 and 3
d) All of the above
Correct Answer: A
Your Answer: Unanswered
Explanation

Solution (a)

IASbaba
Web: http://ilp.iasbaba.com/ Score:
Email: ilp@iasbaba.com 4.00 / 200
Page 1
Exam Title : Test 21-Revision: Polity & ...
Email
Contact :

Pitt’s India Act of 1784

1. It distinguished between the commercial and political functions of the Company.

2. It allowed the Court of Directors to manage the commercial affairs but created a new body
called Board of Control to manage the political affairs. Thus, it established a system of double
government.

3. It empowered the Board of Control to supervise and direct all operations of the civil and
military government or revenues of the British possessions in India.

Thus, the act was significant for two reasons: first, the Company’s territories in India were for
the first time called the ‘British possessions in India’; and second, the British Government was
given the supreme control over Company’s affairs and its administration in India.

Note:

Third statement is a feature of Regulating Act of 1773.

Think

· Board of Control

· Board of Directors

QUESTION 3.
Who of the following is known as ‘Father of Communal Electorate’?

a) Lord Chemsford
b) Lord Minto
c) Lord Dalhousie
d) Lord Curzon
Correct Answer: B
Your Answer: Unanswered
Explanation

Solution (b)

Lord Minto introduced Morley – Minto Reforms (Indian Councils Act 1909). In this for the first
time separate electorate were introduced for Muslims, Chamber of Commerce Bombay and
madras and Zamindars.

Since it was based on communal basis (for Muslims), Minto is known as father of communal
electorate.

Features of the Act of 1909

This Act is also known as Morley-Minto Reforms (Lord Morley was the then Secretary of State
for India and Lord Minto was the then Viceroy of India).

IASbaba
Web: http://ilp.iasbaba.com/ Score:
Email: ilp@iasbaba.com 4.00 / 200
Page 2
Exam Title : Test 21-Revision: Polity & ...
Email
Contact :

1. It considerably increased the size of the legislative councils, both Central and provincial. The
number of members in the Central Legislative Council was raised from 16 to 60. The number of
members in the provincial legislative councils was not uniform.

2. It retained official majority in the Central Legislative Council but allowed the provincial
legislative councils to have non-official majority.

3. It enlarged the deliberative functions of the legislative councils at both the levels. For
example, members were allowed to ask supplementary questions, move resolutions on the
budget, and so on.

4. It provided (for the first time) for the association of Indians with the executive Councils of the
Viceroy and Governors. Satyendra Prasad Sinha became the first Indian to join the Viceroy’s
Executive Council. He was appointed as the law member.

5. It introduced a system of communal representation for Muslims by accepting the concept of


‘separate electorate’. Under this, the Muslim members were to be elected only by Muslim
voters. Thus, the Act ‘legalised communalism’ and Lord Minto came to be known as the Father
of Communal Electorate .

6. It also provided for the separate representation of presidency corporations, chambers of


commerce, universities and zamindars.

Think

· Communal Award

· Puna Pact

QUESTION 4.
Consider the following statements regarding Constituent Assembly of India:

1. It was a sovereign body.

2. It acted as a constitution making body as well as legislature.

3. It ratified India’s membership of the Commonwealth.

Which of the above statements are correct?

a) 1 and 2
b) 2 and 3
c) 1 and 3
d) All of the above
Correct Answer: D
Your Answer: Unanswered
Explanation

Solution (d)

The Indian Independence Act of 1947 made the following three changes in the position of the
Constituent Assembly:

IASbaba
Web: http://ilp.iasbaba.com/ Score:
Email: ilp@iasbaba.com 4.00 / 200
Page 3
Exam Title : Test 21-Revision: Polity & ...
Email
Contact :

1. The Assembly was made a fully sovereign body, which could frame any Constitution it
pleased. The act empowered the Assembly to abrogate or alter any law made by the British
Parliament in relation to India.

2. The Assembly also became a legislative body. In other words, two separate functions were
assigned to the Assembly that is, making of a constitution for free India and enacting of
ordinary laws for the country. These two tasks were to be performed on separate days. Thus,
the Assembly became the first Parliament of free India (Dominion Legislature). Whenever the
Assembly met as the Constituent body it was chaired by Dr. Rajendra Prasad and when it met as
the legislative body6, it was chaired by G V Mavlankar. These two functions continued till
November 26, 1949, when the task of making the Constitution was over.

3. The Muslim League members (hailing from the areas7 included in the Pakistan) withdrew
from the Constituent Assembly for India. Consequently, the total strength of the Assembly came
down to 299 as against 389 originally fixed in 1946 under the Cabinet Mission Plan. The
strength of the Indian provinces (formerly British Provinces) was reduced from 296 to 229 and
those of the princely states from 93 to 70. The state-wise membership of the Assembly as on
December 31, 1947, is shown in Table 2.1 at the end of this chapter.

Other Functions Performed

In addition to the making of the Constitution and enacting of ordinary laws, the Constituent
Assembly also performed the following functions:

1. It ratified the India’s membership of the Commonwealth in May 1949.

2. It adopted the national flag on July 22, 1947.

3. It adopted the national anthem on January 24, 1950.

4. It adopted the national song on January 24, 1950.

5. It elected Dr Rajendra Prasad as the first President of India on January 24, 1950.

Think

· Indian Independence Act 1947

QUESTION 5.
Which of the following Committees are NOT correctly matched with their Chairman?

a) Steering Committee – Dr Rajendra Prasad


b) Drafting Committee – Dr. B R Ambedkar
c) States Committee – Sardar Pater
d) Union Powers Committee – Pt Nehru
Correct Answer: C
Your Answer: C
Explanation

Solution (c)

Major Committees

IASbaba
Web: http://ilp.iasbaba.com/ Score:
Email: ilp@iasbaba.com 4.00 / 200
Page 4
Exam Title : Test 21-Revision: Polity & ...
Email
Contact :

1. Union Powers Committee – Jawaharlal Nehru

2. Union Constitution Committee – Jawaharlal Nehru

3. Provincial Constitution Committee – Sardar Patel

4. Drafting Committee – Dr. B.R. Ambedkar

5. Advisory Committee on Fundamental Rights, Minorities and Tribal and Excluded


Areas – Sardar Patel.

This committee had the following sub-committees:

(a) Fundamental Rights Sub-Committee – J.B. Kripalani

(b) Minorities Sub-Committee – H.C. Mukherjee

(c) North-East Frontier Tribal Areas and Assam Excluded & Partially Excluded Areas Sub-
Committee – Gopinath Bardoloi

(d) Excluded and Partially Excluded Areas (Other than those in Assam) Sub-Committee – A.V.
Thakkar

6. Rules of Procedure Committee – Dr. Rajendra Prasad

7. States Committee (Committee for Negotiating with States ) – Jawaharlal Nehru

8. Steering Committee – Dr. Rajendra Prasad

QUESTION 6.
Which of the following statements are correct regarding Indian judiciary?

1. Indian Judicial System is integrated as well as independent

2. Single system of courts enforces both the central laws as well as the state laws

Select the code from following:

a) 1 only
b) 2 only
c) Both 1 and 2
d) Neither 1 nor 2
Correct Answer: C
Your Answer: Unanswered
Explanation

Solution (c)

Integrated and Independent Judiciary

The Indian Constitution establishes a judicial system that is integrated as well as independent.

IASbaba
Web: http://ilp.iasbaba.com/ Score:
Email: ilp@iasbaba.com 4.00 / 200
Page 5
Exam Title : Test 21-Revision: Polity & ...
Email
Contact :

The Supreme Court stands at the top of the integrated judicial system in the country. Below it,
there are high courts at the state level. Under a high court, there is a hierarchy of subordinate
courts, that is, district courts and other lower courts. This single system of courts enforces both
the central laws as well as the state laws, unlike in USA, where the federal laws are enforced by
the federal judiciary and the state laws are enforced by the state judiciary.

The Supreme Court is a federal court, the highest court of appeal, the guarantor of the
fundamental rights of the citizens and the guardian of the Constitution. Hence, the Constitution
has made various provisions to ensure its independence—security of tenure of the judges, fixed
service conditions for the judges, all the expenses of the Supreme Court charged on the
Consolidated Fund of India, prohibition on discussion on the conduct of judges in the
legislatures, ban on practice after retirement, power to punish for its contempt vested in the
Supreme Court, separation of the judiciary from the executive, and so on.

Think

· How does integrated judiciary important for federal structure?

QUESTION 7.
India is a secular state. Which of the following statements regarding Indian Secularism are
correct?

1. There is no official religion of Indian state.

2. State does not discriminate against any citizen on the grounds of religion.

3. There is complete separation of the religion and the State.

Select the code from following:

a) 1 and 2
b) 2 and 3
c) 1 and 3
d) All of the above
Correct Answer: A
Your Answer: Unanswered
Explanation

Solution (a)

A Secular State

The Constitution of India stands for a secular state. Hence, it does not uphold any particular
religion as the official religion of the Indian State. The following provisions of the Constitution
reveal the secular character of the Indian State:

(a) The term ‘secular’ was added to the Preamble of the Indian Constitution by the 42 nd Consti
tutional Amendment Act of 1976.

(b) The Preamble secures to all citizens of India liberty of belief, faith and worship.

IASbaba
Web: http://ilp.iasbaba.com/ Score:
Email: ilp@iasbaba.com 4.00 / 200
Page 6
Exam Title : Test 21-Revision: Polity & ...
Email
Contact :

(c) The State shall not deny to any person equality before the law or equal protection of the
laws (Article 14).

(d) The State shall not discriminate against any citizen on the ground of religion (Article 15).

(e) Equality of opportunity for all citizens in matters of public employment (Article 16).

(f) All persons are equally entitled to freedom of conscience and the right to freely profess,
practice and propagate any religion (Article 25).

(g) Every religious denomination or any of its section shall have the right to manage its
religious affairs (Article 26).

(h) No person shall be compelled to pay any taxes for the promotion of a particular religion
(Article 27).

(i) No religious instruction shall be provided in any educational institution maintained by the
State (Article 28).

(j) Any section of the citizens shall have the right to conserve its distinct language, script or
culture (Article 29).

(k) All minorities shall have the right to establish and administer educational institutions of
their choice (Article 30).

(l) The State shall endeavour to secure for all the citizens a Uniform Civil Code (Article 44).

The Western concept of secularism connotes a complete separation between the religion (the
church) and the state (the politics). This negative concept of secularism is inapplicable in the
Indian situation where the society is multi religious. Hence, the Indian Constitution embodies
the positive concept of secularism, i.e., giving equal respect to all religions or protecting all
religions equally.

Moreover, the Constitution has also abolished the old system of communal representation13,
that is, reservation of seats in the legislatures on the basis of religion. However, it provides for
the temporary reservation of seats for the scheduled castes and scheduled tribes to ensure
adequate representation to them.

QUESTION 8.
Which of the following features of Indian Constitution have been taken from Government of
India Act 1935?

1. Federal Scheme

2. Emergency provisions

3. Judiciary

4. Office of Governor

Select the code from below:

a) 1,2 and 3

IASbaba
Web: http://ilp.iasbaba.com/ Score:
Email: ilp@iasbaba.com 4.00 / 200
Page 7
Exam Title : Test 21-Revision: Polity & ...
Email
Contact :

b) 1,3 and 4
c) 2,3 and 4
d) All of the above
Correct Answer: D
Your Answer: Unanswered
Explanation

Solution (d)

Sources of the Constitution at a Glance

1. Government of India Act of 1935 : Federal Scheme, Office of governor, Judiciary, Public
Service Commissions, Emergency provisions and administrative details.

2. British Constitution : Parliamentary government, Rule of Law, legislative procedure, single


citizenship, cabinet system, prerogative writs, parliamentary privileges and bicameralism.

3. US Constitution : Fundamental rights, independence of judiciary, judicial review,


impeachment of the president, removal of Supreme Court and high court judges and post of
vice-president.

4. Irish Constitution : Directive Principles of State Policy, nomination of members to Rajya


Sabha and method of election of president.

5. Canadian Constitution : Federation with a strong Centre, vesting of residuary powers in


the Centre, appointment of state governors by the Centre, and advisory jurisdiction of the
Supreme Court.

6. Australian Constitution : Concurrent List, freedom of trade, commerce and intercourse,


and joint sitting of the two Houses of Parliament.

7. Weimar Constitution of Germany : Suspension of Fundamental Rights during Emergency.

8. Soviet Constitution (USSR, now Russia): Fundamental duties and the ideal of justice
(social, economic and political) in the Preamble.

9. French Constitution: Republic and the ideals of liberty, equality and fraternity in the
Preamble.

10. South African Constitution: Procedure for amendment of the Constitution and election of
members of Rajya Sabha.

11. Japanese Constitution: Procedure established by Law.

QUESTION 9.
Preamble promises ‘justice’ for every citizen of India. What type of justice is promised in
Preamble?

1. Social

2. Economic

3. Political

IASbaba
Web: http://ilp.iasbaba.com/ Score:
Email: ilp@iasbaba.com 4.00 / 200
Page 8
Exam Title : Test 21-Revision: Polity & ...
Email
Contact :

4. Physical

Select the code from following:

a) 1,2 and 3
b) 2,3 and 4
c) 1,3 and 4
d) All of the above
Correct Answer: A
Your Answer: Unanswered
Explanation

Solution (a)

Justice

The term ‘justice’ in the Preamble embraces three distinct forms—social, economic and
political, secured through various provisions of Fundamental Rights and Directive Principles.

Social justice denotes the equal treatment of all citizens without any social distinction based
on caste, colour, race, religion, sex and so on. It means absence of privileges being extended to
any particular section of the society, and improvement in the conditions of backward classes
(SCs, STs and OBCs) and women.

Economic justice denotes the non-discrimination between people on the basis of economic
factors. It involves the elimination of glaring in-equalities in wealth, income and property. A
combination of social justice and economic justice denotes what is known as ‘distributive
justice’.

Political justice implies that all citizens should have equal political rights, equal access to all
political offices and equal voice in the government.

The ideal of justice—social, economic and political—has been taken from the Russian Revolution
(1917).

Think

· Terms used in preamble should be very clear.

QUESTION 10.
The term ‘fraternity’ in Preamble of Indian Constitution signifies which of the following:

1. Spirit of Brotherhood amongst all citizens of India

2. It assures dignity of an Individual

3. It assures unity and integrity of the Nation

Select the code from following:

a) 1 only
b) 2 and 3

IASbaba
Web: http://ilp.iasbaba.com/ Score:
Email: ilp@iasbaba.com 4.00 / 200
Page 9
Exam Title : Test 21-Revision: Polity & ...
Email
Contact :

c) 1 and 3
d) All of the above
Correct Answer: D
Your Answer: Unanswered
Explanation

Solution (d)

Fraternity

Fraternity means a sense of brotherhood. The Constitution promotes this feeling of fraternity by
the system of single citizenship. Also, the Fundamental Duties (Article 51-A) say that it shall be
the duty of every citizen of India to promote harmony and the spirit of common brotherhood
amongst all the people of India transcending religious, linguistic, regional or sectional
diversities.

The Preamble declares that fraternity has to assure two things—the dignity of the individual
and the unity and integrity of the nation.

The word ‘integrity’ has been added to the preamble by the 42 nd Constitutional Amendment
(1976).

According to K M Munshi, a member of the Drafting Committee of the Constituent Assembly,


the phrase ‘dignity of the individual’ signifies that the Constitution not only ensures material
betterment and maintain a democratic set-up, but that it also recognises that the personality of
every individual is sacred. This is highlighted through some of the provisions of the
Fundamental Rights and Directive Principles of State Policy, which ensure the dignity of
individuals. Further, the Fundamental Duties (Article 51A) also protect the dignity of women by
stating that it shall be the duty of every citizen of India to renounce practices derogatory to the
dignity of women, and also makes it the duty of every citizen of India to uphold and protect the
sovereignty, unity and integrity of India.

The phrase ‘unity and integrity of the nation’ embraces both the psychological and territorial
dimensions of national integration. Article 1 of the Constitution describes India as a ‘Union of
States’ to make it clear that the states have no right to secede from the Union, implying the
indestructible nature of the Indian Union. It aims at overcoming hindrances to national
integration like communalism, regionalism, casteism, linguism, secessionism and so on.

QUESTION 11.
Indian Constitution defines India as a ‘Union of States’. The term ‘Union of India’ includes?

1. States

2. Union Territories

3. Territories that may be acquired by Government of India

Select the code from following:

a) 1 only
b) 1 and 2
c) 1,2 and 3

IASbaba
Web: http://ilp.iasbaba.com/ Score:
Email: ilp@iasbaba.com 4.00 / 200
Page 10
Exam Title : Test 21-Revision: Polity & ...
Email
Contact :

d) 1 and 3
Correct Answer: A
Your Answer: Unanswered
Explanation

Solution (a)

Union of India

The ‘Territory of India’ is a wider expression than the ‘Union of India’ because the latter
includes only states while the former includes not only the states but also union territories and
territories that may be acquired by the Government of India at any future time. The states are
the members of the federal system and share a distribution of powers with the Centre. The
union territories and the acquired territories, on the other hand, are directly administered by
the Central government.

Think

· Different meanings of ‘State’ in Indian Constitution.

QUESTION 12.
India is described as ‘Indestructible Union of Destructible States’. Which of the following
statements supports this description?

1. The Parliament can redraw the political map of India according to its will.

2. Consent of States is necessary before changing their boundary or area

3. Formation of new states, altering boundary and name is considered as Constitutional


Amendment under Art 368.

Select the code from following:

a) 1 only
b) 2 and 3
c) 1 and 3
d) All of the above
Correct Answer: A
Your Answer: Unanswered
Explanation

Solution (a)

Article 3 authorises the Parliament to:

(a) form a new state by separation of territory from any state or by uniting two or more states
or parts of states or by uniting any territory to a part of any state,

(b) increase the area of any state,

(c) diminish the area of any state,

(d) alter the boundaries of any state, and

IASbaba
Web: http://ilp.iasbaba.com/ Score:
Email: ilp@iasbaba.com 4.00 / 200
Page 11
Exam Title : Test 21-Revision: Polity & ...
Email
Contact :

(e) alter the name of any state.

However, Article 3 lays down two conditions in this regard: one, a bill contemplating the above
changes can be introduced in the Parliament only with the prior recommendation of the
President; and two, before recommending the bill, the President has to refer the same to the
state legistature concerned for expressing its views within a specified period.

Further, the power of Parliament to form new states includes the power to form a new state or
union territory by uniting a part of any state or union territory to any other state or union
territory.

The President (or Parliament) is not bound by the views of the state legislature and may either
accept or reject them, even if the views are received in time. Further, it is not necessary to
make a fresh reference to the state legislature every time an amendment to the bill is moved
and accepted in Parliament. In case of a union territory, no reference need be made to the
concerned legislature to ascertain its views and the Parliament can itself take any action as it
deems fit.

It is thus clear that the Constitution authorises the Parliament to form new states or alter the
areas, boundaries or names of the existing states without their consent. In other words, the
Parliament can redraw the political map of India according to its will. Hence, the territorial
integrity or continued existence of any state is not guaranteed by the Constitution. Therefore,
India is rightly described as ‘an indestructible union of destructible states’. The Union
government can destroy the states whereas the state governments cannot destroy the Union. In
USA, on the other hand, the territorial integrity or continued existence of a state is guaranteed
by the Constitution. The American Federal government cannot form new states or alter the
borders of existing states without the consent of the states concerned. That is why the USA is
described as ‘an indestructible union of indestructible states.’

Moreover, the Constitution (Article 4) itself declares that laws made for admission or
establishment of new states (under Article 2) and formation of new states and alteration of
areas, boundaries or names of existing states (under Articles 3) are not to be considered as
amendments of the Constitution under Article 368. This means that such laws can be passed by
a simple majority and by the ordinary legislative process.

QUESTION 13.
Which of the following commissions accepted ‘Language’ as the basis for reorganization of
States?

a) Dhar Commission
b) JVP committee
c) Fazal Ali Commission
d) None of the above
Correct Answer: C
Your Answer: Unanswered
Explanation

Solution (c)

Fazl Ali Commission

IASbaba
Web: http://ilp.iasbaba.com/ Score:
Email: ilp@iasbaba.com 4.00 / 200
Page 12
Exam Title : Test 21-Revision: Polity & ...
Email
Contact :

The creation of Andhra state intensified the demand from other regions for creation of states on
linguistic basis. This forced the Government of India to appoint (in December 1953) a three-
member States Reorganisation Commission under the chairmanship of Fazl Ali to re-examine
the whole question. Its other two members were K M Panikkar and H N Kunzru. It submitted its
report in September 1955 and broadly accepted language as the basis of reorganisation of
states. But, it rejected the theory of ‘one language–one state’. Its view was that the unity of
India should be regarded as the primary consideration in any redrawing of the country’s
political units. It identified four major factors that can be taken into account in any scheme of
reorganisation of states:

(a) Preservation and strengthening of the unity and security of the country.

(b) Linguistic and cultural homogeneity.

(c) Financial, economic and administrative considerations.

(d) Planning and promotion of the welfare of the people in each state as well as of the nation as
a whole.

The commission suggested the abolition of the four-fold classification of states under the
original Constitution and creation of 16 states and 3 centrally administered territories. The
Government of India accepted these recommendations with certain minor modifications. By the
States Reorganisation Act (1956) and the 7th Constitutional Amendment Act (1956), the
distinction between Part-A and Part-B states was done away with and Part-C states were
abolished. Some of them were merged with adjacent states and some other were designated as
union territories. As a result, 14 states and 6 union territories were created on November 1,
1956.

Think

· Reorganisation of States

QUESTION 14.
Consider the following statements regarding citizenship in India:

1. Children of Foreign delegates born in India get Indian citizenship.

2. A child born in 2018 outside India will get Indian citizenship automatically by descent if both
its parents are Indians.

Which of the above statements are correct?

a) 1 only
b) 2 only
c) Both 1 and 2
d) Neither 1 nor 2
Correct Answer: D
Your Answer: Unanswered
Explanation

Solution (d)

IASbaba
Web: http://ilp.iasbaba.com/ Score:
Email: ilp@iasbaba.com 4.00 / 200
Page 13
Exam Title : Test 21-Revision: Polity & ...
Email
Contact :

Citizenship in India

The Citizenship Act of 1955 prescribes five ways of acquiring citizenship, viz, birth, descent,
registration, naturalisation and incorporation of territory:

1. By Birth A person born in India on or after 26th January 1950 but before 1st July 1987 is a
citizen of India by birth irrespective of the nationality of his parents.

A person born in India on or after 1st July 1987 is considered as a citizen of India only if either
of his parents is a citizen of India at the time of his birth.

Further, those born in India on or after 3rd December 2004 are considered citizens of India only
if both of their parents are citizens of India or one of whose parents is a citizen of India and the
other is not an illegal migrant at the time of their birth.

The children of foreign diplomats posted in India and enemy aliens cannot acquire Indian
citizenship by birth.

2. By Descent A person born outside India on or after 26th January 1950 but before 10th
December 1992 is a citizen of India by descent, if his father was a citizen of India at the time of
his birth.

A person born outside India on or after 10th December 1992 is considered as a citizen of India
if either of his parents is a citizen of India at the time of his birth.

From 3rd December 2004 onwards, a person born outside India shall not be a citizen of India by
descent, unless his birth is registered at an Indian consulate within one year of the date of birth
or with the permission of the Central Government, after the expiry of the said period. An
application, for registration of the birth of a minor child, to an Indian consulate shall be
accompanied by an undertaking in writing from the parents of such minor child that he or she
does not hold the passport of another country.

Think

· Other ways of acquiring citizenship

QUESTION 15.
Which of the following statements are correct regarding Fundamental Rights?

1. They are negative in character i.e. they put restriction on authority.

2. Aggrieved can directly go to Supreme Court if these rights are violated

3. FRs can’t be amended by the Parliament.

Select the code from following:

a) 1 and 2
b) 2 and 3
c) 1 and 3
d) All of the above
Correct Answer: D

IASbaba
Web: http://ilp.iasbaba.com/ Score:
Email: ilp@iasbaba.com 4.00 / 200
Page 14
Exam Title : Test 21-Revision: Polity & ...
Email
Contact :

Your Answer: Unanswered


Explanation

Solution (d)

FEATURES OF FUNDAMENTAL RIGHTS

The Fundamental Rights guaranteed by the Constitution are characterised by the following:

1. Some of them are available only to the citizens while others are available to all persons
whether citizens, foreigners or legal persons like corporations or companies.

2. They are not absolute but qualified. The state can impose reasonable restrictions on them.
However, whether such restrictions are reasonable or not is to be decided by the courts. Thus,
they strike a balance between the rights of the individual and those of the society as a whole,
between individual liberty and social control.

3. Most of them are available against the arbitrary action of the State, with a few exceptions
like those against the State’s action and against the action of private individuals. When the
rights that are available against the State’s action only are violated by the private individuals,
there are no constitutional remedies but only ordinary legal remedies.

4. Some of them are negative in character, that is, place limitations on the authority of the
State, while others are positive in nature, conferring certain privileges on the persons.

5. They are justiciable, allowing persons to move the courts for their enforcement, if and when
they are violated.

6. They are defended and guaranteed by the Supreme Court. Hence, the aggrieved person can

directly go to the Supreme Court, not necessarily by way of appeal against the judgement of the
high courts.

7. They are not sacrosanct or permanent. The Parliament can curtail or repeal them but only by
a constitutional amendment act and not by an ordinary act. Moreover, this can be done without
affecting the ‘basic structure’ of the Constitution.

8. They can be suspended during the operation of a National Emergency except the rights
guaranteed by Articles 20 and 21. Further, the six rights guaranteed by Article 19 can be
suspended only when emergency is declared on the grounds of war or external aggression (i.e.,
external emergency) and not on the ground of armed rebellion (i.e., internal emergency).

9. Their scope of operation is limited by Article 31A (saving of laws providing for acquisition of
estates, etc.), Article 31B (validation of certain acts and regulations included in the 9th

Schedule) and Article 31C (saving of laws giving effect to certain directive principles).

10. Their application to the members of armed forces, paramilitary forces, police forces,
intelligence agencies and analogous services can be restricted or abrogated by the Parliament
(Article 33).

11. Their application can be restricted while martial law is in force in any area. Martial law
means ‘military rule’ imposed under abnormal circumstances to restore order (Article 34). It is
different from the imposition of national emergency.

12. Most of them are directly enforceable (self-executory) while a few of them can be enforced
on the basis of a law made for giving effect to them. Such a law can be made only by the

IASbaba
Web: http://ilp.iasbaba.com/ Score:
Email: ilp@iasbaba.com 4.00 / 200
Page 15
Exam Title : Test 21-Revision: Polity & ...
Email
Contact :

Parliament and not by state legislatures so that uniformity throughout the country is maintained
(Article 35).

QUESTION 16.
Which of the following Fundamental Rights are available only to Indian Citizens?

1. Equal opportunity in Public employment.

2. No discrimination on the ground of religion, race, caste, sex or place of birth.

3. Protection of language and script

4. Right to establish and administer educational institutions

Select the code from below:

a) 1,2 and 3
b) 2,3 and 4
c) 1,3 and 4
d) All of the above
Correct Answer: D
Your Answer: Unanswered
Explanation

Solution (d)

FR available only to citizens and not to foreigners

· Prohibition of discrimination on grounds of religion, race, caste, sex or place of birth (Article
15).

· Equality of opportunity in matters of public employment (Article 16).

· Protection of six rights regarding freedom of : (i) speech and expression, (ii) assembly, (iii)
association, (iv) movement, (v) residence, and (vi) profession (Article 19).

· Protection of language, script and culture of minorities (Article 29).

· Right of minorities to establish and administer educational institutions (Article 30).

QUESTION 17.
Article 17 of the Constitution abolished ‘Untouchability’. Which of the following statements
regarding article 17 is/are correct?

1. It clearly defines the practice of Untouchability.

2. It does not include social boycott of certain individuals or their exclusion from religious
ceremonies.

Select the code from below:

IASbaba
Web: http://ilp.iasbaba.com/ Score:
Email: ilp@iasbaba.com 4.00 / 200
Page 16
Exam Title : Test 21-Revision: Polity & ...
Email
Contact :

a) 1 only
b) 2 only
c) Both 1 and 2
d) Neither 1 nor 2
Correct Answer: B
Your Answer: Unanswered
Explanation

Solution (b)

Article 17 abolishes ‘untouchability’ and forbids its practice in any form. The enforcement of
any disability arising out of untouchability shall be an offence punishable in accordance with
law.

In 1976, the Untouchability (Offences) Act, 1955 has been comprehensively amended and
renamed as the Protection of Civil Rights Act, 1955 to enlarge the scope and make penal
provisions more stringent.

The term ‘untouchability’ has not been defined either in the Constitution or in the Act. However,
the Mysore High Court held that the subject matter of Article 17 is not untouchability in its
literal or grammatical sense but the ‘practice as it had developed historically in the country’. It
refers to the social disabilities imposed on certain classes of persons by reason of their birth in
certain cases. Hence, it does not cover social boycott of a few individuals or their exclusion
from religious services, etc.

The act declares the following acts as offences:

(a) Preventing any person from entering any place of public worship or from worshipping
therein;

(b) Justifying untouchability on traditional, religious, philosophical or other grounds;

(c) Denying access to any shop, hotel or places of public entertainment;

(d) Insulting a person belonging to scheduled caste on the ground of untouchability;

(e) Refusing to admit persons in hospitals, educational institutions or hostels established for
public benefit;

(f) Preaching untouchability directly or indirectly; and

(g) Refusing to sell goods or render services to any person.

The Supreme Court held that the right under Article 17 is available against private individuals
and it is the constitutional obligation of the State to take necessary action to ensure that this
right is not violated.

QUESTION 18.
Indian Constitution has given the citizens Right to Assembly. Which of the statements are
correct regarding that?

1. Every citizen has right to assemble peacefully and without arms.

IASbaba
Web: http://ilp.iasbaba.com/ Score:
Email: ilp@iasbaba.com 4.00 / 200
Page 17
Exam Title : Test 21-Revision: Polity & ...
Email
Contact :

2. Citizens have right to hold public meetings, demonstrations and take out processions.

3. It includes the right to strike.

Select the code from the following:

a) 1 and 2
b) 2 and 3
c) 1 and 3
d) All of the above
Correct Answer: A
Your Answer: Unanswered
Explanation

Solution (a)

Freedom of Assembly

Every citizen has the right to assemble peaceably and without arms. It includes the right to hold
public meetings, demonstrations and take out processions. This freedom can be exercised only
on public land and the assembly must be peaceful and unarmed. This provision does not protect
violent, disorderly, riotous assemblies, or one that causes breach of public peace or one that
involves arms. This right does not include the right to strike.

The State can impose reasonable restrictions on the exercise of right of assembly on two
grounds, namely, sovereignty and integrity of India and public order including the maintenance
of traffic in the area concerned.

Think

· Jallianwala Bagh

QUESTION 19.
Consider the following statements:

1. No person shall be prosecuted and punished for the same offence more than once.

2. Prevention from retrospective penalties is there only in case of criminal offence and not in
civil cases.

3. No self incrimination extends only to criminal proceeding and not to civil proceedings.

Which of the above statements are correct?

a) 1 and 2
b) 2 and 3
c) 1 and 3
d) All of the above
Correct Answer: D
Your Answer: Unanswered
Explanation

IASbaba
Web: http://ilp.iasbaba.com/ Score:
Email: ilp@iasbaba.com 4.00 / 200
Page 18
Exam Title : Test 21-Revision: Polity & ...
Email
Contact :

Solution (d)

Article 20

Article 20 grants protection against arbitrary and excessive punishment to an accused person,
whether citizen or foreigner or legal person like a company or a corporation. It contains three
provisions in that direction:

(a) No ex-post-facto law: No person shall be (i) convicted of any offence except for violation of
a law in force at the time of the commission of the act, nor (ii) subjected to a penalty greater
than that prescribed by the law in force at the time of the commission of the act.

(b) No double jeopardy : No person shall be prosecuted and punished for the same offence
more than once.

(c) No self-incrimination: No person accused of any offence shall be compelled to be a


witness

against himself.

An ex-post-facto law is one that imposes penalties retrospectively (retroactively), that is, upon
acts already done or which increases the penalties for such acts. The enactment of such a law is
prohibited by the first provision of Article 20. However, this limitation is imposed only on
criminal laws and not on civil laws or tax laws. In other words, a civil liability or a tax can be
imposed retrospectively.

Further, this provision prohibits only conviction or sentence under an ex-post-facto criminal law
and not the trial thereof. Finally, the protection (immunity) under this provision cannot be
claimed in case of preventive detention or demanding security from a person.

The protection against double jeopardy is available only in proceedings before a court of law or
a judicial tribunal. In other words, it is not available in proceedings before departmental or
administrative authorities as they are not of judicial nature.

The protection against self-incrimination extends to both oral evidence and documentary
evidence.

However, it does not extend to (i) compulsory production of material objects, (ii) compulsion to
give thumb impression, specimen signature, blood specimens, and (iii) compulsory exhibition of
the body. Further, it extends only to criminal proceedings and not to civil proceedings or
proceedings which are not of criminal nature.

Think

· Preventive Detention

QUESTION 20.
Which of the following Rights have been brought under the ambit of Article 21?

1. Right to Privacy

2. Right to free legal aid

IASbaba
Web: http://ilp.iasbaba.com/ Score:
Email: ilp@iasbaba.com 4.00 / 200
Page 19
Exam Title : Test 21-Revision: Polity & ...
Email
Contact :

3. Right to travel abroad

4. Right to information

Select the code from below:

a) 1 and 4
b) 2,3 and 4
c) 1,3 and 4
d) All of the above
Correct Answer: D
Your Answer: Unanswered
Explanation

Solution (d)

Article 21

The Supreme Court has reaffirmed its judgement in the Menaka case in the subsequent cases.
It has declared the following rights as part of Article 21:

(a) Right to live with human dignity.

(b) Right to decent environment including pollution free water and air and protection against
hazardous industries.

(c) Right to livelihood.

(d) Right to privacy.

(e) Right to shelter.

(f) Right to health.

(g) Right to free education up to 14 years of age.

(h) Right to free legal aid.

(i) Right against solitary confinement.

(j) Right to speedy trial.

(k) Right against handcuffing.

(l) Right against inhuman treatment.

(m) Right against delayed execution.

(n) Right to travel abroad.

(o) Right against bonded labour.

(p) Right against custodial harassment.

(q) Right to emergency medical aid.

(r) Right to timely medical treatment in government hospital.

IASbaba
Web: http://ilp.iasbaba.com/ Score:
Email: ilp@iasbaba.com 4.00 / 200
Page 20
Exam Title : Test 21-Revision: Polity & ...
Email
Contact :

(s) Right not to be driven out of a state.

(t) Right to fair trial.

(u) Right of prisoner to have necessities of life.

(v) Right of women to be treated with decency and dignity.

(w) Right against public hanging.

(x) Right to hearing.

(y) Right to information.

(z) Right to reputation.

QUESTION 21.
Which of the following statements are correct regarding Preventive Detention?

1. The detention cannot exceed 3 months unless an advisory board reports sufficient cause of
detention.

2. Grounds of detention need not be communicated to the detenu.

3. Detenu is allowed to an opportunity to make a representation against the detention order.

Select the code from following:

a) 1 and 2
b) 2 and 3
c) 1 and 3
d) All of the above
Correct Answer: C
Your Answer: Unanswered
Explanation

Solution (c)

The Article 22 has two parts—the first part deals with the cases of ordinary law and the second
part deals with the cases of preventive detention law.

The second part of Article 22 grants protection to persons who are arrested or detained under a
preventive detention law. This protection is available to both citizens as well as aliens and
includes the following:

(i) The detention of a person cannot exceed three months unless an advisory board reports
sufficient cause for extended detention. The board is to consist of judges of a high court.

(ii) The grounds of detention should be communicated to the detenu. However, the facts
considered to be against the public interest need not be disclosed.

IASbaba
Web: http://ilp.iasbaba.com/ Score:
Email: ilp@iasbaba.com 4.00 / 200
Page 21
Exam Title : Test 21-Revision: Polity & ...
Email
Contact :

(iii) The detenu should be afforded an opportunity to make a representation against the
detention order.

Article 22 also authorises the Parliament to prescribe

(a) The circumstances and the classes of cases in which a person can be detained for more than
three months under a preventive detention law without obtaining the opinion of an advisory
board;

(b) The maximum period for which a person can be detained in any classes of cases under a
preventive detention law; and

(c) The procedure to be followed by an advisory board in an inquiry.

The 44th Amendment Act of 1978 has reduced the period of detention without obtaining the
opinion of an advisory board from three to two months. However, this provision has not yet
been brought into force, hence, the original period of three months still continues.

QUESTION 22.
In which of the following educational institutes religious instruction is completely prohibited?

1. Institutions wholly maintained by state

2. Institutions administered by state but established by trust

3. Institutions recognized by state

4. Institutions receiving aid from State

Select the code from below:

a) 1 only
b) 2,3 and 4
c) 1,2 and 4
d) All of the above
Correct Answer: A
Your Answer: Unanswered
Explanation

Solution (a)

Article 28 distinguishes between four types of educational institutions:

(a) Institutions wholly maintained by the State.

(b) Institutions administered by the State but established under any endowment or trust.

(c) Institutions recognized by the State.

(d) Institutions receiving aid from the State.

IASbaba
Web: http://ilp.iasbaba.com/ Score:
Email: ilp@iasbaba.com 4.00 / 200
Page 22
Exam Title : Test 21-Revision: Polity & ...
Email
Contact :

In (a) religious instruction is completely prohibited while in (b), religious instruction is


permitted. In (c) and (d), religious instruction is permitted on a voluntary basis.

QUESTION 23.
Which of the following statements correctly differentiates between Writ jurisdiction of Supreme
Court and High Court?

1. SC can issue writs only for enforcement of FRs while HC can issue writs for other purposes
also.

2. SC can issue writs throughout the country while HC can issue writs only in its state.

3. SC can’t refuse to exercise its writ jurisdiction while HC can refuse.

Select the code from below:

a) 1 and 2
b) 2 and 3
c) 1 and 3
d) All of the above
Correct Answer: D
Your Answer: Unanswered
Explanation

Solution (d)

The writ jurisdiction of the Supreme Court differs from that of a high court in three respects:

1. The Supreme Court can issue writs only for the enforcement of fundamental rights whereas a
high court can issue writs not only for the enforcement of Fundamental Rights but also for any
other purpose. The expression ‘for any other purpose’ refers to the enforcement of an ordinary
legal right. Thus, the writ jurisdiction of the Supreme Court, in this respect, is narrower than
that of high court.

2. The Supreme Court can issue writs against a person or government throughout the territory
of India whereas a high court can issue writs against a person residing or against a government
or authority located within its territorial jurisdiction only or outside its territorial jurisdiction
only if the cause of action arises within its territorial jurisdiction.15 Thus, the territorial
jurisdiction of the Supreme Court for the purpose of issuing writs is wider than that of a high
court.

3. A remedy under Article 32 is in itself a Fundamental Right and hence, the Supreme Court
may not refuse to exercise its writ jurisdiction. On the other hand, a remedy under Article 226
is discretionary and hence, a high court may refuse to exercise its writ jurisdiction. Article 32
does not merely confer power on the Supreme Court as Article 226 does on a high court to
issue writs for the enforcement of fundamental rights or other rights as part of its general
jurisdiction. The Supreme Court is thus constituted as a defender and guarantor of the
fundamental rights.

IASbaba
Web: http://ilp.iasbaba.com/ Score:
Email: ilp@iasbaba.com 4.00 / 200
Page 23
Exam Title : Test 21-Revision: Polity & ...
Email
Contact :

QUESTION 24.
Which of the following DPSPs are Socialist in nature?

1. To provide free legal Aid

2. Equal pay for equal work

3. To organize Village panchayat

4. To promote cottage industries

Select the code from following:

a) 1 and 2
b) 3 and 4
c) 1,2 and 3
d) All of the above
Correct Answer: A
Your Answer: Unanswered
Explanation

Solution (a)

Socialistic Principles in DPSP

These principles reflect the ideology of socialism. They lay down the framework of a democratic
socialist state, aim at providing social and economic justice, and set the path towards welfare
state.

They direct the state:

1. To promote the welfare of the people by securing a social order permeated by justice—
social, economic and political—and to minimise inequalities in income, status, facilities and
opportunities (Article 38).

2. To secure (a) the right to adequate means of livelihood for all citizens; (b) the equitable
distribution of material resources of the community for the common good; (c) prevention of
concentration of wealth and means of production; (d) equal pay for equal work for men and
women; (e) preservation of the health and strength of workers and children against forcible
abuse; and (f) opportunities for healthy development of children (Article 39).

3. To promote equal justice and to provide free legal aid to the poor6 (Article 39 A).

4. To secure the right to work, to education and to public assistance in cases of unemployment,
old age, sickness and disablement (Article 41).

5. To make provision for just and humane conditions for work and maternity relief (Article 42).

6. To secure a living wage7, a decent standard of life and social and cultural opportunities for
all workers (Article 43).

7. To take steps to secure the participation of workers in the management of industries8 (Article
43 A).

IASbaba
Web: http://ilp.iasbaba.com/ Score:
Email: ilp@iasbaba.com 4.00 / 200
Page 24
Exam Title : Test 21-Revision: Polity & ...
Email
Contact :

8. To raise the level of nutrition and the standard of living of people and to improve public
health (Article 47).

Note: 3 and 4 are based on Gandhian Principles.

QUESTION 25.
Which of the following statements are correct regarding Amendment procedure under Article
368?

1. Amendment bill can be initiated in each house of Parliament and state Legislative
Assemblies.

2. It can be introduced by a minister and not by a private member.

3. If a bill is not passed by Rajya Sabha then a Joint meeting is called.

Which of the above statements are correct?

a) 1 and 2
b) 2 and 3
c) 3 only
d) None of the above
Correct Answer: D
Your Answer: Unanswered
Explanation

Solution (d)

Amendment Procedure

The procedure for the amendment of the Constitution as laid down in Article 368 is as follows:

1. An amendment of the Constitution can be initiated only by the introduction of a bill for the
purpose in either House of Parliament and not in the state legislatures.

2. The bill can be introduced either by a minister or by a private member and does not require
prior permission of the president.

3. The bill must be passed in each House by a special majority, that is, a majority (that is, more
than 50 per cent) of the total membership of the House and a majority of two-thirds of the
members of the House present and voting.

4. Each House must pass the bill separately. In case of a disagreement between the two Houses,
there is no provision for holding a joint sitting of the two Houses for the purpose of deliberation
and passage of the bill.

5. If the bill seeks to amend the federal provisions of the Constitution, it must also be ratified by
the legislatures of half of the states by a simple majority, that is, a majority of the members of
the House present and voting.

6. After duly passed by both the Houses of Parliament and ratified by the state legislatures,
where necessary, the bill is presented to the president for assent.

IASbaba
Web: http://ilp.iasbaba.com/ Score:
Email: ilp@iasbaba.com 4.00 / 200
Page 25
Exam Title : Test 21-Revision: Polity & ...
Email
Contact :

7. The president must give his assent to the bill. He can neither withhold his assent to the bill
nor return the bill for reconsideration of the Parliament.2

8. After the president’s assent, the bill becomes an Act (i.e., a constitutional amendment act)
and the Constitution stands amended in accordance with the terms of the Act.

QUESTION 26.
Which of the following are the features of Federal Government?

1. Dual Government

2. Written Constitution

3. Bicameral Legislature

4. Independent Judiciary

Select the code from below:

a) 1 and 4
b) 2 and 3
c) 1,2 and 3
d) All of the above
Correct Answer: D
Your Answer: Unanswered
Explanation

Solution (d)

Characteristics of Federal Government

· Dual Government (that is, national government and regional government)

· Written Constitution

· Division of powers between the national and regional

· government

· Supremacy of the Constitution

· Rigid Constitution

· Independent judiciary

· Bicameral legislature

QUESTION 27.
Parliament can legislate on State Subject if Rajya Sabha passes a resolution for the same.
Which of the following statements are correct regarding it?

IASbaba
Web: http://ilp.iasbaba.com/ Score:
Email: ilp@iasbaba.com 4.00 / 200
Page 26
Exam Title : Test 21-Revision: Polity & ...
Email
Contact :

1. Resolution must be supported by two thirds of members present and Voting.

2. The resolution remains in force for one year and after that it has to be renewed.

3. The resolution restricts state Legislature to make laws on the same matter.

Select the code from following:

a) 1 and 2
b) 2 and 3
c) 1 and 3
d) All of the above
Correct Answer: A
Your Answer: Unanswered
Explanation

Solution (a)

Parliamentary Legislation in the State Field

The above scheme of distribution of legislative powers between the Centre and the states is to
be maintained in normal times. But, in abnormal times, the scheme of distribution is either
modified or suspended. In other words, the Constitution empowers the Parliament to make laws
on any matter enumerated in the State List under the following five extraordinary
circumstances:

When Rajya Sabha Passes a Resolution

If the Rajya Sabha declares that it is necessary in the national interest that Parliament should
make laws on a matter in the State List, then the Parliament becomes competent to make laws
on that matter. Such a resolution must be supported by two-thirds of the members present and
voting. The resolution remains in force for one year; it can be renewed any number of times but
not exceeding one year at a time. The laws cease to have effect on the expiration of six months
after the resolution has ceased to be in force. This provision does not restrict the power of a
state legislature to make laws on the same matter. But, in case of inconsistency between a state
law and a parliamentary law, the latter is to prevail.

During a National Emergency

The Parliament acquires the power to legislate with respect to matters in the State List, while a
proclamation of national emergency is in operation. The laws become inoperative on the
expiration of six months after the emergency has ceased to operate. Here also, the power of a
state legislature to make laws on the same matter is not restricted. But, in case of repugnancy
between a state law and a parliamentary law, the latter is to prevail.

When States Make a Request

When the legislatures of two or more states pass resolutions requesting the Parliament to enact
laws on a matter in the State List, then the Parliament can make laws for regulating that
matter. A law so enacted applies only to those states which have passed the resolutions.
However, any other state may adopt it afterwards by passing a resolution to that effect in its
legislature. Such a law can be amended or repealed only by the Parliament and not by the
legislatures of the concerned states. The effect of passing a resolution under the above
provision is that the Parliament becomes entitled to legislate with respect to a matter for which
it has no power to make a law. On the other hand, the state legislature ceases to have the power

IASbaba
Web: http://ilp.iasbaba.com/ Score:
Email: ilp@iasbaba.com 4.00 / 200
Page 27
Exam Title : Test 21-Revision: Polity & ...
Email
Contact :

to make a law with respect to that matter. The resolution operates as abdication or surrender of
the power of the state legislature with respect to that matter and it is placed entirely in the
hands of Parliament which alone can then legislate with respect to it.

Some examples of laws passed under the above provision are Prize Competition Act, 1955; Wild

Life (Protection) Act, 1972; Water (Prevention and Control of Pollution) Act, 1974; Urban Land

(Ceiling and Regulation) Act, 1976; and Transplantation of Human Organs Act, 1994.

To Implement International Agreements

The Parliament can make laws on any matter in the State List for implementing the
international treaties, agreements or conventions. This provision enables the Central
government to fulfill its international obligations and commitments.

Some examples of laws enacted under the above provision are United Nations (Privileges and

Immunities) Act, 1947; Geneva Convention Act, 1960; Anti-Hijacking Act, 1982 and legislations
relating to environment and TRIPS.

During President’s Rule

When the President’s rule is imposed in a state, the Parliament becomes empowered to make
laws with respect to any matter in the State List in relation to that state. A law made so by the
Parliament continues to be operative even after the president’s rule. This means that the period
for which such a law remains in force is not co-terminus with the duration of the President’s
rule. But, such a law can be repealed or altered or re-enacted by the state legislature.

QUESTION 28.
Constitution has provided for a Finance Commission under Article 280. Which of the following
statements regarding Finance Commission are correct?

1. It is constituted by the President every fifth year or even earlier.

2. It recommends for distribution of taxes between center and state.

3. The recommendations of Finance Commission are binding on the Government.

Select the code from below:

a) 1 and 2
b) 2 and 3
c) 1 and 3
d) All of the above
Correct Answer: A
Your Answer: Unanswered
Explanation

Solution (a)

Finance Commission

IASbaba
Web: http://ilp.iasbaba.com/ Score:
Email: ilp@iasbaba.com 4.00 / 200
Page 28
Exam Title : Test 21-Revision: Polity & ...
Email
Contact :

Article 280 provides for a Finance Commission as a quasi-judicial body. It is constituted by the
President every fifth year or even earlier. It is required to make recommendations to the
President on the following matters:

• The distribution of the net proceeds of taxes to be shared between the Centre and the states,
and the allocation between the states, the respective shares of such proceeds.

• The principles which should govern the grants-in-aid to the states by the Centre (i.e., out of
the Consolidated Fund of India).

• The measures needed to augment the consolidated fund of a state to supplement the
resources of the panchayats and the municipalities in the state on the basis of the
recommendations made by the State Finance Commission.

• Any other matter referred to it by the President in the interests of sound finance.

Till 1960, the Commission also suggested the amounts paid to the States of Assam, Bihar,
Orissa and West Bengal in lieu of assignment of any share of the net proceeds in each year of
export duty on jute and jute products.

The Constitution envisages the Finance Commission as the balancing wheel of fiscal federalism
in India. However, its role in the Centre–state fiscal relations has been undermined by the
emergence of the planning commission, a non-constitutional and non-statutory body.

QUESTION 29.
Which of the following Commissions were formed to examine the Center State Relations?

1. Dhar Commission

2. Rajamannar Commission

3. Punchi Commission

4. Sarkaria Commission

Select the code from following:

a) 1,2 and 3
b) 2,3 and 4
c) 1,2 and 4
d) All of the above
Correct Answer: B
Your Answer: Unanswered
Explanation

Solution (b)

The issues in Centre-State relations have been under consideration since the mid 1960s. In this
direction, the following commissions have been formed:

· Administrative Reforms Commission

IASbaba
Web: http://ilp.iasbaba.com/ Score:
Email: ilp@iasbaba.com 4.00 / 200
Page 29
Exam Title : Test 21-Revision: Polity & ...
Email
Contact :

· Rajamannar Committee

· Anandpur Sahib Resolution

· West Bengal Memorandum

· Sarkaria Commission

· Punchhi Commission

QUESTION 30.
Which of the following statements regarding National Emergency is/are correct?

1. Article 359 suspends the Fundamental Rights in case of National Emergency.

2. Article 20 and 21 are restricted during National Emergency.

Select the code from below:

a) 1 only
b) 2 only
c) Both 1 and 2
d) Neither 1 nor 2
Correct Answer: D
Your Answer: Unanswered
Explanation

Solution (d)

Distinction between Articles 358 and 359

The differences between Articles 358 and 359 can be summarized as follows:

1. Article 358 is confined to Fundamental Rights under Article 19 only whereas Article 359
extends to all those Fundamental Rights whose enforcement is suspended by the Presidential
Order.

2. Article 358 automatically suspends the fundamental rights under Article 19 as soon as the
emergency is declared. On the other hand, Article 359 does not automatically suspend any
Fundamental Right. It only empowers the president to suspend the enforcement of the specified
Fundamental Rights.

3. Article 358 operates only in case of External Emergency (that is, when the emergency is
declared on the grounds of war or external aggression) and not in the case of Internal
Emergency (ie, when the Emergency is declared on the ground of armed rebellion). Article

359, on the other hand, operate in case of both External Emergency as well as Internal
Emergency.

4. Article 358 suspends Fundamental Rights under Article 19 for the entire duration of
Emergency while Article 359 suspends the enforcement of Fundamental Rights for a period

IASbaba
Web: http://ilp.iasbaba.com/ Score:
Email: ilp@iasbaba.com 4.00 / 200
Page 30
Exam Title : Test 21-Revision: Polity & ...
Email
Contact :

specified by the president which may either be the entire duration of Emergency or a shorter
period.

5. Article 358 extends to the entire country whereas Article 359 may extend to the entire
country or a part of it.

6. Article 358 suspends Article 19 completely while Article 359 does not empower the
suspension of the enforcement of Articles 20 and 21.

7. Article 358 enables the State to make any law or take any executive action inconsistent with

Fundamental Rights under Article 19 while Article 359 enables the State to make any law or
take any executive action inconsistent with those Fundamental Rights whose enforcement is
suspended by the Presidential Order.

There is also a similarity between Article 358 and Article 359. Both provide immunity from
challenge to only those laws which are related with the Emergency and not other laws. Also, the
executive action taken only under such a law is protected by both.

The 44 th Amendment Act of 1978 restricted the scope of Article 359 in two ways -

· Firstly, the President cannot suspend the right to move the Court for the enforcement of
fundamental rights guaranteed by Articles 20 to 21. In other words, the right to protection in
respect of conviction for offences (Article 20) and the right to life and personal liberty (Article
21) remain enforceable even during emergency.

· Secondly, only those laws which are related with the emergency are protected from being
challenged and not other laws and the executive action taken only under such a law, is
protected.

QUESTION 31.
Which of the following qualifications are required to contest for Presidential Elections in India?

1. A person should be a citizen of India by Birth.

2. The person should have completed 35 years of age

3. The person should be qualified for election as a member of Lok Sabha

4. He should not hold any office of profit.

Select the code from following:

a) 1,2 and 3
b) 2,3 and 4
c) 1,3 and 4
d) All of the above
Correct Answer: B
Your Answer: Unanswered
Explanation

Solution (b)

IASbaba
Web: http://ilp.iasbaba.com/ Score:
Email: ilp@iasbaba.com 4.00 / 200
Page 31
Exam Title : Test 21-Revision: Polity & ...
Email
Contact :

Qualifications for Election as President

A person to be eligible for election as President should fulfill the following qualifications:

1. He should be a citizen of India.

2. He should have completed 35 years of age.

3. He should be qualified for election as a member of the Lok Sabha.

4. He should not hold any office of profit under the Union government or any state government
or any local authority or any other public authority. A sitting President or Vice-President of the

Union, the Governor of any state and a minister of the Union or any state is not deemed to hold
any office of profit and hence qualified as a presidential candidate.

Further, the nomination of a candidate for election to the office of President must be subscribed
by at least 50 electors as proposers and 50 electors as seconders. Every candidate has to make
a security deposit of Rs 15,000 in the Reserve Bank of India. The security deposit is liable to be
forefeited in case the candidate fails to secure one-sixth of the votes polled.

Think

· Qualifications for election of Vice President

QUESTION 32.
Which of the following Veto Power is not enjoyed by the President of India?

a) Absolute Veto
b) Qualified Veto
c) Suspensive Veto
d) Pocket Veto
Correct Answer: B
Your Answer: Unanswered
Explanation

Solution (b)

The veto power enjoyed by the executive in modern states can be classified into the following
four types:

1. Absolute veto that is, withholding of assent to the bill passed by the legislature.

2. Qualified veto, which can be overridden by the legislature with a higher majority.

3. Suspensive veto, which can be over ridden by the legislature with an ordinary majority.

4. Pocket veto that is, taking no action on the bill passed by the legislature.

Of the above four, the President of India is vested with three—absolute veto, suspensive veto
and pocket veto. There is no qualified veto in the case of Indian President ; it is possessed
by the American President.

IASbaba
Web: http://ilp.iasbaba.com/ Score:
Email: ilp@iasbaba.com 4.00 / 200
Page 32
Exam Title : Test 21-Revision: Polity & ...
Email
Contact :

QUESTION 33.
Which of the following statements correctly differentiates between the pardoning power of the
President and a Governor?

1. The President can pardon sentences inflicted by court martial while the governor cannot.

2. The President can pardon death sentence while governor cannot.

Select the code from below:

a) 1 only
b) 2 only
c) Both 1 and 2
d) Neither 1 nor 2
Correct Answer: C
Your Answer: Unanswered
Explanation

Solution (c)

Difference in pardoning powers

The pardoning power of the governor differs from that of the President in following two
respects:

1. The President can pardon sentences inflicted by court martial (military courts) while the
governor cannot.

2. The President can pardon death sentence while governor cannot. Even if a state law
prescribes death sentence, the power to grant pardon lies with the President and not the
governor. However, the governor can suspend, remit or commute a death sentence. In other
words, both the governor and the President have concurrent power in respect of suspension,
remission and commutation of death sentence.

QUESTION 34.
Consider the following statements regarding the upper house of the Parliament:

1. The representatives of states are elected by the elected members of state legislative
assemblies.

2. All states are given equal representation in Rajya Sabha.

3. Union Territories are not represented in Rajya Sabha

Which of the above statements is/are correct?

a) 1 only
b) 2 and 3
c) 1 and 3

IASbaba
Web: http://ilp.iasbaba.com/ Score:
Email: ilp@iasbaba.com 4.00 / 200
Page 33
Exam Title : Test 21-Revision: Polity & ...
Email
Contact :

d) All of the above


Correct Answer: A
Your Answer: Unanswered
Explanation

Solution (a)

Composition of Rajya Sabha

The maximum strength of the Rajya Sabha is fixed at 250, out of which, 238 are to be the
representatives of the states and union territories (elected indirectly) and 12 are nominated by
the president.

At present, the Rajya Sabha has 245 members. Of these, 229 members represent the states, 4
members represent the union territories and 12 members are nominated by the president.

The Fourth Schedule of the Constitution deals with the allocation of seats in the Rajya Sabha to
the states and union territories.

1. Representation of States

The representatives of states in the Rajya Sabha are elected by the elected members of state
legislative assemblies. The election is held in accordance with the system of proportional
representation by means of the single transferable vote. The seats are allotted to the states in
the Rajya Sabha on the basis of population. Hence, the number of representatives varies from
state to state. For example, Uttar Pradesh has 31 members while Tripura has 1 member only.

However, in USA, all states are given equal representation in the Senate irrespective of their
population. USA has 50 states and the Senate has 100 members—2 from each state.

2. Representation of Union Territories

The representatives of each union territory in the Rajya Sabha are indirectly elected by
members of an electroral college specially constituted for the purpose. This election is also held
in accordance with the system of proportional representation by means of the single
transferable vote. Out of the seven union territories, only two (Delhi and Puducherry) have
representation in Rajya Sabha. The populations of other five union territories are too small to
have any representative in the Rajya Sabha.

3. Nominated Members

The president nominates 12 members to the Rajya Sabha from people who have special
knowledge or practical experience in art, literature, science and social service. The rationale
behind this principle of nomination is to provide eminent persons a place in the Rajya Sabha
without going through the process of election. It should be noted here that the American Senate
has no nominated members.

QUESTION 35.
The Speaker of Lok Sabha can resign from his post by writing to

a) President
b) Vice President

IASbaba
Web: http://ilp.iasbaba.com/ Score:
Email: ilp@iasbaba.com 4.00 / 200
Page 34
Exam Title : Test 21-Revision: Polity & ...
Email
Contact :

c) Prime Minister
d) Deputy Speaker
Correct Answer: D
Your Answer: Unanswered
Explanation

Solution (d)

Tenure of a speaker

The Speaker remains in office during the life of the Lok Sabha. However, he has to vacate his
office earlier in any of the following three cases:

1. If he ceases to be a member of the Lok Sabha;

2. If he resigns by writing to the Deputy Speaker; and

3. If he is removed by a resolution passed by a majority of all the members of the Lok Sabha.

Such a resolution can be moved only after giving 14 days’ advance notice. When a resolution for
the removal of the Speaker is under consideration of the House, he cannot preside at the sitting
of the House, though he may be present. However, he can speak and take part in the
proceedings of the House at such a time and vote in the first instance, though not in the case of
an equality of votes.

Think

· Removal of chairman of Rajya Sabha

QUESTION 36.
Center for Development of Advance Computing (C – DAC) has developed a mobile based
application ‘Darshak’. Which of the following statements is/are correct regarding this
application?

1. It is aimed at improving the museum visit experience among the differently-abled.

2. It allows real-time museum visitors gather all details about objects or artifacts simply by
scanning a QR code placed near the object.

Select the code from following:

a) 1 only
b) 2 only
c) Both 1 and 2
d) Neither 1 nor 2
Correct Answer: C
Your Answer: Unanswered
Explanation

Solution (c)

IASbaba
Web: http://ilp.iasbaba.com/ Score:
Email: ilp@iasbaba.com 4.00 / 200
Page 35
Exam Title : Test 21-Revision: Polity & ...
Email
Contact :

Darshak

• It is a mobile-based application. It is aimed at improving the museum visit experience


among the differently-abled.
• It allows real-time museum visitors gather all details about objects or artifacts simply by
scanning a QR code placed near the object.

Jatan

• The objective of the software is to make a digital imprint of all the objects preserved in
museums. It will help researchers, curators and also people interested in the field.

Think

· C – DAC

· Tourism in India

QUESTION 37.
Central Government has started a new scheme named ‘Atal Bhujal Yojana’. Which of the
following statements is/are correct regarding this scheme is/are correct?

1. It is aimed at efficient management of river water with local participation.

2. Half of the cost will funded by World Bank through loan and fifty percent will be funded by
Central Government.

Select the code from following:

a) 1 only
b) 2 only
c) Both 1 and 2
d) Neither 1 nor 2
Correct Answer: B
Your Answer: Unanswered
Explanation

Solution (b)

Atal Bhujal Yojana

• It is aimed at efficient management of available water resources and strengthening of


recharge mechanism through community participation.
• The emphasis of the scheme will be on recharge (of ground water sources) and efficient
use of water by involving people at the local level.
• The Atal Bhujal Yojana will be launched in Gujarat, Maharashtra, Haryana, Karnataka,
Rajasthan, Uttar Pradesh and Madhya Pradesh
• Half of the total cost of this central scheme, named Atal Bhujal Yojana, will be supported
by the World Bank as loan while the remaining half (Rs 3,000 crore) will be funded by the
government.

IASbaba
Web: http://ilp.iasbaba.com/ Score:
Email: ilp@iasbaba.com 4.00 / 200
Page 36
Exam Title : Test 21-Revision: Polity & ...
Email
Contact :

• The government plans to give 50% of the money to states, including gram panchayats, as
incentives for achieving targets in groundwater management. That’s a first-ever move to
encourage community participation and behavioural changes.
• The remaining 50% of the funds will be given to states for strengthening institutional
arrangements such as providing a strong database and scientific approach to help them
accomplish sustainable management of groundwater.
• Gram panchayats that prepare water security plans and put infrastructure to augment
water supply will get incentives. The more steps a state takes to augment supply, more
incentives it will get. The idea is to bring in a concept of self-regulation through
community participation.

Do You Know?

• The ministry had initially conceptualised the National Groundwater Management


Improvement Scheme (NGMIS) in response to the budget announcement of 2016-17.
• However, the NGMIS was shot down by the expenditure finance committee in May 2017,
recommending that the ministry recast the scheme with a composite proposal.
• The scheme has since been recast and renamed as Atal Bhujal Yojana.

https://timesofindia.indiatimes.com/home/environment/centre-pushes-rs-6000cr-plan-to-tackle-
water-depletion/articleshow/62923993.cms

QUESTION 38.
A new class of antibiotics called Malacidins has been recently discovered. Which of the
following statements is/are correct regarding this drug?

1. They are produced by microorganisms living in soil and dirt.

2. They are active against multi drug resistant pathogens.

Select the code from following:

a) 1 only
b) 2 only
c) Both 1 and 2
d) Neither 1 nor 2
Correct Answer: C
Your Answer: Unanswered
Explanation

Solution (c)

Malacidins

• They are a distinctive class of antibiotics that are commonly encoded in soil microbiomes
but have never been reported in culture-based NP (Natural Products) discovery efforts
• They are active against multidrug-resistant pathogens, sterilise methicillin-resistant
Staphylococcus aureus (MRSA) skin infections in an animal wound model and did not
select for resistance (i.e., did not trigger resistance) under laboratory conditions

IASbaba
Web: http://ilp.iasbaba.com/ Score:
Email: ilp@iasbaba.com 4.00 / 200
Page 37
Exam Title : Test 21-Revision: Polity & ...
Email
Contact :

• Malacidins only target gram-positive bacteria.

Do You Know?

• Gram-negative bacteria can cause cholera, pneumonia, sexually transmitted diseases, and
plague.
• Gram negative cell walls contain a thin peptidoglycan layer (without techoic acids) that is
surrounded by a thick plasma membrane.
• Gram positive bacteria will stain purple because of their thick peptidoglycan cell wall.

Think

• Superbugs

http://indianexpress.com/article/explained/hunt-for-new-antibiotics-hits-pay-dirt-5064385/

QUESTION 39.
India’s first ever radio festival was held recently in which of the following places?

a) Hyderabad
b) Chennai
c) Bangalore
d) Delhi
Correct Answer: D
Your Answer: Unanswered
Explanation

Solution (d)

India’s first radio festival was recently held in Delhi. The festival was organised by the
International Association of Women in Radio and Television, in collaboration with UNESCO.

Think

· Prasar Bharti

· Community Radio

QUESTION 40.
Which of the following statements correctly defines ‘Vientiane Vision’?

a) It is a guiding principle for Japan’s defence cooperation with ASEAN


b) It is China’s policy to contain Vietnam.
c) It is the guiding principle of SCO.
d) It is the name given to China’s economic policy to improve economic ties with CIS.
Correct Answer: A
Your Answer: Unanswered
Explanation

IASbaba
Web: http://ilp.iasbaba.com/ Score:
Email: ilp@iasbaba.com 4.00 / 200
Page 38
Exam Title : Test 21-Revision: Polity & ...
Email
Contact :

Solution (a)

Vientiane Vision

• It is a guiding principle for Japan’s defence cooperation with ASEAN, announced as


Japan’s own initiative by Defence Minister Inada at the second ASEAN-Japan Defence
Ministers' Informal Meeting held in Vientiane, Lao PDR in 16 November 2016.
• The vision for the first time shows, in a transparent manner, the full picture of the future
direction of defence cooperation with the ASEAN as a whole in the priority fields.

QUESTION 41.
The Global Antimicrobial Resistance Surveillance System (GLASS) is a platform for global data
sharing on antimicrobial resistance worldwide. Which of the following Organisations have
launched it?

a) UNICEF
b) WHO
c) World Economic Forum
d) Red Cross Society
Correct Answer: B
Your Answer: Unanswered
Explanation

Solution (b)

Global Antimicrobial Resistance Surveillance System (GLASS)

• The Global Antimicrobial Resistance Surveillance System (GLASS) is a platform for global
data sharing on antimicrobial resistance worldwide.
• It has been launched by WHO as part of the implementation of the Global Action Plan on
Antimicrobial Resistance (AMR).
• The data generated will help to inform national, regional and global decision-making,
strategies and advocacy.
• The system aims to foster national AMR surveillance systems and to enable the collection,
integrated analysis and sharing of standardized and validated data on antimicrobial
resistance, captured by participating countries around the world.
• Epidemiological and microbiological information will be combined to enhance
understanding of the extent and impact of AMR on populations, to monitor trends, to
detect emerging resistance and to measure the effectiveness of interventions to control
AMR.

Benefits

• Capacity building for national AMR surveillance


• An implementation package including surveillance software
• Access to a web-based platform for data sharing, data management and reporting
• Assistance with monitoring and evaluation
• Support from a global network of WHO Collaborating Centres

IASbaba
Web: http://ilp.iasbaba.com/ Score:
Email: ilp@iasbaba.com 4.00 / 200
Page 39
Exam Title : Test 21-Revision: Polity & ...
Email
Contact :

• Regular reports on global AMR situation and trends

Think

• Global action plan on AMR


• Strategic and Technical Advisory Group (STAG) on antimicrobial resistance

QUESTION 42.
Consider the following statements regarding Society for Worldwide Interbank Financial
Telecommunications (SWIFT):

1. It is a messaging system used by banks the world over to send information and instructions
in an encrypted format through a secure channel.

2. It helps in day to day fund transfer for short term interbank loans.

3. It is headquartered in Belgium.

Which of the above statements are correct?

a) 1 and 2
b) 2 and 3
c) 1 and 3
d) All of the above
Correct Answer: C
Your Answer: Unanswered
Explanation

Solution (c)

Society for Worldwide Interbank Financial Telecommunications (SWIFT)

• It is a messaging system used by banks the world over to send information and
instructions in an encrypted format through a secure channel.
• It is a messaging system, and does not hold money or manage accounts.
• It does not facilitate funds transfer: rather, it sends payment orders, which must be settled
by correspondent accounts that the institutions have with each other.
• It is headquartered in Belgium
• SWIFT hosts an annual conference every year, called Sibos, specifically aimed at the
financial services industry.

Working

• Suppose a customer of say, a State Bank of India branch in New Delhi wants to send
money to her friend who has an account at a Citibank branch in London.
• The person in New Delhi needs to provide the SBI branch with his friend’s account
number and Citibank’s unique SWIFT code for its London branch. SBI will send a payment
transfer SWIFT message to the Citibank branch over the secure SWIFT network.
• Once Citibank receives the SWIFT message about the incoming payment, it will clear and
credit the money to the London friend’s account.

Do You Know?

IASbaba
Web: http://ilp.iasbaba.com/ Score:
Email: ilp@iasbaba.com 4.00 / 200
Page 40
Exam Title : Test 21-Revision: Polity & ...
Email
Contact :

A core banking system is a software that supports the daily transactions and accounts within a
bank internally. It lets customers perform basic transactions from any member branch office of
the bank.

http://indianexpress.com/article/india/17-months-before-pnb-scandal-rbi-had-cautioned-against-
swift-abuse-at-union-bank-just-in-time-nirav-modi-5070570/

QUESTION 43.
World Information Technology Congress (WITC) 2018 is going to be held in India. This is for the
first time India will be hosting it since its inception. Which of the following statements is/are
correct regarding WITC?

1. It aims provide single platform to IT experts, policy and decision makers and Government
officials from all over the world together to discuss various challenges and possible solutions to
them.

2. It is an annual event being held since 1978.

Select the code from following:

a) 1 only
b) 2 only
c) Both 1 and 2
d) Neither 1 nor 2
Correct Answer: A
Your Answer: Unanswered
Explanation

Solution (a)

World Information Technology Congress (WITC) 2018

• It is a biennial event and considered as the biggest event of its kind. It aims provide single
platform to IT experts, policy and decision makers and Government officials from all over
the world together to discuss various challenges and possible solutions to them
• It was first held in 1978 since then held after every two years

Where – Hyderabad

Theme - Future Enterprises

This is for first time India is hosting WITC. It will second time that the event will be taking place
in Asia after it was hosted in Kualalumpur, Malaysia

Nasscom is the host of the of WCIT

http://www.business-standard.com/article/economy-policy/modi-to-address-world-congress-on-
information-technology-in-hyderabad-today-118021800207_1.html

IASbaba
Web: http://ilp.iasbaba.com/ Score:
Email: ilp@iasbaba.com 4.00 / 200
Page 41
Exam Title : Test 21-Revision: Polity & ...
Email
Contact :

QUESTION 44.
Cabinet has recently approved the formation of Tribunal to settle Mahanadi water dispute. The
dispute is between

a) Madhya Pradesh and Chattisgarh


b) Chattisgarh and Odisha
c) Odisha and Jharkhand
d) Jharkhand and Chattisgarh
Correct Answer: B
Your Answer: Unanswered
Explanation

Solution (b)

Tribunal to settle Mahanadi water dispute

Cabinet approved the setting up of a tribunal to settle a row between Odisha and Chhattisgarh
on sharing the waters of the river Mahanadi.

Highlights

• The tribunal is expected to determine water sharing among basin States on the basis of
the overall availability of water in the complete Mahanadi basin, the contribution of each
State, the present utilisation of water resource in each State and the potential for future
development
• As per provisions of the Inter-State River Water Disputes (ISRWD) Act, 1956, the Tribunal
shall consist of a Chairman and two other Members nominated by the Chief Justice of
India from amongst the Judges of the Supreme Court or High Court.
• As per provisions of the ISRWD Act, 1956 the Tribunal is required to submit its report and
decision within a period of 3 years which can be extended to a further period not
exceeding 2 years due to unavoidable reasons.

IASbaba
Web: http://ilp.iasbaba.com/ Score:
Email: ilp@iasbaba.com 4.00 / 200
Page 42
Exam Title : Test 21-Revision: Polity & ...
Email
Contact :

Think

• Inter-State River Water Disputes (Amendment) Bill

http://pib.nic.in/PressReleseDetail.aspx?PRID=1521034

QUESTION 45.

India has been ranked 81 st in the Global Corruption Perception Index (GCPI). Which of the
following statements regarding GCPI are correct?

1. It is released by Amnesty International

2. The index uses a scale of 0 to 100, where 0 is very clean and 100 is highly corrupt.

Select the code from below:

a) 1 only
b) 2 only
c) Both 1 and 2
d) Neither 1 nor 2
Correct Answer: D
Your Answer: Unanswered
Explanation

Solution (d)

Global corruption perception index

• India has been ranked 81st in the global corruption perception index for 2017, released by
Transparency International
• India was named among the “worst offenders” in terms of graft and press freedom in the
Asia Pacific region.
• The index uses a scale of 0 to 100, where 0 is highly corrupt and 100 is very clean. India’s
score in the latest ranking, however, remained unchanged at 40.

Think

· Transparency International

http://indianexpress.com/article/india/india-ranks-81st-in-global-corruption-perception-
index-5073800/

QUESTION 46.
Consider the following statements regarding Clinical Establishments (Registration and
Regulation) Act, 2010:

1. The Act requires all clinical establishments (except military) to register themselves and
provides a set of standard treatment guidelines for common diseases and conditions.

IASbaba
Web: http://ilp.iasbaba.com/ Score:
Email: ilp@iasbaba.com 4.00 / 200
Page 43
Exam Title : Test 21-Revision: Polity & ...
Email
Contact :

2. It is mandatory for all states and Union territory to implement this Act.

3. The Act is applicable to all types (both therapeutic and diagnostic types) of clinical
establishments from the public and private sectors, belonging to all recognized systems of
medicine, including single doctor clinics.

Which of the above statements are correct?

a) 1 and 2
b) 2 and 3
c) 1 and 3
d) All of the above
Correct Answer: C
Your Answer: Unanswered
Explanation

Solution (c)

Clinical Establishments (Registration and Regulation) Act, 2010

• It has been enacted by the Central Government to provide for registration and regulation
of all clinical establishments in the country with a view to prescribing the minimum
standards of facilities and services provided by them
• The Act requires all clinical establishments to register themselves and provides a set of
standard treatment guidelines for common diseases and conditions.
• The Act cannot be directly applied to all states of India. The states have the choice of
passing a resolution to adopt the bill or passing a similar bill.
• With the exception of the establishments under the military forces, all public and private
establishments, including AYUSH establishments, are required to register.
• The Act is applicable to all types (both therapeutic and diagnostic types) of clinical
establishments from the public and private sectors, belonging to all recognized systems of
medicine, including single doctor clinics.
• The Act lays down establishment for the a Council Body called The National Council for
Clinical Establishment which is responsible primarily for setting up standards for ensuring
proper healthcare by the clinical establishment and develop the minimum standards and
their periodic review.

Do You Know?

• The Clinical Establishments (Registration and Regulation) Act, 2010 has been in effect in
the four states—Arunachal Pradesh, Himachal Pradesh, Mizoram, Sikkim—and all Union
Territories except Delhi since 1 March 2012.
• Uttar Pradesh, Uttarakhand, Rajasthan, Bihar, Jharkhand and Assam have also adopted
the Act under clause (1) of article 252 of the Constitution.
• Article 47 of the Constitution lays down a responsibility upon the State for aiming at
improvement in public health and shall consider this responsibility as among its primary
duties in particular, the State shall endeavour to bring about prohibition of the
consumption except for medicinal purposes of intoxicating drinks and of drugs which are
injurious to health.

Think

• Article 252 of the Constitution

IASbaba
Web: http://ilp.iasbaba.com/ Score:
Email: ilp@iasbaba.com 4.00 / 200
Page 44
Exam Title : Test 21-Revision: Polity & ...
Email
Contact :

http://www.livemint.com/Politics/jVlkSNP683orAm5qDe4POK/Complaints-over-medical-bills-
Centre-seeks-states-response.html

QUESTION 47.
Electoral Bond Scheme has been started to regulate the Political Party funding. Which of the
following statements regarding Electoral bonds are correct?

1. They can be purchased by a person who is a citizen of India or incorporated or established in


India.

2. The Bond shall be encashed by an eligible political party only through a bank account with
the authorized bank.

3. Electoral Bond shall be valid for fifteen days from the date of issue and no payment shall be
made to any payee Political Party if the Bond is deposited after expiry of the validity period.

Select the code from below:

a) 1 and 2
b) 2 and 3
c) 1 and 3
d) All of the above
Correct Answer: D
Your Answer: Unanswered
Explanation

Solution (d)

Electoral Bond Scheme 2018

• Electoral Bonds may be purchased by a person, who is a citizen of India or incorporated


or established in India.
• A person being an individual can buy Electoral Bonds, either singly or jointly with other
individuals. Only the Political Parties registered under section 29A of the Representation
of the People Act, 1951 (43 of 1951) and which secured not less than one per cent of the
votes polled in the last general election to the House of the People or the Legislative
Assembly of the State, shall be eligible to receive the Electoral Bonds.
• The Bond shall be encashed by an eligible political party only through a bank account with
the authorized bank.
• It may be noted that Electoral Bond shall be valid for fifteen days from the date of issue
and no payment shall be made to any payee Political Party if the Bond is deposited after
expiry of the validity period.
• The bond deposited by any eligible political party to its account shall be credited on the
same day.
• The first sale of Electoral Bonds will commence from 01st March 2018 for a period of 10
days i.e. up to 10th March 2018.

Think

• Recent Electoral Reforms

IASbaba
Web: http://ilp.iasbaba.com/ Score:
Email: ilp@iasbaba.com 4.00 / 200
Page 45
Exam Title : Test 21-Revision: Polity & ...
Email
Contact :

QUESTION 48.
The policy of the Kingdom of Saudi Arabia implemented by its Ministry of Labour, whereby
Saudi companies and enterprises are required to fill up their workforce with Saudi nationals up
to certain levels is called

a) Nitaqat System
b) Liyaqat System
c) Himaqat System
d) Nazakat System
Correct Answer: A
Your Answer: Unanswered
Explanation

Solution (a)

Nitaqat system

• Saudization, officially known as Saudi nationalization scheme, or Nitaqat system in Arabic,


is the policy of the Kingdom of Saudi Arabia implemented by its Ministry of Labour,
whereby Saudi companies and enterprises are required to fill up their workforce with
Saudi nationals up to certain levels.

QUESTION 49.
Echidnas are robust creatures and found in various environments. Which of the following
statements are correct regarding Echidnas?

1. They usually have much lower body temperatures than other mammals

2. Their back feet points backwards which help them to dig burrows.

3. They lay eggs but keep their young ones in the mother’s pouch.

Select the code from below:

a) 1 and 2
b) 2 and 3
c) 1 and 3
d) All of the above
Correct Answer: D
Your Answer: Unanswered
Explanation

Solution (d)

Echidna

• Echidnas are robust and are found in wildly different environments, from the desert to the
snow, likely to having much lower body temperature than all other mammals — around
30C — which can fluctuate by up to 10C in a single day.

IASbaba
Web: http://ilp.iasbaba.com/ Score:
Email: ilp@iasbaba.com 4.00 / 200
Page 46
Exam Title : Test 21-Revision: Polity & ...
Email
Contact :

• Echidnas can travel great distances — often several km a day


• Echidnas lay eggs but keep their young — puggles — in the mother’s pouch
• The back feet of the echidnas point backwards to help them dig their burrows
• They are found in places such as Australia and New Guinea
• Echidnas are one of the world’s oldest surviving mammals
• They are smart, having the biggest frontal cortex in relation to their body size of all
mammals (including humans).

Think

• Platypus

http://www.thehindu.com/todays-paper/tp-in-school/whats-threatening-echidnas/
article22830002.ece

QUESTION 50.
Which of the following statements are correct regarding Milan Exercise?

1. It is a congregation of littoral navies conducted by Indian Navy.

2. It is being organised at the Andaman and Nicobar Islands

3. It is held once in every five years.

Select the code from following:

a) 1 and 2
b) 2 and 3
c) 1 and 3
d) All of the above
Correct Answer: A
Your Answer: Unanswered
Explanation

Solution (a)

Milan 2018

• It is a congregation of littoral navies conducted biennially by Indian Navy at the Andaman


and Nicobar Islands
• It is being organised at the Andaman and Nicobar Islands
• The exercise is being hosted by the Indian Navy under the aegis of the Andaman and
Nicobar Command.
• It was first held in 1995 and saw participation of four littoral navies
• Theme - 'Friendship Across the Seas'
• The theme of this year’s MILAN International Maritime Seminar is ‘In Pursuit of Maritime
Good Order - Need for Comprehensive Information Sharing Apparatus’

IASbaba
Web: http://ilp.iasbaba.com/ Score:
Email: ilp@iasbaba.com 4.00 / 200
Page 47
Exam Title : Test 21-Revision: Polity & ...
Email
Contact :

• Participating countries - Australia, Malaysia, Maldives, Mauritius, Myanmar, New Zealand,


Oman, Vietnam, Thailand, Tanzania, Sri Lanka, Singapore, Bangladesh, Indonesia, Kenya
and Cambodia.

Think

• IOR-ARC

http://www.thehindu.com/news/national/navy-to-host-milan-from-march-6-to-13/
article22852221.ece

QUESTION 51.
The President is not elected directly, but by members of Electoral College. The College
comprises of

1. Elected members of both the Houses of Parliament.

2. Elected members of state legislature.

3. Elected members of legislative assemblies of union territories.

Select the correct answer using code below

a) 1 and 2
b) 2 and 3
c) 1 and 3
d) 1,2 and 3
Correct Answer: C
Your Answer: Unanswered
Explanation

Solution (c)

The President is elected not directly by the people but by members of Electoral College
consisting of:

1. The elected members of both the Houses of Parliament

2. The elected members of the legislative assemblies of the states

3. The elected members of the legislative assemblies of the Union Territories of Delhi and
Puducherry

Thus, the nominated members of both of Houses of Parliament, the nominated members of the
state legislative assemblies, the members (both elected and nominated) of the state legislative
councils (in case of the bicameral legislature) and the nominated members of the Legislative
Assemblies of Delhi and Puducherry do not participate in the election of the President.

Note: State legislature consists of Governor, legislative assembly and legislative council.

THINK!

· Manner of election of President

IASbaba
Web: http://ilp.iasbaba.com/ Score:
Email: ilp@iasbaba.com 4.00 / 200
Page 48
Exam Title : Test 21-Revision: Polity & ...
Email
Contact :

· Qualifications for election as President

QUESTION 52.
Which of the following changes to the constitution are outside the scope of Article 368?

1. Establishment of new state

2. Representation of states in Parliament

3. Changes in elections to Parliament

4. Changes in scheduled areas under Fifth Schedule

Select the correct answer using code below

a) 1,3 and 4
b) 1,2 and 3
c) 2 and 4
d) 1,2,3 and 4
Correct Answer: A
Your Answer: Unanswered
Explanation

Solution (a)

A number of provisions in the Constitution can be amended by a simple majority of the two
Houses of Parliament outside the scope of Article 368. These provisions include:

· Admission or establishment of new states.

· Elections to Parliament and state legislatures.

· Fifth Schedule—administration of scheduled areas and scheduled tribes.

Amendment in representation of states in Parliament is done by a special majority of the


Parliament and also with the consent of half of the state legislatures by a simple majority.

Do you know?

· Unlike ordinary bill, for constitutional bill there is no provision for holding a joint sitting of the
two Houses in case of deadlock.

THINK!

· Types of majority

· Indian constitution is neither rigid nor flexible, but a synthesis of both

· Basic structure doctrine

IASbaba
Web: http://ilp.iasbaba.com/ Score:
Email: ilp@iasbaba.com 4.00 / 200
Page 49
Exam Title : Test 21-Revision: Polity & ...
Email
Contact :

QUESTION 53.
As per Indian Constitution, Environment protection is a part of

1. Fundamental Rights

2. Directive Principles of state Policy

3. Fundamental Duties

Select the correct answer using code below

a) 1 and 2
b) 2 and 3
c) 1 and 3
d) 1,2 and 3
Correct Answer: D
Your Answer: Unanswered
Explanation

Solution (d)

According to Article 21 of the constitution, “no person shall be deprived of his life or personal
liberty except according to procedure established by law”.

As per Maneka Gandhi vs. Union of India, Supreme Court has declared right to healthy
environment as a part of Article 21.

Under Directive Principles of State Policy, Article 48 -A of the constitution says, “the state shall
endeavor to protect and improve the environment and to safeguard the forests and wild life of
the country”.

Under Fundamental Duties, Article 51-A (g), says, “It shall be duty of every citizen of India to
protect and improve the natural environment including forests, lakes, rivers and wild life and to
have compassion for living creatures.”

For more details:

http://pib.nic.in/newsite/PrintRelease.aspx?relid=105411

THINK!

· Maneka Gandhi case and its implications on the constitutional interpretation

· Environment Protection laws in India

QUESTION 54.
Consider the following statements with reference to Inter-State Council

1. It is a constitutional body.

2. In case of any legal controversy between the governments, the decision of council is final and
binding.

IASbaba
Web: http://ilp.iasbaba.com/ Score:
Email: ilp@iasbaba.com 4.00 / 200
Page 50
Exam Title : Test 21-Revision: Polity & ...
Email
Contact :

3. Constitution allows for creation of only pan-India councils and not regional councils.

Which of the statements given above is/are correct?

a) Only 1
b) 1 and 2
c) Only 3
d) 1,2 and 3
Correct Answer: A
Your Answer: Unanswered
Explanation

Solution (a)

Article 263 contemplates the establishment of an Inter-State Council to effect coordination


between the states and between Centre and states.

The Council can deal with any controversy whether legal or non-legal, but its function is
advisory unlike that of the court which gives a binding decision.

Four Regional Councils for Sales Tax for the Northern, Eastern, Western and Southern Zones
have already been established under this article by the President.

Do you know?

· The permanent Inter-State Council under Article 263 was created on the recommendations of
Sarkaria Commission and was established in 1990.

THINK!

· Role of inter-state council in co-operative federalism

QUESTION 55.
The Constitution describes India as a ‘Union of States’ because

1. The federation is a result of agreement among the states

2. The states have no right to secede from the federation

Select the correct answer using code below

a) Only 1
b) Only 2
c) Both 1 and 2
d) Neither 1 nor 2
Correct Answer: B
Your Answer: Unanswered
Explanation

Solution (b)

Article 1 of the Constitution describes India as a ‘Union of States’.

IASbaba
Web: http://ilp.iasbaba.com/ Score:
Email: ilp@iasbaba.com 4.00 / 200
Page 51
Exam Title : Test 21-Revision: Polity & ...
Email
Contact :

According to Dr B R Ambedkar, the phrase ‘Union of States’ has been preferred to ‘Federation
of States’ to indicate two things:

(i) The Indian federation is not the result of an agreement among the states like the American
federation

(ii) The states have no right to secede from the federation.

Do you know?

· Indian federation is sometimes called Quasi-federal due to increased unitary bias.

THINK!

· Unitary and federal features of Indian Constitution

· S R Bommai vs Union of India

QUESTION 56.
As per Supreme Court, which of the following rights are part of Right to Freedom of Speech and
expression?

1. Freedom of press

2. Right to know about government activities

3. Freedom of silence

4. Right to strike

Select the correct answer using code below

a) 1 and 3
b) 1,2 and 3
c) 2,3 and 4
d) 1,2 and 4
Correct Answer: B
Your Answer: Unanswered
Explanation

Solution (b)

The Supreme Court held that the freedom of speech and expression includes the following:

(a) Right to propagate one’s views as well as views of others.

(b) Freedom of the press.

(c) Freedom of commercial advertisements.

(d) Right against tapping of telephonic conversation.

(e) Right to telecast, that is, government has no monopoly on electronic media.

IASbaba
Web: http://ilp.iasbaba.com/ Score:
Email: ilp@iasbaba.com 4.00 / 200
Page 52
Exam Title : Test 21-Revision: Polity & ...
Email
Contact :

(f) Right against bundh called by a political party or organisation.

(g) Right to know about government activities.

(h) Freedom of silence.

(i) Right against imposition of pre-censorship on a newspaper.

(j) Right to demonstration or picketing but not right to strike.

Do you know?

· Though Indian constitution provides for a guaranteed fundamental right to form association or
Labour unions but there is no fundamental right to go on strike. In fact, it is a conditional right
covered under the Industrial Dispute Act, 1947, and only available after certain pre-condition
are fulfilled.

THINK!

· Cyber defamation vs Freedom of Speech and expression

· Sedition laws vs Freedom of Speech and Expression

QUESTION 57.
Consider the following statements with reference to the Fundamental Duties in Indian
Constitution

1. These are inspired by the Japanese Constitution.

2. These are applicable only on citizens.

3. Unlike Directive Principles of State Policy, these are non-justiciable.

Which of the statements given above is/are correct?

a) 1 and 2
b) Only 2
c) 2 and 3
d) 1,2 and 3
Correct Answer: B
Your Answer: Unanswered
Explanation

Solution (b)

The Fundamental Duties in the Indian Constitution are inspired by the Constitution of erstwhile
USSR.

The Fundamental Duties are confined to citizens only and do not extend to foreigners.

Like the Directive Principles, the fundamental duties are also nonjusticiable.

THINK!

IASbaba
Web: http://ilp.iasbaba.com/ Score:
Email: ilp@iasbaba.com 4.00 / 200
Page 53
Exam Title : Test 21-Revision: Polity & ...
Email
Contact :

· Recommendation of Swaran Singh Committee

· Criticism of Fundamental duties

QUESTION 58.
Which of the following is/are the consequences of declaration of National Emergency?

1. Centre can give directions to state on ‘any’ matter.

2. The legislative power of state is suspended.

3. The President cannot issue an ordinance while emergency is in operation.

Select the correct answer using code below

a) Only 1
b) 1 and 2
c) 2 and 3
d) Only 3
Correct Answer: A
Your Answer: Unanswered
Explanation

Solution (a)

During a national emergency, the Centre becomes entitled to give executive directions to a
state on ‘any’ matter. Thus, the state governments are brought under the complete control of
the Centre, though they are not suspended.

During a national emergency, the Parliament becomes empowered to make laws on any subject
mentioned in the State List. Though the state Legislatures are not suspended, but the
Constitution becomes unitary rather than federal.

While a proclamation of national emergency is in operation, the President can issue ordinances
on the state subjects also, if the Parliament is not in session.

THINK!

· Effects of national emergency on fundamental rights

· Difference between president’s rule and national emergency

QUESTION 59.
According to Constitution, a person shall be disqualified for being elected as a Member of
Parliament if

1. He holds an office of profit.

2. He is not a citizen of India.

IASbaba
Web: http://ilp.iasbaba.com/ Score:
Email: ilp@iasbaba.com 4.00 / 200
Page 54
Exam Title : Test 21-Revision: Polity & ...
Email
Contact :

3. He is found guilty of election offences.

4. He has been punished for practicing untouchability.

Select the correct answer using code below

a) 1 and 2
b) 1 and 3
c) 2 and 4
d) 3 and 4
Correct Answer: A
Your Answer: Unanswered
Explanation

Solution (a)

Under the Constitution, a person shall be disqualified for being elected as a Member of
Parliament, if:

1. He holds any office of profit under the Union or state government (except that of a minister
or any other office exempted by Parliament).

2. He is of unsound mind and stands so declared by a court.

3. He is an undischarged insolvent.

4. He is not a citizen of India or has voluntarily acquired the citizenship of a foreign state or is
under any acknowledgement of allegiance to a foreign state.

5. He is so disqualified under any law made by Parliament.

Criteria of being found guilty of election offences and for practising untouchability, are
disqualifications under the Representation of People Act (1951).

Do you know?

· Holding an office of profit is an important criterion of disqualification of President, Vice-


President or any member of Parliament, but it has not been defined in the Constitution or in the
Representation of the People Act of 1951.

THINK!

· Disqualification on the basis of defection

· Representation of the People Act of 1951

QUESTION 60.
Supreme Court does not have original jurisdiction in which of the following cases?

1. Dispute between two states

2. Dispute arising out of a pre-constitutional treaty

IASbaba
Web: http://ilp.iasbaba.com/ Score:
Email: ilp@iasbaba.com 4.00 / 200
Page 55
Exam Title : Test 21-Revision: Polity & ...
Email
Contact :

3. Inter-state water dispute

Select the correct answer using code below

a) Only 2
b) 2 and 3
c) Only 1
d) Only 3
Correct Answer: B
Your Answer: Unanswered
Explanation

Solution (b)

The Supreme Court has exclusive original jurisdiction on any dispute between:

(a) The Centre and one or more states

(b) The Centre and any state or states on one side and one or more states on the other

(c) Between two or more states.

Supreme Court does not have a jurisdiction on disputes arising out of pre-constitutional treaty.

Do you know?

· Inter State Water Disputes Act, 1956 has excluded Supreme Court from jurisdiction over inter-
state water disputes. However, by its own declaration, Supreme Court has given itself appellate
jurisdiction over such matters.

THINK!

· Appellate jurisdiction

· Special Leave Petition

QUESTION 61.
Chairman of State Public Service Commission is appointed by

a) President
b) Prime Minister
c) Governor
d) Cabinet Secretary
Correct Answer: C
Your Answer: Unanswered
Explanation

Solution (c)

Governor appoints the chairman and members of the state public service commission.

However, they can be removed only by the president and not by a governor.

IASbaba
Web: http://ilp.iasbaba.com/ Score:
Email: ilp@iasbaba.com 4.00 / 200
Page 56
Exam Title : Test 21-Revision: Polity & ...
Email
Contact :

THINK!

· Executive powers of Governor

QUESTION 62.
Consider the following statements with reference to judicial review in India

1. It can be done only for legislative actions and not for executive actions.

2. The phrase ‘judicial review’ has been mentioned in Article 13.

3. Ninth schedule is outside the purview of judicial review.

Which of the statements given above is correct?

a) Only 1
b) Only 2
c) Only 3
d) None of the above
Correct Answer: D
Your Answer: Unanswered
Explanation

Solution (d)

Justice Syed Shah Mohamed Quadri has classified the judicial review into the following three
categories:

1. Judicial review of constitutional amendments.

2. Judicial review of legislation of the Parliament and State Legislatures and subordinate
legislations.

3. Judicial review of administrative action of the Union and State and authorities under the
state.

Thus, judicial review is applicable to both legislative and executive actions.

The phrase ‘Judicial Review’ has nowhere been used in the Constitution.

Article 31B saves the acts and regulations included in the Ninth Schedule from being
challenged and invalidated on the ground of contravention of any of the Fundamental Rights.

However, in a significant judgment delivered in I.R. Coelho case (2007), the Supreme Court
ruled that the laws placed under the Ninth Schedule after April 24, 1973, are open to challenge
in court if they violated Fundamental Rights guaranteed under the Articles 14, 15, 19 and 21 or
the ‘basic structure’ of the Constitution.

Do you know?

IASbaba
Web: http://ilp.iasbaba.com/ Score:
Email: ilp@iasbaba.com 4.00 / 200
Page 57
Exam Title : Test 21-Revision: Polity & ...
Email
Contact :

· The Supreme Court has declared the power of judicial review as an element of the basic
structure of the Constitution. Hence, the power of judicial review cannot be curtailed or
excluded even by a constitutional amendment.

THINK!

· Difference between power of judicial review in India and U.S.A.

· Basic Structure Doctrine

QUESTION 63.

Consider the following statements with reference to 73 rd Constitutional Amendment Act

1. The act provides for a mandatory three-tier system of panchayati raj in every state.

2. The chairperson of panchayats at all levels shall be elected indirectly.

3. The act provides for the reservation of one-third of the total number of seats for women.

Which of the statements given above is/are correct?

a) 1 and 3
b) Only 3
c) 2 and 3
d) 1,2 and 3
Correct Answer: B
Your Answer: Unanswered
Explanation

Solution (b)

The act provides for a three-tier system of panchayati raj in every state, that is, panchayats at
the village, intermediate, and district levels. However, a state having a population not
exceeding 20 lakh may not constitute panchayats at the intermediate level.

All the members of panchayats at the village, intermediate and district levels shall be elected
directly by the people. Further, the chairperson of panchayats at the intermediate and district
levels shall be elected indirectly—by and from amongst the elected members thereof. However,
the chairperson of a panchayat at the village level shall be elected in such manner as the state
legislature determines.

The act provides for the reservation of not less than one-third of the total number of seats for
women (including the number of seats reserved for women belonging the SCs and STs).

THINK!

· Issues with Panchayati Raj Institutions

QUESTION 64.

IASbaba
Web: http://ilp.iasbaba.com/ Score:
Email: ilp@iasbaba.com 4.00 / 200
Page 58
Exam Title : Test 21-Revision: Polity & ...
Email
Contact :

Which of the following are constitutionally established independent bodies?

1. Election Commission

2. Office of Comptroller and Auditor General of India

3. Joint Public Service Commission

4. National Human Rights Commission

Select the correct answer using code below

a) 1 and 2
b) 1,2 and 3
c) 3 and 4
d) 1,2,3 and 4
Correct Answer: A
Your Answer: Unanswered
Explanation

Solution (a)

Election Commission and the Office of Comptroller and Auditor General of India are
constitutionally established independent bodies.

Joint Public Service Commission and National Human Rights Commission are statutory bodies.

THINK!

· Constitutional bodies

· National Human Rights Commission

QUESTION 65.
Comptroller and Auditor General of India (CAG) do not audit which of the following?

1. Local bodies

2. Oil and Natural Gas Commission

3. State governments

4. All institutions established by act of Central government

Select the correct answer using code below

a) Only 1
b) 1 and 2
c) Only 4
d) 3 and 4
Correct Answer: C
Your Answer: Unanswered

IASbaba
Web: http://ilp.iasbaba.com/ Score:
Email: ilp@iasbaba.com 4.00 / 200
Page 59
Exam Title : Test 21-Revision: Polity & ...
Email
Contact :

Explanation

Solution (c)

CAG audits Local bodies, State governments and Central government.

However, it has limited role in case of public corporations.

(i) Some corporations are audited totally and directly by the CAG, for example, Damodar Valley
Corporation, Oil and Natural Gas Commission, Air India, Indian Airlines Corporation, and
others.

(ii) Some other corporations are audited by private professional auditors who are appointed by
the Central Government in consultation with the CAG. The examples are, Central Warehousing
Corporation, Industrial Finance Corporation, and others.

(iii) Some other corporations are totally subjected to private audit. In other words, their audit is
done exclusively by private professional auditors and the CAG does not come into the picture at
all. Examples of such corporations are Life Insurance Corporation of India, Reserve Bank of
India, State Bank of India, Food Corporation of India, and others.

Do you know?

· The secret service expenditure is a limitation on the auditing role of the CAG. In this regard,
the CAG cannot call for particulars of expenditure incurred by the executive agencies.

THINK!

· Criticism of role of CAG

QUESTION 66.

Which of the following were the provisions of the 42 nd amendment act?

1. Addition of word ‘Socialist’ in the Preamble.

2. Shifting of few subjects from state list to concurrent list.

3. Making right to property a legal right.

Select the correct answer using code below

a) 1 and 2
b) Only 1
c) 2 and 3
d) 1,2 and 3
Correct Answer: A
Your Answer: Unanswered
Explanation

Solution (a)

42 nd Amendment Act, 1976 did the following:

IASbaba
Web: http://ilp.iasbaba.com/ Score:
Email: ilp@iasbaba.com 4.00 / 200
Page 60
Exam Title : Test 21-Revision: Polity & ...
Email
Contact :

1. Addition of words ‘Socialist, secular and integrity’ in the Preamble.

2. Addition of fundamental duties under Article 51-A.

3. Shifting of few subjects from state list to concurrent list.

4. Addition of new DPSPs.

Right to property was made a legal right under the 44 th amendment act, 1978.

THINK!

· Important provisions of 44 th amendment act

QUESTION 67.
Receipts from which of the following does not form a major source of non-tax revenue of the
Centre?

a) Banking
b) Forests
c) Railway
d) Coinage and currency
Correct Answer: B
Your Answer: Unanswered
Explanation

Solution (b)

The receipts from the following form the major sources of non-tax revenues of the Centre:

(i) Posts and telegraphs

(ii) Railways

(iii) Banking

(iv) Broadcasting

(v) Coinage and currency

(vi) Central public sector enterprises

(vii) Escheat and lapse.

Receipts from forests form non-tax revenue for states.

THINK!

· Distribution of tax and non-tax revenue between Centre and states

IASbaba
Web: http://ilp.iasbaba.com/ Score:
Email: ilp@iasbaba.com 4.00 / 200
Page 61
Exam Title : Test 21-Revision: Polity & ...
Email
Contact :

QUESTION 68.
Consider the following statements with reference to the speaker of Lok Sabha

1. He holds a casting vote in case of a tie.

2. He cannot vote in the house while a resolution for his removal is under consideration in the
house.

3. He remains in his office even after the dissolution of Lok Sabha.

Which of the statements given above is/are correct?

a) Only 1
b) 1 and 3
c) 2 and 3
d) 1,2 and 3
Correct Answer: B
Your Answer: Unanswered
Explanation

Solution (b)

Normally, speaker does not vote in the first instance. But he can exercise a casting vote in the
case of a tie.

When a resolution for the removal of the Speaker is under consideration of the House, he
cannot preside at the sitting of the House, though he may be present. However, he can speak
and take part in the proceedings of the House at such a time and vote in the first instance,
though not in the case of an equality of votes.

Whenever the Lok Sabha is dissolved, the Speaker does not vacate his office and continues till
the newly-elected Lok Sabha meets.

THINK!

· Powers and position of speaker

· Controversies on role of speaker

QUESTION 69.
Consider the following statements with reference to the Money bills

1. A money bill can be introduced only by a minister.

2. The Rajya Sabha cannot reject or amend the money bill.

3. The President cannot withhold his assent to the bill.

Which of the statements given above are correct?

a) 1 and 2
b) 1 and 3

IASbaba
Web: http://ilp.iasbaba.com/ Score:
Email: ilp@iasbaba.com 4.00 / 200
Page 62
Exam Title : Test 21-Revision: Polity & ...
Email
Contact :

c) 2 and 3
d) 1,2 and 3
Correct Answer: A
Your Answer: Unanswered
Explanation

Solution (a)

Article 110 of the Constitution deals with the definition of money bills.

Every such bill is considered to be a government bill and can be introduced only by a minister.

The Rajya Sabha has restricted powers with regard to a money bill. It cannot reject or amend a
money bill.

When a money bill is presented to the president, he may either give his assent to the bill or
withhold his assent to the bill but cannot return the bill for reconsideration of the Houses.

THINK!

· Money bill vs Financial bill

QUESTION 70.
Consider the following statements with reference to the Estimates Committee

1. The Rajya Sabha has no representation in this committee.

2. Its members are selected by the speaker.

3. It examines the budget estimates after voting by the Parliament.

Which of the statements given above are correct?

a) 1 and 2
b) 1 and 3
c) 2 and 3
d) 1,2 and 3
Correct Answer: B
Your Answer: Unanswered
Explanation

Solution (b)

Estimate Committee examines the estimates included in the budget and suggest ‘economies’ in
public expenditure.

It has 30 members and all are from Lok Sabha only. The Rajya Sabha has no representation in
this committee.

These members are elected by the Lok Sabha every year from amongst its own members,
according to the principles of proportional representation by means of a single transferable
vote.

IASbaba
Web: http://ilp.iasbaba.com/ Score:
Email: ilp@iasbaba.com 4.00 / 200
Page 63
Exam Title : Test 21-Revision: Polity & ...
Email
Contact :

It examines the budget estimates only alter they have been voted by the Parliament, and not
before that.

Do you know?

· The role of the committee is limited, neither it can question the policy laid down by the
Parliament nor its recommendations are binding on the ministries.

THINK!

· Standing committees and ad hoc committees

QUESTION 71.
Consider the following statements with reference to High Court

1. The judges of a high court are appointed by the Governor after consultation with the chief
justice of the concerned High Court.

2. The President determines the strength of a high court from time to time depending upon its
workload.

Which of the statements given above is/are correct?

a) Only 1
b) Only 2
c) Both 1 and 2
d) Neither 1 nor 2
Correct Answer: B
Your Answer: Unanswered
Explanation

Solution (b)

The judges of a high court are appointed by the President after consultation with the chief
justice of the concerned high court.

The Constitution does not specify the strength of a high court and leaves it to the discretion of
the president. Accordingly, the

President determines the strength of a high court from time to time depending upon its
workload.

Do you know?

· Delhi is the only union territory that has a high court of its own.

THINK!

· National Judicial Appointments Commission (NJAC)

· Procedure of removal of a high court judge

IASbaba
Web: http://ilp.iasbaba.com/ Score:
Email: ilp@iasbaba.com 4.00 / 200
Page 64
Exam Title : Test 21-Revision: Polity & ...
Email
Contact :

QUESTION 72.
Special provisions under Part XXI are not provided for which of the following states?

a) Nagaland
b) Goa
c) Sikkim
d) Punjab
Correct Answer: D
Your Answer: Unanswered
Explanation

Solution (d)

Articles 371 to 371-J in Part XXI of the constitution contain special provisions for twelve states
viz., Maharashtra, Gujarat, Nagaland, Assam, Manipur, Andhra Pradesh, Telangana, Sikkim,
Mizoram, Arunachal Pradesh, Goa and Karnataka.

Do you know?

· Originally, the constitution did not make any special provisions for these states. They have
been incorporated by the various subsequent amendments.

THINK!

· Special status of Jammu & Kashmir

QUESTION 73.
Right to vote in a presidential election is a

a) Natural right
b) Constitutional right
c) Fundamental right
d) Legal right
Correct Answer: D
Your Answer: Unanswered
Explanation

Solution (d)

Constitutional rights are those which are explicitly mentioned in the constitution.

Article 54 mentions about the presidential elections and the members of Electoral College but
not right of vote to them. This is taken care under the Representation of the People Act. So it is
a legal right.

Also recently Maharashtra government protested about jailed legislator voting and stated it’s
not a constitutional right.

Further reading:

IASbaba
Web: http://ilp.iasbaba.com/ Score:
Email: ilp@iasbaba.com 4.00 / 200
Page 65
Exam Title : Test 21-Revision: Polity & ...
Email
Contact :

http://pib.nic.in/newsite/PrintRelease.aspx?relid=167193

THINK!

· Representation of the People Act

QUESTION 74.
Consider the following statement in reference to the making of Indian constitution

1. Indian National Congress officially demanded the constituent assembly for the first time in
1935.

2. The demand for constituent assembly was accepted in Cripps mission plan.

3. Jawaharlal Nehru was made the head of the interim government and the constituent
assembly.

Which of the statements given above are incorrect ?

a) 1 and 2
b) 2 and 3
c) 1 and 3
d) 1,2 and 3
Correct Answer: B
Your Answer: Unanswered
Explanation

Solution (b)

In 1934, M.N Roy first put forth the idea of constituent assembly and in 1935, the Indian
National Congress officially demanded it.

The demand was finally accepted in principle by British government in 1940 in August offer.

The members of the interim government were members of the Viceroy’s Executive Council. The
Viceroy continued to be the head of the Council. But, Jawaharlal Nehru was designated as the
Vice-President of the Council.

THINK!

· Composition of Constituent Assembly

· Major committees of Constituent Assembly

QUESTION 75.
Which of the following constitutional provisions with regard to the enactment of budget is incor
rect ?

a) Parliament cannot increase a tax.

IASbaba
Web: http://ilp.iasbaba.com/ Score:
Email: ilp@iasbaba.com 4.00 / 200
Page 66
Exam Title : Test 21-Revision: Polity & ...
Email
Contact :

b) Rajya Sabha cannot vote on demand for grants.


c) Unlike a money bill, a finance bill dealing with taxation can be introduced in Rajya Sabha.
d) No tax shall be levied except by authority of law.
Correct Answer: C
Your Answer: Unanswered
Explanation

Solution (c)

Parliament can reduce or abolish a tax, but cannot increase it.

Rajya Sabha cannot vote on demand for grants.

A money bill or a finance bill dealing with taxation cannot be introduced in the Rajya Sabha- it
must be introduced only in the Lok Sabha.

No tax shall be levied except by authority of law.

THINK!

· Stages in enactment of budget

· Charged expenditure

· Different types of funds of the Central Government

QUESTION 76.
Consider the following statements with reference to Finance Commission

1. It is a quasi-judicial body.

2. The chairman of the commission is not eligible for reappointment.

3. The qualifications of the members of commission are not specified in the Constitution.

Which of the statements given above are correct?

a) 1 and 2
b) 1 and 3
c) 2 and 3
d) 1,2 and 3
Correct Answer: B
Your Answer: Unanswered
Explanation

Solution (b)

Article 280 of the Constitution of India provides for a Finance Commission as a quasi-judicial
body. It is constituted by the president of India every fifth year or at such earlier time, as he
considers necessary.

IASbaba
Web: http://ilp.iasbaba.com/ Score:
Email: ilp@iasbaba.com 4.00 / 200
Page 67
Exam Title : Test 21-Revision: Polity & ...
Email
Contact :

The Finance Commission consists of a chairman and four other members to be appointed by the
president. They hold office for such period as specified by the president in his order. They are
eligible for reappointment.

The Constitution authorises the Parliament to determine the qualifications of members of the
commission and the manner in which they should be selected.

Do you know?

· The constitution of India envisages the Finance commission as the balancing wheel of fiscal
federalism in India.

THINK!

· Functions of finance commission

· Why co-operative federalism cannot be achieved without fiscal federalism?

QUESTION 77.
Original jurisdiction of Central Administrative Tribunal (CAT) does not extend to which of the
following?

a) All-India services
b) Civilian employees of defence services
c) Secretarial staff of Parliament
d) Civil posts under the Centre
Correct Answer: C
Your Answer: Unanswered
Explanation

Solution (c)

Central Administrative Tribunal (CAT) exercises original jurisdiction in relation to recruitment


and all service matters of public servants covered by it. Its jurisdiction extends to the all-India
services, the Central civil services, civil posts under the Centre and civilian employees of
defence services. However, the members of the defence forces, officers and servants of the
Supreme Court and the secretarial staff of the Parliament are not covered by it.

Do you know?

· With the amendment in Administrative Tribunals Act, 1985 in 2006, the members have been
given the status of judges of High Courts.

THINK!

· Article 323 A and Article 323 B

· Tribunalization of justice

IASbaba
Web: http://ilp.iasbaba.com/ Score:
Email: ilp@iasbaba.com 4.00 / 200
Page 68
Exam Title : Test 21-Revision: Polity & ...
Email
Contact :

QUESTION 78.
Consider the following statements with reference to office of Attorney General for India

1. He is the highest law officer in the country.

2. He must be a person who is qualified to be appointed a judge of the Supreme Court.

3. He can be removed in the same manner as a judge of Supreme Court.

Which of the statements given above are correct?

a) 1 and 2
b) 1 and 3
c) 2 and 3
d) 1,2 and 3
Correct Answer: A
Your Answer: Unanswered
Explanation

Solution (a)

The Constitution (Article 76) has provided for the office of the Attorney General for India. He is
the highest law officer in the country.

The Attorney General (AG) is appointed by the president. He must be a person who is qualified
to be appointed a judge of the Supreme Court.

The Constitution does not contain the procedure and grounds for his removal. He holds office
during the pleasure of the president.

Do you know?

· The Attorney General is not a full-time counsel for the Government and is allowed private legal
practice.

THINK!

· Solicitor General of India

· Advocate General of the state

QUESTION 79.
Which of the following constitutional amendments made elementary education a fundamental
right?

a) 84 th
b) 91 st
c) 93 rd
d) 86 th
Correct Answer: D

IASbaba
Web: http://ilp.iasbaba.com/ Score:
Email: ilp@iasbaba.com 4.00 / 200
Page 69
Exam Title : Test 21-Revision: Polity & ...
Email
Contact :

Your Answer: Unanswered


Explanation

Solution (d)

86 th constitutional amendment of 2002 made elementary education a fundamental right.

It added Article 21A, clause (k) under Article 51A and changed the text of Article 45.

THINK!

· Analysis of Right to Education Act

QUESTION 80.
Who among the following takes an oath to ‘uphold the Constitution’?

a) President
b) Prime Minister
c) Chief Justice of India
d) Member of Parliament
Correct Answer: C
Your Answer: Unanswered
Explanation

Solution (c)

Chief Justice of India and Judges of Supreme Court and high courts take the oath to uphold the
Constitution.

Do you know?

· Other than these judges, CAG and Information Commissioners also take similar oath.

THINK!

· Oath of President

QUESTION 81.
Yucatan peninsula was in news for its two large networks of underwater caves. In which of the
following countries is it located?

a) New Zealand
b) Mexico
c) China
d) Argentina
Correct Answer: B
Your Answer: Unanswered
Explanation

IASbaba
Web: http://ilp.iasbaba.com/ Score:
Email: ilp@iasbaba.com 4.00 / 200
Page 70
Exam Title : Test 21-Revision: Polity & ...
Email
Contact :

Solution (b)

The two giant underwater caverns under Mexico's Yucatan Peninsula make the largest known
flooded cave system on the planet (347 kilometer).

Fossils of giant sloths and an elaborate shrine of the Mayan god of commerce have been found
there.

The Yucatan peninsula is studded with monumental relics of the Maya people, whose cities
drew upon an extensive network of sinkholes linked to subterranean waters known as cenotes.

Do you know?

· An Ancient pyramid city, known as Angamuco, was discovered underground in heartland of


Mexico by using LIDAR Technology.

THINK!

· Maya civilization

· Countries of Central America

QUESTION 82.
Consider the following statements

1. He wrote Majma-ul-Bahrain.

2. He translated Sanskrit Upanishads to Persian.

Which of the following best describe the above given statements?

a) Shivaji
b) Dara Shikoh
c) Abul Fazal
d) Akbar
Correct Answer: B
Your Answer: Unanswered
Explanation

Solution (b)

IASbaba
Web: http://ilp.iasbaba.com/ Score:
Email: ilp@iasbaba.com 4.00 / 200
Page 71
Exam Title : Test 21-Revision: Polity & ...
Email
Contact :

Dara Shikoh commissioned the translation of all the Upanishads from Sanskrit to Persian for
Muslim scholar to learn.

He wrote the Majma-ul-Bahrain (The confluence of two seas) to elaborate upon the syncretism
between Sufism and Vedic philosophies.

THINK!

· Facts about Dara Shikoh

· Auragzeb’s policies

QUESTION 83.
Aeons, eras, periods, epochs, and ages denote different segments on the geological timescale.
Which global body is responsible for setting these time standards to express Earth’s history?

a) The International Society of Ankylography


b) International Commission on Stratigraphy
c) The International Union of Balneography
d) The International Commission on Earth and Sciences
Correct Answer: B
Your Answer: Unanswered
Explanation

Solution (b)

The International Commission on Stratigraphy is the largest and oldest constituent scientific
body in the International Union of Geological Sciences (IUGS).

Its primary objective is to precisely define global units (systems, series, and stages) of the
International Chronostratigraphic Chart that, in turn, are the basis for the units (periods,
epochs, and age) of the International Geologic Time Scale; thus setting global standards for the
fundamental scale for expressing the history of the Earth.

QUESTION 84.
Consider the following statements with reference to The Khadi and Village Industries
Commission (KVIC)

1. It is a statutory body under the Ministry of Micro, Small & Medium Enterprises.

2. It has an objective of providing employment to rural youth.

3. It is provides financial assistance to village industries.

Which of the statements given above are correct?

a) 1 and 2
b) 1 and 3

IASbaba
Web: http://ilp.iasbaba.com/ Score:
Email: ilp@iasbaba.com 4.00 / 200
Page 72
Exam Title : Test 21-Revision: Polity & ...
Email
Contact :

c) 2 and 3
d) 1,2 and 3
Correct Answer: D
Your Answer: Unanswered
Explanation

Solution (d)

The Khadi and Village Industries Commission (KVIC) is a statutory body established by an Act of
Parliament in 1956.

The broad objectives that the KVIC are

· The social objective of providing employment.

· The economic objective of producing saleable articles.

· The wider objective of creating self-reliance amongst the poor and building up of a strong
rural community spirit.

KVIC is entrusted with the task of providing financial assistance to institutions and individuals
for development and operation of Khadi and village industries.

KVIC may take steps as to ensure genuineness of the products and to set standards of quality
and ensure that the products of Khadi and village industries do conform to the standards.

Do you know?

· As per KVIC Act, any industry located in a rural area which produces any goods or renders any
service with or without the use of power and in which the fixed capital investment per head of
an artisan or a worker does not exceed one lakh rupees, is a village industry.

THINK!

· Khaadi Haat

· Sweet Revolution

http://www.business-standard.com/article/pti-stories/kvic-divisional-office-comes-up-in-hubli-91-
yr-old-khadi-centre-revived-in-mysuru-118022701086_1.html

QUESTION 85.
Recently Finance Ministry has announced an initiative called “Operation Green” which of the
below statement best describes it?

a) Initiative to turn all government offices into paperless.


b) Initiative to increase fruits and vegetable production in country.
c) Initiative to turn all major airports of country carbon neutral.
d) Initiative to set up solar panels on Major River routes to conserve water from evaporation
and also generate electricity.
Correct Answer: B
Your Answer: Unanswered

IASbaba
Web: http://ilp.iasbaba.com/ Score:
Email: ilp@iasbaba.com 4.00 / 200
Page 73
Exam Title : Test 21-Revision: Polity & ...
Email
Contact :

Explanation

Solution (b)

Operation Green has been announced on the lines of Operation Flood for enhancing the
production of tomato, onion and potatoes.

THINK!

· Measures taken to address agrarian distress

https://www.thehindubusinessline.com/economy/budget/rs-500-crore-allocated-for-operation-
green/article22619481.ece

QUESTION 86.
Consider the following statements with reference to Global Seed Vault

1. It is located in Greenland.

2. It has no ownership rights and is only for humanity’s food security.

3. India has also sent seeds to the vault.

Which of the statements given above is/are correct?

a) 1 and 2
b) Only 3
c) 1 and 3
d) 2 and 3
Correct Answer: B
Your Answer: Unanswered
Explanation

Solution (b)

The Global Seed Vault is located deep inside a mountain on Svalbard, a remote Arctic island in a
Norwegian archipelago. It is called Svalbard Global Seed Vault (SGSV).

It is a vault that contains humanity’s ultimate food security.

It is built to withstand manmade and natural disasters. It is theoretically protected against


conventional bombs, and sits out of reach of rising sea levels.

Norway’s government owns the vault and depositors retain ownership rights. India has sent
around 9.5 crore seeds to Svalbard.

The vault is facing threat from climate change, due to melting of permafrost.

Do you know?

· India has its own seed vault at Chang La in the Himalayas, at a height of 17,300 feet
maintained jointly by National Bureau of Plant Genetic Resources (which comes under the

IASbaba
Web: http://ilp.iasbaba.com/ Score:
Email: ilp@iasbaba.com 4.00 / 200
Page 74
Exam Title : Test 21-Revision: Polity & ...
Email
Contact :

Indian Council of Agricultural Research) and the Defence Institute of High Altitude Research
(under Defence Research and Development Organisation).

THINK!

· Permafrost

· Cryosphere

http://www.thehindu.com/sci-tech/science/whats-inside-the-svalbard-global-seed-vault/
article22858978.ece

QUESTION 87.
Recently a decision by government to bring prices of coronary stents under control has created
shock waves across medical community. Which authority has the mandate to control the prices
of such pharmaceutical products in India?

a) Ministry of Chemicals and Fertilizers


b) Ministry of Finance
c) Ministry of Health and Family welfare
d) Ministry of Women and Child Development
Correct Answer: A
Your Answer: Unanswered
Explanation

Solution (a)

National Pharmaceutical Pricing Authority is the regulatory authority to control pricing of


pharmaceutical drugs in India, which is under department of pharmaceuticals, Ministry of
Chemicals and Fertilizers.

THINK!

· Pros and cons of capping prices of medical instruments and medicines

http://www.thehindu.com/news/national/pharma-pricing-panel-chief-shunted-out/
article22897704.ece

QUESTION 88.
A supercritical carbon dioxide Brayton cycle test loop facility, recently in news is related to

a) Clean Energy
b) Hyperloop
c) Hydrogen Bomb
d) Space Programme
Correct Answer: A
Your Answer: Unanswered
Explanation

IASbaba
Web: http://ilp.iasbaba.com/ Score:
Email: ilp@iasbaba.com 4.00 / 200
Page 75
Exam Title : Test 21-Revision: Polity & ...
Email
Contact :

Solution (a)

Indian scientists have developed a super critical carbon di oxide Brayton test loop facility that
would help generate clean energy from future power plants including solar thermal.

· The new generation high efficiency power plants with closed cycle CO2 as the working fluid
have the potential to replace steam based nuclear and thermal power plants, thus reducing the
carbon foot print significantly.

· This test loop is designed to generate the necessary data for future development of scaled up
S-CO2 power plants, which would require overcoming several technological challenges –
developing critical components such as the turbine, compressor and heat exchangers that can
work at the desired pressure and temperature ranges and using materials that can withstand
these conditions.

· Today’s thermal power plants use steam to carry heat away from the source and turn a turbine
to generate power. However, it could generate more power if, instead of steam, supercritical
CO2 (SCO2) is used. The term “supercritical” describes the state of carbon dioxide above its
critical temperature of 31°C and critical pressure of 73 atmospheres making it twice as dense
as steam.

http://www.thehindu.com/todays-paper/tp-national/tp-karnataka/supercritical-co2-brayton-cycle-
facility-opened-at-iisc/article22830320.ece

QUESTION 89.
As per a recent initiative, Viral Load testing will be used for

a) Detecting amount of HIV in blood


b) Instant detection of H1N1
c) Security of critical cyber infrastructure
d) Preventing cybercrimes like ransomware
Correct Answer: A
Your Answer: Unanswered
Explanation

Solution (a)

The government on Monday launched a scheme to provide free of cost viral load testing, at
least once a year, for 1.2 million people who are living with HIV/AIDS (PLHIV) and are on
treatment.

The viral load is used to monitor the effectiveness of Antiretroviral Therapy (ART) over time. It
measures the amount of HIV genetic material (RNA) in the blood and reports how many copies
of the virus are present.

THINK!

· Antiretroviral Therapy (ART)

· HIV tests (ELISA,PCR)

IASbaba
Web: http://ilp.iasbaba.com/ Score:
Email: ilp@iasbaba.com 4.00 / 200
Page 76
Exam Title : Test 21-Revision: Polity & ...
Email
Contact :

QUESTION 90.
SWIFT transaction was in news recently, what is the abbreviation of the same?

a) Society for Worldwide Interbank Financial Transfer


b) Society for Worldwide Interbank Financial Telecommunication
c) Society for Worldwide and Internal Financial Transfer
d) Society for Worldwide Institutional Financial Telecommunication.
Correct Answer: B
Your Answer: Unanswered
Explanation

Solution (b)

The Society for Worldwide Interbank Financial Telecommunications (SWIFT) provides a


network that enables financial institutions worldwide to send and receive information about
financial transactions in a secure, standardized and reliable environment.

A SWIFT transfer is a type of international money transfer sent via the SWIFT international
payment network.

The Punjab National Bank fraud where fund transfer through an inter-bank messaging system
was not reported to the core banking solution has made use of SWIFT important for
international banking.

Do you know?

· The core role of SWIFT is to provide a secure transmission channel so that Bank A knows that
its message to Bank B goes to Bank B and no one else. Bank B, in turn, knows that Bank A, and
no one other than Bank A, sent, read or altered the message en route.

THINK!

· Basel Norms for banks

· Qualitative and quantitative measures of RBI for regulation of banks

http://www.thehindu.com/news/national/a-swift-autopsy-how-nirav-modi-defrauded-pnb/
article22844201.ece

QUESTION 91.
Consider the following statements with reference to Financial Action Task Force (FATF)

1. It is an intergovernmental organization.

2. Its mandate includes regulating banking, combating money laundering and terrorism
financing.

Which of the statements given above is/are correct?

a) Only 1

IASbaba
Web: http://ilp.iasbaba.com/ Score:
Email: ilp@iasbaba.com 4.00 / 200
Page 77
Exam Title : Test 21-Revision: Polity & ...
Email
Contact :

b) Only 2
c) Both 1 and 2
d) Neither 1 nor 2
Correct Answer: A
Your Answer: Unanswered
Explanation

Solution (a)

The Financial Action Task Force (FATF) is an inter-governmental body established in 1989.

Its mandate includes combating money laundering, terrorist financing and other related threats
to the integrity of the international financial system.

THINK!

· FATF Grey-list

· Legislative measures in India to counter financial crimes

https://timesofindia.indiatimes.com/india/india-congratulates-china-on-financial-action-task-
force-post-hopes-beijing-will-be-balanced/articleshow/63066160.cms

QUESTION 92.
Which of the following countries are part of ‘QUAD’?

1. India

2. U.S.A.

3. Russia

4. Japan

Select the correct answer using code below

a) 1 and 3
b) 2 and 4
c) 1,2 and 4
d) 1,2,3 and 4
Correct Answer: C
Your Answer: Unanswered
Explanation

Solution (c)

Quad (quadrilateral) refers to India, U.S.A., Japan and Australia.

THINK!

· How quad can counter the rise of China?

IASbaba
Web: http://ilp.iasbaba.com/ Score:
Email: ilp@iasbaba.com 4.00 / 200
Page 78
Exam Title : Test 21-Revision: Polity & ...
Email
Contact :

http://www.thehindu.com/todays-paper/tp-opinion/cornered-by-the-quad/article22872575.ece

QUESTION 93.
In order to increase vigilance on poaching and to control wife life related crimes, wildlife fast
track courts were set up in which of the following state?

a) Delhi
b) Bihar
c) Assam
d) Maharashtra
Correct Answer: C
Your Answer: Unanswered
Explanation

Solution (c)

For the first time in the country, 10 wildlife fast-track courts have been set up in Assam to
exclusively deal with poaching and other related crimes against wild animals.

Do you know?

· Assam has five national parks and 19 wildlife sanctuaries. Also, It is home to more than 91% of
Indian rhinos.

THINK!

· Legislative and International measures for protection of wildlife and biodiversity

· List of Biosphere reserve and national parks of Assam

http://www.thehindu.com/todays-paper/tp-national/guards-get-modern-weapons-to-fight-
poaching/article22793099.ece

QUESTION 94.
Which of the following are the objectives of GOBAR-DHAN scheme?

1. Cleanliness of villages

2. Wealth from waste

Select the correct answer using code below

a) Only 1
b) Only 2
c) Both 1 and 2
d) Neither 1 nor 2
Correct Answer: C
Your Answer: Unanswered

IASbaba
Web: http://ilp.iasbaba.com/ Score:
Email: ilp@iasbaba.com 4.00 / 200
Page 79
Exam Title : Test 21-Revision: Polity & ...
Email
Contact :

Explanation

Solution (c)

“GOBAR-Dhan” (Galvanising Organic Bio-Agro Resources-Dhan) scheme would focus on


managing and converting cattle dung and solid waste in farms to compost, biogas and bio-CNG.

The initiative has two objectives: To make villages clean and generate wealth and energy from
cattle and other waste.

THINK!

· Solid and liquid waste management

· Challenges in realizing waste to wealth

http://www.business-standard.com/article/opinion/govt-s-gobar-dhan-initiative-is-a-
commendable-plan-letter-to-bs-118022801336_1.html

QUESTION 95.
Consider the following statements with reference to Rustom-2

1. It is being developed by Defence Research and Development Organisation (DRDO).

2. It is a Medium Altitude Long Endurance drone (MALE).

Which of the statements given above is/are correct?

a) Only 1
b) Only 2
c) Both 1 and 2
d) Neither 1 nor 2
Correct Answer: C
Your Answer: Unanswered
Explanation

Solution (c)

Rustom-2 is an Unmanned Aerial Vehicle being developed by the Defence Research and
Development Organisation (DRDO).

It is a Medium Altitude Long Endurance drone (MALE).

The drone was developed for use by all three services of the Indian armed forces, primarily for
intelligence, surveillance and reconnaissance (ISR) operations.

Do you know?

· Rustom 2 can fly missions on manual as well as autonomous modes.

THINK!

· Indigenous defense production

IASbaba
Web: http://ilp.iasbaba.com/ Score:
Email: ilp@iasbaba.com 4.00 / 200
Page 80
Exam Title : Test 21-Revision: Polity & ...
Email
Contact :

https://economictimes.indiatimes.com/news/defence/drdo-successfully-carries-out-test-flight-of-
rustom-2-drone/articleshow/63068375.cms

QUESTION 96.
Consider the following statements with respect to World Environment Day, 2018

1. India is the global host of 2018 World Environment Day which will take place on June 5, 2018

2. The theme for WED, 2018 is ‘Beat Plastic Pollution’.

Which of statements given above is/are correct?

a) 1 and 2
b) 1 only
c) 2 only
d) None of the above
Correct Answer: A
Your Answer: Unanswered
Explanation

Solution (a)

World Environment Day [WED] was established by the UN General Assembly in 1972 on the
first day of Stockholm Conference on the Human Environment, resulting from discussions on
the integration of human interactions and the environment. Two years later, in 1974 the first
WED was held with the theme "Only One Earth".

India is the global host of 2018 World Environment Day which will take place on June 5, 2018.

With “Beat Plastic Pollution” as the theme for this year’s edition, the world is coming together
to combat single-use plastic pollution.

“Beat Plastic Pollution”, the theme for World Environment 2018, urges governments, industry,
communities, and individuals to come together and explore sustainable alternatives and
urgently reduce the production and excessive use of single-use plastic polluting our oceans,
damaging marine life and threatening human health.

Do you know?

· The theme for 2017 is 'Connecting People to Nature – in the city and on the land, from the
poles to the equator'. The host nation was Canada.

https://timesofindia.indiatimes.com/home/environment/pollution/drive-against-plastic-pollution-
to-gain-momentum-from-india-the-global-host-of-world-environment-day-2018/
articleshow/62994154.cms

QUESTION 97.

IASbaba
Web: http://ilp.iasbaba.com/ Score:
Email: ilp@iasbaba.com 4.00 / 200
Page 81
Exam Title : Test 21-Revision: Polity & ...
Email
Contact :

Recently Myanmar began construction of dam across a river close to Indian border which has
stroked fears of water scarcity in Manipur’s Chandel district. Which of the following states
doesn’t border Manipur?

a) Assam
b) Nagaland
c) Mizoram
d) Tripura
Correct Answer: D
Your Answer: Unanswered
Explanation

Solution (d)

THINK!

· Major rivers and lakes of Manipur

QUESTION 98.
Which of the following were involved in Sambandh exercise conducted by Western Naval
command of Indian Navy?

a) ASEAN
b) Indian Ocean littoral countries
c) BIMSTEC countries
d) India-Vietnam- Malaysia-Thailand.
Correct Answer: B
Your Answer: Unanswered
Explanation

Solution (b)

Participants were from the 10 Indian Ocean Littoral countries Maldives, Myanmar,

Indonesia, Sri Lanka, Bangladesh, Oman, Qatar, United Arab Emirates, Malaysia and Mauritius.

IASbaba
Web: http://ilp.iasbaba.com/ Score:
Email: ilp@iasbaba.com 4.00 / 200
Page 82
Exam Title : Test 21-Revision: Polity & ...
Email
Contact :

Naval officers and cadets from smaller countries that do not have big navies were welcomed
aboard INS Vikramaditya –India’s aircraft carrier as a part of the initiative.

THINK!

· Indian naval exercises

· Importance of conducting military exercises

http://www.livemint.com/Opinion/A2oieg9GPqS7R39VecSOOK/Building-maritime-capacity-in-
SouthEast-Asia.html

QUESTION 99.
Which of the following are the features of National Savings Certificates?

1. Can be kept as collateral.

2. Maximum investment is Rs. 5 Lakhs.

3. Trusts cannot invest in these.

Select the correct answer using code below

a) 1 and 2
b) 1 and 3
c) 2 and 3
d) 1,2 and 3
Correct Answer: B
Your Answer: Unanswered
Explanation

Solution (b)

National Savings Certificates (NSC) is an investment scheme floated by the Government of


India. It has following features:

· Scheme specially designed for Government employees, Businessmen and other salaried
classes who are Income Tax assesses.

· No maximum limit for investment.

· No Tax deduction at source.

· Certificates can be kept as collateral security to get loan from banks.

· Trust and HUF cannot invest.

Government is planning to merge different small savings schemes under various acts into a
single act, Government Savings Banks (GSB) Act, 1873.

THINK!

· Small savings schemes

IASbaba
Web: http://ilp.iasbaba.com/ Score:
Email: ilp@iasbaba.com 4.00 / 200
Page 83
Exam Title : Test 21-Revision: Polity & ...
Email
Contact :

· Government Savings Banks (GSB) Act, 1873

http://www.livemint.com/Money/v7KOGkxFAFMYnuiq1RawfN/Government-proposes-
amendments-in-small-savings-schemes.html

QUESTION 100.
Consider the following statements with reference to Chit Funds:

1. These are a kind of savings scheme practiced in India.

2. These are totally unregulated.

3. A transaction, where some alone but not all, get the prize money without obligation to pay
future installments, is not a chit fund.

Which of the statements given above are correct?

a) 1 and 2
b) 1 and 3
c) 2 and 3
d) 1,2 and 3
Correct Answer: B
Your Answer: D
Explanation

Solution (b)

Chit fund is a kind of savings scheme practiced in India.

Chit fund business is regulated under the Central Act of Chit Funds Act, 1982 and the Rules
framed under this Act by the various State Governments for this purpose.

A transaction is not a chit, if in such transaction, -

· Some alone, but not all, of the subscribers get the prize amount without any liability to pay
future subscriptions; or

· All the subscribers get the chit amount by turns with a liability to pay future subscriptions.

· Functionally, Chit funds are included in the definition of Non- Banking Financial Companies by
RBI under the sub-head miscellaneous non-banking company (MNBC). But RBI has not laid out
any separate regulatory framework for them.

· Now, a chit fund company needs to obtain a certificate of incorporation (CIN) from the
Registrar of Companies and then apply for registration with chit fund department of its
respective state.

Do you know?

· Amendments in Chit fund Act- Recently; Cabinet has given nod to amend Chit Funds Act,
1982. One of the amendments is the use of the words “Fraternity Fund” for chit business in the

IASbaba
Web: http://ilp.iasbaba.com/ Score:
Email: ilp@iasbaba.com 4.00 / 200
Page 84
Exam Title : Test 21-Revision: Polity & ...
Email
Contact :

Act, to signify its inherent nature, and distinguish its working from ‘Prize Chits’ which are
banned under a separate legislation.

THINK!

· Non-Banking Financial Companies

http://www.livemint.com/Money/aasZcBKy9PcW3jK0ZOi6dN/New-rules-for-chit-funds-and-
deposit-schemes.html

IASbaba
Web: http://ilp.iasbaba.com/ Score:
Email: ilp@iasbaba.com 4.00 / 200
Page 85
Exam Title : Test 21-Revision: Polity & ...
Email
Contact :

Review in Hindi
QUESTION 1.
नौ�जज��क��संवैधा�नक�पीठ�ने�घोषणा��कया�है��क� ' �नजता�का�अ�धकार�' सं�वधान�म��उ��ल�खत�एक�मूल�अ�धकार�है।�उ�चतम��यायालय�के
अनुसार� , �नजता�का�अ�धकार��न�न�ल�खत�म��से��कसका�आंत�रक�भाग�है ?

a) वाक् �एवं�अ�भ����क���वतं�ता�का�अ�धकार�
b) जीवन�तथा����गत��वतं�ता�का�अ�धकार�
c) समानता�का�अ�धकार�
d) शोषण�के ��व���अ�धकार�
Correct Answer: B
Your Answer:
Explanation

Solution (b)

भारत�के �चीफ�ज��टस�जे .एस. खेहर�क��अगुवाई�म��सव��च��यायालय�के �नौ�जज��से��मलकर�बने�सं�वधा�नक�पीठ�ने�सव�स�म�त�से�फै सला


�लया��क�सं�वधान�के �तहत��नजता�का�अ�धकार(राईट�टू ��ाइवेसी ) एक�मौ�लक�अ�धकार�है।�

सव��च��यायालय�का�फै सला�

· सव��च��यायालय�ने��नण�य��दया��क��नजता�का�अ�धकार (राईट�टू ��ाइवेसी ), सं�वधान�के �अनु�छे द� 21 के �तहत�जीवन�का�अ�धकार�(राइट�टू


लाइफ) एवं����गत��वतं�ता� (पस�नल��लबट� ) तथा�पूरे�भाग� III का�एक�आंत�रक��ह�सा�है।�

· इसने�सव��च��यायालय�के �पूव��के �दो�फै सल��को�खा�रज�कर��दया�है��क��नजता�का�अ�धकार�सं�वधान�के �तहत�संर��त�नह��ह�।�

�वचार�कर��

· �नजता�का�अ�धकार�

QUESTION 2.
1784 के ��पट् स�इं�डया�अ�ध�नयम�के �स�दभ��म���न�न�ल�खत�म��से�कौन -सा�कथन�सह��है� ?

1. इसने�कं पनी�के �वा�ण��यक�और�राजनी�तक�काय��पृथक् -पृथक् �कर��दया।�

2. इसने�राजनी�तक�मामल��के ��बंधन�के ��लए��नयं�ण�बोड��का�सृजन��कया।�

3. इसके ��ारा�बंगाल�और�म�ास��े�सड�सी�के �गवन�र�को�बंगाल�के �गवन�र�जनरल�के �अधीन�कर��दया�गया।�

नीचे�कू ट�से�चयन�क��जए :

a) 1 और�2
b) 2 और�3
c) 1 और�3
d) उपरो��सभी�
Correct Answer: A
Your Answer:
Explanation

Solution (a)

�पट् स�इं�डया�अ�ध�नयम� 1784

· इसने�कं पनी�के �वा�ण��यक�और�राजनी�तक�काय��पृथक् -पृथक् �कर��दया।�

IASbaba
Web: http://ilp.iasbaba.com/ Score:
Email: ilp@iasbaba.com 4.00 / 200
Page 86
Exam Title : Test 21-Revision: Polity & ...
Email
Contact :

· इसके ��ारा�कोट� �ऑफ़�डायरे�टस��को�वा�ण��यक�मामल��का��बंधन�करने�क��अनुम�त�द�� , ले�कन�राजनी�तक�मामल��के ��बंधन�के ��लए


बोड��ऑफ़�क��ोल�नामक�एक�नया��नकाय�बनाया।�इस��कार� , उसने��ै ध�सरकार�क��एक��णाली�को��था�पत��कया।�

· इसने�भारत�म�����टश�अ�धकृ त�भाग��के �नाग�रक�तथा�सै�य�सरकार�या�राज�व�के �सभी�काय��क��दे खरेख�और��नद� शन�करने�के ��लए��नयं�ण


बोड��को�सश��बनाया।�

· इस��कार�, अ�ध�नयम�दो�कारण��के ��लए�मह�वपूण��था : पहला�, भारत�म��कं पनी�के ��े���को�पहली�बार� ' भारत�म�����टश�संप��� ' कहा
गया�; और��सरा�, ���टश�सरकार�को�कं पनी�के �मामल��और�भारत�म��उसके ��शासन�पर�सव��च��नयं�ण��दया�गया।�

नोट: तीसरा�कथन�रेगुले�ट�ग�ए�ट� 1773 क��एक��वशेषता�है।�

�वचार�कर��

· बोड��ऑफ़�क��ोल�

· कोट� �ऑफ़�डायरे�टर�

QUESTION 3.
�न�न�ल�खत�म��से��कसे� ' सां�दा�यक��नवा�चक�म�डल�का��पता� ' कहा�जाता�है� ?

a) लॉड��चे�सफोड��
b) लॉड���म�टो�
c) लॉड��डलहौज़ी�
d) लॉड��कज�न�
Correct Answer: B
Your Answer:
Explanation

Solution (b)

लॉड���म�टो�ने�मॉल� -�म�टो�सुधार (भारतीय�प�रषद�अ�ध�नयम�1909) को���तुत��कया।�इस�दौरान�पहली�बार�मुसलमान�� , ब�बई�वा�ण�यक


मंडल�तथा�मदरसा�एवं�जम�दार��के ��लए�पृथक��नवा�चकम�डल (सेपरेट�एले�टोरेट ) क��शु�आत�क�।�

चूं�क�यह�सां�दा�यक�बु�नयाद� (मु��लम��के ��लए) पर�आधा�रत�था�, इस�लए��म�टो�को�सां�दा�यक��नवा�चक�म�डल��का��पता�भी�कहा�जाता


है।�

1909 के �अ�ध�नयम�क���वशेषताएं�

1. इस�अ�ध�नयम�को�मॉल� -�म�टो��रफॉ�स�� (लॉड��मोल��भारत�के �त�कालीन�स�चव�रा�य�के ��प�म��भी�जाना�जाता�है�और�लॉड���म�टो�भारत�का


त�कालीन�वायसराय�था)।�

2. इसके ��ारा�क� ��य�और��ांतीय�दोन�� , �वधान�प�रषद��के �आकार�म��काफ��वृ����ई�है।�के ���य��वधान�प�रषद�म��सद�य��क��सं�या� 16 से


बढ़कर�60 कर��दया�गया।��ांतीय��वधान�प�रषद��म��सद�य��क��सं�या�एकसमान�नह��थी।�

3. इसके ��ारा�के ���य��वधान�प�रषद�म��आ�धका�रक�ब�मत�को�बरकरार�रखा�गया� , ले�कन��ांतीय��वधान�प�रषद��को�गैर -सरकारी�ब�मत


�ा�त�करने�क��अनुम�त�द��गई।�

4. इसने�दोन���तर��पर��वधान�प�रषद��के ��वमश��काय��को�बढ़ा��दया।�उदाहरण�के ��लए�, सद�य��को�पूरक����पूछने� , बजट�पर���ताव�लाने�


, और�इसी�तरह�के �अ�य�क��अनुम�त�द��गई।�

5. इसके ��ारा�सव��थम�वाइसरॉय�तथा�गवन�स��के �काय�कारी�प�रषद��म��भारतीय��शा�मल��कया�गया।�स�य����साद��स�हा�वायसराय�क�


काय�कारी�प�रषद�म��शा�मल�होने�वाले�पहले�भारतीय�बने।�उ�ह���व�ध�सद�य�के ��प�म��शा�मल��कया�गया�था।�

IASbaba
Web: http://ilp.iasbaba.com/ Score:
Email: ilp@iasbaba.com 4.00 / 200
Page 87
Exam Title : Test 21-Revision: Polity & ...
Email
Contact :

6. ' पृथक��नवा�चक�म�डल� ' क��अवधारणा�को��वीकार�करके �मुसलमान��के ��लए�सां�दा�यक���त�न�ध�व��णाली�क��शु�आत�क��गई ;


इसके �तहत�मु��लम�सद�य��को�के वल�मु��लम�मतदाता��के ��ारा�चुना�गया।�इस��कार� , इस�अ�ध�नयम�के ��ारा�' सां�दा�यकता�को�वैधता� '
�दान�क��गयी�तथा�लॉड���म�टो�को�सां�दा�यक��नवा�चकम�डल�के ��पता�कहा�जाता�है।�

7. �ेसीड�सी�कारपोरेशन�� , चै�बर�ऑफ़�कॉमस� , �व��व�ालय��और�जम�दार��के ���त�न�धय��के ��लए�भी�पृथक��नवा�चक�म�डल�क���व�था


क��गई।�

�वचार�कर��

· क�युनल�अवाड��

· पुना�समझौता�

QUESTION 4.
भारत�के �सं�वधान�सभा�के �स�दभ��म���न�न�ल�खत�कथन��पर��वचार�क��जए :

1. यह�एक�स��भु��नकाय�थी।�

2. इसने�एक�सं�वधान��नमा��ी��नकाय�के �साथ�ही�साथ�ही�एक��वधा�यका�के ��प�म��भी�काय���कया।�

3. इसने�रा�मंडल�म��भारत�क��सद�यता�क��पु���क�।�

उपरो��कथन��म��से�कौन�सा /से�सही�है� ?

a) 1 और�2
b) 2 और�3
c) 1 और�3
d) उपरो��सभी�
Correct Answer: D
Your Answer:
Explanation

Solution (d)

1947 के �भारतीय��वतं�ता�अ�ध�नयम�ने�सं�वधान�सभा�क����थ�त�म���न�न�ल�खत�तीन�प�रवत�न��कए :

1. सभा�को�पूण�त : सं�भु��नकाय�बना��दया�गया� , जो�अपनी�इ�छा�से�कोई�भी�सं�वधान�बना�सकती�थी।�इस�अ�ध�नयम�ने�सभा�को�भारत�के


संबंध�म�����टश�संसद��ारा�बनाए�गए��कसी�भी�कानून�को�र��करने�या�बदलने�का�अ�धकार��दया�गया।�

2. सभा�एक��वधायी��नकाय�भी�बन�गई।��सरे�श�द��म�� , सभा�को�दो�अलग-अलग�काय��स�पे�गए� , पहला; �वतं��भारत�के ��लए�एक�सं�वधान


का��नमा�ण�करना�और��सरा�दे श�के ��लए�सामा�य�कानून�बनाना�है।�ये�दोन��काय��अलग -अलग��दन��पर��कए�जाने�थे।�इस��कार� , यह�सभा
�वतं��भारत� (डो�म�नयन��वधानमंडल ) क��पहली�संसद�बनी।�जब�भी�सभा�क��बैठक�सं�वधान�सभा�के ��प�म��होती�थी� , तब�इसक��अ�य�ता
डॉ. राज����साद�के ��ारा��कया�जाता�था�और�जब�बैठक��वधायी��नकाय�के ��प�म��होती�थी�तो�इसक��अ�य�ता�जी . वी. मावलंकर�अ�य�ता
के ��ारा��कया�जाता�था।�सं�वधान�सभा� 26 नवंबर� , 1949 इन�दोन���प��म��काय��करती�रही� , इस�समय�तक�सं�वधान��नमा�ण�का�काय��पूरा
हो�चूका�था।�

3. मु��लम�लीग�के �सद�य�(पा�क�तान�म��शा�मल��कये�गए��े���से�स�ब��धत ) भारत�के ��लए�सं�वधान�सभा�से�अलग�हो�गए।�नतीजतन� , कै �ब


नेट��मशन�योजना�के �तहत�1946 म���नधा��रत� 389 सीट��के �मुकाबले�सद�य��क��कु ल�सं�या� 299 तक�हो�गई।�भारतीय��ांत�� (पूव��म�����टश
�ांत ) क��सं�या� 296 से� 229 तक�कम�हो�गई�और��रयासत��क��सं�या� 93 से� 70 हो�गई।�

अ�य�काय��

सं�वधान�बनाने�और�साधारण�कानून�बनाने�के �अलावा�, सं�वधान�सभा�ने�भी��न�न�ल�खत�काय���कया :

IASbaba
Web: http://ilp.iasbaba.com/ Score:
Email: ilp@iasbaba.com 4.00 / 200
Page 88
Exam Title : Test 21-Revision: Polity & ...
Email
Contact :

· मई�1949 म��इसके ��ारा�भारत�क��रा�मंडल�म��सद�यता�क��पु���क��गई।�

· 22 जुलाई� , 1947 को�इसके ��ारा�रा�ीय��वज�को�अपनाया�गया।�

· 24 जनवरी�1950 को�इसके ��ारा�रा�ीय�गान�को�अपनाया�गया।�

· 24 जनवरी�1950 को�इसके ��ारा�रा�ीय�गीत�अपनाया�गया।�

· 5 जनवरी�, 1950 को�डॉ�राज����साद�को�भारत�का�पहला�रा�प�त�चुना�गया।�

�वचार�कर��

· भारतीय��वतं�ता�अ�ध�नयम� 1947

QUESTION 5.
�न�न�ल�खत�म��से�कौन�सी�स�म�तयां�अपने�अ�य��के �साथ�सुमे�लत�नह��ह�� ?

a) संचालन�स�म�त� - डॉ. राज����साद�


b) �ा�प�स�म�त�- डॉ. बी. आर. अ�बेडकर�
c) रा�य�स�म�त�- सरदार�पटे ल�
d) संघ�श���स�म�त - पं . नेह��
Correct Answer: C
Your Answer:
Explanation

Solution (c)

�मुख�स�म�तयां�

1. संघ�श���स�म�त� - जवाहरलाल�नेह��

2. संघीय�सं�वधान�स�म�त� - जवाहरलाल�नेह��

3. �ांतीय�सं�वधान�स�म�त� - सरदार�पटे ल�

4. �ा�प�स�म�त�- डॉ. बी.आर. अ�बेडकर�

5. मौ�लक�अ�धकार�, अ�पसं�यक�एवं�जनजातीय�और�ब�ह�कृ त��े���के ��लए�सलाहकार�स�म�त�- सरदार�पटे ल�

6. इस�स�म�त�म���न�न�उप -स�म�तयां�थ� :

a. मौ�लक�अ�धकार�उप-स�म�त- जे .बी. कृ पालानी�

b. अ�पसं�यक�उप -स�म�त�- एच.सी. मुखज��

c. उ�र-पूव��सीमांत�जनजातीय��े�� , असम�को�छोड़कर�तथा�आं�शक��प�से�ब�ह�कृ त��े��उप -स�म�त- गोपीनाथ�बरदोई�

d. अपव�ज�त�और�आं�शक��प�से�ब�ह�कृ त��े��� (असम�म���े���के �अलावा) के ��लए�उप-स�म�त- ए.वी. ठ�कर�

7. ���या�स�म�त�के ��नयम�- डॉ�राज����साद�

8. रा�य�स�म�त�(रा�य��के �साथ�बातचीत�के ��लए�स�म�त) - जवाहरलाल�नेह��

9. संचालन�स�म�त� - डॉ�राज����साद�

IASbaba
Web: http://ilp.iasbaba.com/ Score:
Email: ilp@iasbaba.com 4.00 / 200
Page 89
Exam Title : Test 21-Revision: Polity & ...
Email
Contact :

QUESTION 6.
�न�न�ल�खत�म��से�कौन -सा�कथन�भारतीय��यायपा�लका�के �बारे�म��सही�ह�� ?

1. भारतीय��या�यक��णाली�एक�कृ त�होने�के �साथ�ही�साथ��वतं��भी�है।�

2. �यायालय��क��एकल��णाली�क� ��य�कानून��के �साथ-साथ�रा�य�के �कानून��को�भी�लागू�करती�है�

नीचे�कू ट�म��से�चुनाव�क��जए :

a) के वल�1
b) के वल�2
c) दोन��1 और�2
d) न�तो�1 और�न�ही�2
Correct Answer: C
Your Answer:
Explanation

Solution (c)

एक�कृ त�और��वतं���यायपा�लका�

भारतीय�सं�वधान�एक�ऐसी��या�यक��णाली�को��था�पत�करता�है�जो�एक�कृ त�होने�के �साथ�ही�साथ��वतं��है।�

दे श�म��एक�कृ त��या�यक��णाली�के �शीष��पर�उ�चतम��यायालय (सु�ीम�कोट� ) है।�इसके �नीचे� , रा�य��तर�पर�उ�च��यायालय(हाई�कोट� ) ह�।�उ�च


�यायालय�के �नीचे� , �मब���प�से�अधीन�थ��यायालय�है� , अथा�त� , �जला��यायालय�और�अ�य��नचली�अदालत�।��यायालय��क��यह�एकल
�णाली, संयु���प�से�के ���य�कानून�के �साथ-साथ�रा�य�कानून��को�भी�लागू�करती�है� , हालां�क�इसके ��वपरीत�संयु��रा�य�अमे�रका�म�� , स
घीय�कानून�को�संघीय��यायपा�लका��ारा�तथा�रा�य�कानून��को�रा�य��यायपा�लका�लागू��कया�जाता�ह�।�

उ�चतम��यायालय, एक�संघीय��यायालय�है।�यह�शीष���यायालय�है�जो�नाग�रक��के �मूल�अ�धकार��क��र�ा�क��गारंट��दे ता�है�और�सं�वधान�का


संर�क (गा�ड�यन�ऑफ़�द�कं �ट��ूशन ) है।�इस�लए� , सं�वधान�म��इनक���वतं�ता�के ��लए�कई��ावधान��कये�गए�है -जज��के �काय�काल�क�
सुर�ा� , �यायाधीश��के ��लए�तय�क��गई�सेवा�शत�� , भारत�के �सं�चत��न�ध�से�सव��च��यायालय�के �सभी�ख़च��का�वहन�, �वधा�यका�म�
�यायाधीश��के �कामकाज�पर�चचा��पर�रोक� , सेवा�नवृ���के �बाद�कामकाज�पर���तबंध� , उ�चतम��यायालय�क��अवमानना� के ��लए�दं �डत�करने
क��श���, काय�पा�लका�से��यायपा�लका�का�पृथ�करण� , आ�द।�

�वचार�कर��

· संघीय�ढांचे�के ��लए�एक�कृ त��यायपा�लका�मह�वपूण��कै से�है� ?

QUESTION 7.
भारत�एक�धम��नरपे��रा�य�है , भारतीय�धम��नरपे�ता�के �स�दभ��म���न�न�ल�खत�म��से�कौन�सा�कथन�सही�है� ?

1. भारत�रा�य�का�कोई�भी�आ�धका�रक�धम��नह��है।�

2. धम��के �आधार�पर�रा�य��कसी�भी�नाग�रक�के ��व���भेदभाव�नह��करता�है।�

3. धम��तथा�रा�य�का�एक�सरे�से�पूण�त : अलगाव�है।�

नीचे�कू ट�से�चयन�क��जए :

a) 1 और�2
b) 2 और�3
c) 1 और�3
d) उपरो��सभी�

IASbaba
Web: http://ilp.iasbaba.com/ Score:
Email: ilp@iasbaba.com 4.00 / 200
Page 90
Exam Title : Test 21-Revision: Polity & ...
Email
Contact :

Correct Answer: A
Your Answer:
Explanation

Solution (a)

एक�धम��नरपे��रा�य�

भारत�का�सं�वधान�एक�धम��नरपे��रा�य�क���व�था�करता�है।�यह��कसी�भी��व�श��धम��को�भारत�के �आ�धका�रक�धम��के ��प�मा�यता�नह�


दे ता�है।�सं�वधान�के ��न�न�ल�खत��ावधान�भारत�के �धम��नरपे��च�र��को��कट�करते�ह� :

a) 1976 के �42व��सं�वधान�संशोधन�अ�ध�नयम�के ��ारा�भारतीय�सं�वधान�के ���तावना�म�� ' धम��नरपे�� ' श�द�को�जोड़ा�गया।�

b) ��तावना�भारत�के �सभी�नाग�रक��क��आ�था�, �व�ास�और�पूजा -अच�ना�क���वतं�ता�को�सुर��त�करती�है।�

c) रा�य�के ��ारा, �कसी�भी�����को�कानून�के �सम��समान�समझा�जायेगा�या�उसे�कानून�का�समान�संर�ण� (अनु�छे द� 14) �दान��कया


जायेगा।�

d) रा�य, धम��के �आधार�पर��कसी�भी�नाग�रक�के ��खलाफ�भेदभाव�नह��करेगा� (अनु�छे द� 15)।�

e) साव�ज�नक�रोजगार�के �मामले�म��सभी�नाग�रक��के ��लए�अवसर�क��समानता�(अनु�छे द� 16)।�

f) हर�����को��कसी�भी�धम��को�अपनाने�व�उसके �अनुसार�पूजा�अच�ना�करने�का�समान�अ�धकार�है (अनु�छे द� 25)

g) हर�धा�म�क�सं�दाय�या�उसके ��कसी�भी�अनुभाग�को�अपने�धा�म�क�मामल��के ��बंधन� (अनु�छे द� 26) का�अ�धकार�होगा।�

h) �कसी�����को�एक��वशेष�धम��के ��चार�के ��लए��कसी��कार�के �कर�के �भुगतान�करने�के ��लए�मजबूर�नह���कया�जाएगा (अनु�छे द� 27)।�

i) रा�य��ारा�अनुर��त��कसी�शै��क�सं�थान�म��कोई�धा�म�क��श�ा�नह��द��जाएगी� (अनु�छे द� 28)।�

j) नाग�रक��के ��कसी�भी�वग��को�अपनी�अलग�भाषा� , �ल�प�या�सं�कृ �त�(अनु�छे द� 29) को�संर��त�करने�का�अ�धकार�होगा।�

k) सभी�अ�पसं�यक��को�अपनी�पसंद�के �शै��णक�सं�थान�� (अनु�छे द� 30) को��था�पत�करने�और�उनका�संचालन�करने�का�अ�धकार�होगा।�

l) रा�य, सभी�नाग�रक��के ��लए�समान�नाग�रक�सं�हता� (अनु�छे द� 44) बनाने�के ��लए��यास�करेगा।�

धम��नरपे�ता�क��प��मी�अवधारणा�धम�� (चच� ) और�रा�य�(राजनी�त) के �बीच�पूण��अलगाव�का��तीक�है।�धम��नरपे�ता�क��यह�नकारा�मक


अवधारणा�भारतीय�प�र��थ�तय��म��लागू�नह��हो�सकती� , �य��क�यहाँ�का�समाज�ब��धा�म�क�है।�इस�लए� , भारतीय�सं�वधान�म��धम��नरपे�ता
क��सकारा�मक�अवधारणा�अथा�त्� , सभी�धम��को�समान�स�मान�दे ना�या�सभी�धम��को�समान��प�से�र�ा�करना ; को�शा�मल��कया�गया�है।�

इसके �अलावा�, सं�वधान�ने�धम��के �आधार�पर��वधा�यका�म��सीट��के �आर�ण�वाले�सां�दा�यक���त�न�ध�व�के �पुराने��व�था�को�समा�त�कर


�दया�है।�हालां�क� , यह�अनुसू�चत�जा�तय��और�अनुसू�चत�जनजा�तय��के ��लए�सीट��का�अ�थायी�आर�ण��दान�करता�है�ता�क�उ�ह��उ�चत
��त�न�ध�व��मल�सके ।�

QUESTION 8.
भारतीय�सं�वधान�म���न�न�ल�खत�म��से�कौन -कौन�सी��वशेषताएं�भारत�शासन�अ�ध�नयम� 1935 से�ली�गई�ह�� ?

1. संघीय�योजना�

2. आपातकालीन��ावधान�

3. �यायपा�लका�

4. रा�यपाल�का�पद�

नीचे�कू ट�से�चयन�क��जए� ?

IASbaba
Web: http://ilp.iasbaba.com/ Score:
Email: ilp@iasbaba.com 4.00 / 200
Page 91
Exam Title : Test 21-Revision: Polity & ...
Email
Contact :

a) 1,2 और�3
b) 1,3 और�4
c) 2,3 और�4
d) उपरो��सभी�
Correct Answer: D
Your Answer:
Explanation

Solution (d)

सं�वधान�के ��ो�, एक�नज़र�म��

1. भारत�शासन�अ�ध�नयम�1935: संघीय�तं�� , रा�यपाल�का�पद�, �यायपा�लका�, लोक�सेवा�आयोग� , आपातकालीन��ावधान�और


�शास�नक��ववरण।�

2. ���टश�सं�वधान : संसद�य�शासन� , �व�ध�का�शासन�, �वधायी����या�, एकल�नाग�रकता�, कै �बनेट��णाली� , परमा�धकार�लेख� , संसद�य


�वशेषा�धकार�और���सदनीयता।�

3. अमे�रक��सं�वधान : मौ�लक�अ�धकार�, �यायपा�लका�क���वतं�ता� , �या�यक�समी�ा�, रा�प�त�पर�महा�भयोग�, उ�चतम��यायालय�और


उ�च��यायालय�के ��यायाधीश��को�पद�से�हटाना�और�उपरा�प�त�का�पद।�

4. आय�रश�सं�वधान : रा�य�के �नी�त��नद� शक��स�ांत� , रा�यसभा�के �सद�य��का�नामांकन�और�रा�प�त�के �चुनाव�क���व�ध।�

5. कनाडा�का�सं�वधान : एक�मजबूत�क� ��के �साथ�संघीय��व�था� , अव�श��श��य��का�क� ��म���न�हत�होना� , क� ���ारा�रा�य�के �रा�यपाल�


क���नयु���और�उ�चतम��यायालय�का�सलाहकारी��े�ा�धकार।�

6. ऑ��े �लया�का�सं�वधान : समवत��सूची� , �ापार�, वा�ण�य�और�समागम�क���वतं�ता� , और�संसद�के �दोन��सदन��क��संयु��बैठक।�

7. जम�नी�का�वाईमर�सं�वधान : आपातकाल�के �दौरान�मूल�अ�धकार��का��नलंबन�

8. सो�वयत�सं�वधान� (यूएसएसआर� , अब��स): ��तावना�म��मौ�लक�कत����और��याय�का�आदश�� (सामा�जक�, आ�थ�क�और�राजनी�तक )

9. �ांसीसी�सं�वधान : गणतं��और���तावना�म���वतं�ता� , समानता�और�बंधु�व�के �आदश�।�

10. द��ण�अ��क��सं�वधान : सं�वधान�म��संशोधन�क�����या�और�रा�यसभा�के �सद�य��का��नवा�चन।�

11. जापानी�सं�वधान : �व�ध��ारा��था�पत����या�

QUESTION 9.
��तावना�भारत�के �हर�नाग�रक�के ��लए�' �याय�' का�वचन�दे ता�है���तावना�म���कस�तरह�के ��याय�का�वचन��दया�गया�है� ?

1. सामा�जक�

2. आ�थ�क�

3. राजनी�तक�

4. शारी�रक�

नीचे�कू ट�से�चयन�क��जए� ?

a) 1,2 और�3
b) 2,3 और�4
c) 1,3 और�4

IASbaba
Web: http://ilp.iasbaba.com/ Score:
Email: ilp@iasbaba.com 4.00 / 200
Page 92
Exam Title : Test 21-Revision: Polity & ...
Email
Contact :

d) उपरो��सभी�
Correct Answer: A
Your Answer:
Explanation

Solution (a)

�याय�

��तावना�म�� ' �याय�' श�द�तीन��भ���प�-सामा�जक�, आ�थ�क�और�राजनी�तक�म��शा�मल�है� , इनक��सुर�ा�मूल�अ�धकार��और��नद� शक


�स�ांत��के ��व�भ���ावधान��के �मा�यम�से�क��जाती�है।�

सामा�जक��याय�का�अथ��है - सभी�नाग�रक��के �साथ�जा�त�, रंग� , धम�� , �ल�ग�के �आधार�पर��बना�भेदभाव��कये�समान��वहार।�इसका�मतलब


है��क�समाज�म���कसी�भी��वशेष�वग��के ��लए��वशेषा�धकार��क��अनुप��थ�त�और��पछड़े�वग�� (एससी�, एसट��और�ओबीसी) तथा�म�हला��क�
��थ�त�म��सुधार।�

आ�थ�क��याय�का�अथ��है�आ�थ�क�कारक��के �आधार�पर��कसी�भी�����से�भेदभाव�नह���कया�जायेगा।�इसम��धन� , आय�और�संप���क�


असमानता�का�उ�मूलन�भी�शा�मल�है।�सामा�जक��याय�और�आ�थ�क��याय�का��मलाजुला��प� ' �वतरणना�मक��याय�' को�प�रल��त�करता�है।

राजनी�तक��याय�का�अथ��है��क�सभी�नाग�रक��के �समान�राजनै�तक�अ�धकार��ा�त�ह�गे� , चाहे�राजनी�तक�काया�लय��तक�समान�प�ंच�क��बात


हो�अथवा�अपनी�बात�सरकार�तक�प�ँचाने�का�अ�धकार।�

सामा�जक�, आ�थ�क�और�राजनी�तक��याय�के �इन�आदश��को��सी��ां�त� (1917) से��लया�गया�है।�

�वचार�कर��

· ��तावना�म���यु��श�द�ब�त�ही��प��होने�चा�हए।�

QUESTION 10.
भारतीय�सं�वधान�के ���तावना�म���न�हत� ' बंधु�व� ' श�द�का�अथ��है :

1. भारत�के �सभी�नाग�रक��म��म�य�भाईचारे�क��भावना।�

2. यह�एक�����क��ग�रमा�का�आ�ासन�दे ता�है।�

3. यह�रा��क��एकता�और�अखंडता�का�आ�ासन�दे ता�है।�

नीचे�कू ट�से�चयन�क��जए :

a) के वल�1
b) 2 और�3
c) 1 और�3
d) उपरो��सभी�
Correct Answer: D
Your Answer:
Explanation

Solution (d)

बंधु�व�

बंधु�व�का�अथ��है - भाइचारे�क��भावना।�सं�वधान�एकल�नाग�रकता�क���व�था�से�भाईचारे�क��भावना�को�बढ़ावा�दे ता�है।�इसके �अलावा�, मूल


कत��� (अनु�छे द� 51-ए) म��कहा�गया�है��क�यह�भारत�के ���येक�नाग�रक�का�कत���होगा� , �क�वह�धा�म�क� , भाषाई�, �े�ीय�या�वग�
�व�वधता��से�ऊपर�उठकर�सभी�लोग��के �बीच�समानता�और�आपसी�भाईचारे�क��भावना�को�बढ़ावा�दे गा।�

IASbaba
Web: http://ilp.iasbaba.com/ Score:
Email: ilp@iasbaba.com 4.00 / 200
Page 93
Exam Title : Test 21-Revision: Polity & ...
Email
Contact :

��तावना�म��कहा�गया�है��क�बंधु�व�म��दो�बात��को�सु�न��त�करना�होगा - ����क��ग�रमा�तथा�दे श�क��एकता�और�अखंडता।�

42वां�संवैधा�नक�संशोधन� (1976) के ��ारा���तावना�म�� ' अखंडता� ' श�द�को�जोड़ा�गया।�

सं�वधान�सभा�के ��ा�प�स�म�त�के �एक�सद�य�के.एम. मुंशी�के �अनुसार� , वा�यांश� ' ����क��ग�रमा�' का�अथ��यह�है��क�सं�वधान�न�के वल


भलाई�और�लोकतां��क��व�था�को�सु�न��त�करता�है� , ब��क�यह�भी��वीकार�करता�है��क�हर�����का�����व�प�व��है�इसे�मूल�अ�धकार�
और�रा�य�के �नी�त��नद� शक��स�ांत��के �कु छ��ावधान��के �मा�यम�से�रेखां�कत��कया�गया�है� , जो����य��क��ग�रमा�सु�न��त�करता�है।�इसके
अलावा�, मौ�लक�कत���� (अनु�छे द� 51 ए) म��भी�कहा�गया�है , �क�यह�भारत�के ���येक�नाग�रक�का�कत���होगा��क�वह�म�हला��के �स�मान
को�ठे स�प�चने�वाले��कसी�भी�हरकत�का��याग�कर��और�भारत�क��सं�भुता� , एकता�और�अखंडता�क��र�ा�कर�।�

' रा��क��एकता�और�अखंडता� ' वा�यांश�म��रा�ीय�अख�डता�म��मनोवै�ा�नक�और��े�ीय�आयाम��दोन��को�शा�मल��कया�गया�है।�सं�वधान�के


अनु�छे द� 1 म��भारत�को� ' रा�य��का�संघ� ' कहा�गया�है�ता�क�यह��प��हो�जाये��क�रा�य��को�संघ�से�अलग�होने�का�कोई�अ�धकार�नह��है� , जो
भारतीय�संघ�के �अ�वनाशी��वभाव�को�बताता�है।�इसका�उ�े �य�सां�दा�यकता� , �े�ीयवाद� , जा�तवाद�, भाषा�व�ान�, अलगाववाद�आ�द�जैसे
रा�ीय�एक�करण�के ��लए�बाधक�त�व��पर�काबू�पाना�है।�

QUESTION 11.
भारतीय�सं�वधान�भारत�को� ' रा�य��के �संघ� ' के ��प�म��प�रभा�षत�करता�है� ' भारत�का�संघ� ' श�द�म���या -�या�शा�मल�है� ?

1. रा�य�

2. संघ�शा�सत��दे श�

3. भारत�सरकार��ारा�अ�ज�त��कए�जा�सकने�वाले��े��

नीचे�कू ट�से�चयन�क��जए :

a) के वल�1
b) 1 और�2
c) 1,2 और�3
d) 1 और�3
Correct Answer: A
Your Answer:
Explanation

Solution (a)

भारत�का�संघ�

' भारत�का��े�� ' ' भारत�का�संघ� ' क��तुलना�म��अ�धक��ापक�अथ��को�समेटे��ए�है��य��क�बाद�वाले�म��के वल�रा�य�शा�मल�ह�� , जब�क�पहल


म��न��सफ� �रा�य�ब��क�क� ��शा�सत��दे श�और�वे��दे श�शा�मल�ह���ज�ह��क� ��सरकार��ारा�भ�व�य�म��कभी�भी��ा�त��कया�जा�सकता�ह��।�संघीय
�णाली�म��रा�य�इसके �सद�य�ह��और�क� ��के �साथ�श��य��का��वतरण�साझा�करते�ह�।��सरी�तरफ�क� ��शा�सत��दे श�और�क� ���ारा�अ�ध�हीत
�े�� , सीधे�क� ��सरकार��ारा��शा�सत��कए�जाते�ह�।�

�वचार�कर��

· भारतीय�सं�वधान�म�� ' रा�य�' के ��व�भ��अथ�।�

QUESTION 12.
भारत�को�' �वनाशकारी�रा�य��का�अ�वनाशी�संघ� ' कहा�जाता�है ; �न�न�ल�खत�म��से�कौन�सा�कथन�इस��ववरण�का�समथ�न�करता�है� ?

1. संसद�अपनी�इ�छा�के �अनुसार�भारत�के �राजनी�तक�मान�च��को�पुनः�रेखां�कत�कर�सकती�है।�

IASbaba
Web: http://ilp.iasbaba.com/ Score:
Email: ilp@iasbaba.com 4.00 / 200
Page 94
Exam Title : Test 21-Revision: Polity & ...
Email
Contact :

2. उनक��अपनी�सीमा�या��े��म��प�रवत�न�से�पूव��रा�य��क��सहम�त�आव�यक�है।�

3. नए�रा�य��का�गठन�, सीमा�और�नाम�प�रवत�न�को�अनु�छे द� 368 के �तहत�सं�वधान�संशोधन�के �अंतग�त�माना�जाता�है।�

नीचे�कू ट�से�चयन�क��जए :

a) के वल�1
b) 2 और�3
c) 1 और�3
d) उपरो��सभी�
Correct Answer: A
Your Answer:
Explanation

Solution (a)

अनु�छे द� 3 संसद�को�अ�धकृ त�करता�है :

a) �कसी�भी�रा�य�से�उसके ��े��को�अलग�करके �या�दो�या�दो�से�अ�धक�रा�य��को�या�रा�य��के �कु छ��ह�स��को��मलाकर�या��कसी�भी�रा�य


�े��को��कसी�रा�य�के �भाग�के �साथ��मलाकर�नए�रा�य�का��नमा�ण�कर�सके गी�,

b) �कसी�भी�रा�य�के ��े��म��वृ���कर�सके गी�,

c) �कसी�भी�रा�य�के ��े��को�घटा�सके गी�,

d) �कसी�रा�य�क��सीमा��म��प�रवत�न�कर�सके गी�, और�

e) �कसी�भी�रा�य�के �नाम�म��प�रवत�न�कर�सके गी�

हालां�क� , इस�स�ब�ध�म��अनु�छे द� 3 म��दो�शत��का�उ�लेख��कया�गया�है : पहला�, उपरो��प�रवत�न��से�स�बं�धत�कोई�अ�यादे श�रा�प�त�के


पूव���सफा�रश�के �बाद�ही�संसद�म��पेश��कया�जा�सकता�है� ; और��सरा�, सं�तु�त�से�पहले� , रा�प�त�उस�अ�यादे श�को�स�बं�धत�रा�य
�वधानमंडल�का�मत�जानने�के ��लए�भेजता�है ; यह�मत��न��त�सीमा�के �भीतर��दया�जाना�चा�हए।�

इसके �अलावा�, नए�रा�य��के ��नमा�ण�करने�क��श���म���कसी�भी�रा�य�या�क� �शा�सत��दे श�के ��कसी�भी��ह�से�को��कसी�अ�य�रा�य�या


क� �शा�सत��दे श�को�एकजुट�करके �नए�रा�य�या�क� �शा�सत��दे श�बनाने�क��श���शा�मल�है।�

रा�प�त�(या�संसद ) रा�य��वधा�यका�के ��वचार��के �मत�को�मानने�के ��लए�बा�य�नह��है , और�इसे��वीकार�या�अ�वीकार�कर�सकती�है� , भले�ही


उसका�मत�समय�पर�आ�गया�हो।�इसके �अलावा�, जब�भी��वधेयक�म��संशोधन��कया�जाता�है�और�संसद�म���वीकार��कया�जाता�है� , हर�बार
रा�य��वधा�यका�को�एक�नया�संदभ��बनाना�आव�यक�नह��है।�संघ�शा�सत��दे श�के �मामले�म�� , संबं�धत��वधा�यका�के �संदभ��क��कोई
आव�यकता�नह��है�और�संसद�जब�उ�चत�समझे��वयं�कदम�उठा�सकती�है।�

इस�तरह�यह��प��है��क�सं�वधान , संसद�को�नए�रा�य��का��नमा�ण�करने�या�रा�य��क��सहम�त�के ��बना�इनके ��े��� , सीमा��या�नाम�म�


प�रवत�न�के ��लए�अ�धकृ त�करता�है।��सरे�श�द��म�� , संसद�अपनी�इ�छा�के �अनुसार�भारत�के �राजनी�तक�मान�च��को�रेखां�कत�कर�सकती�है।
इस�लए�, सं�वधान��ारा��कसी�भी�रा�य�क���े�ीय�अखंडता�या��नरंतर�अ��त�व�क��गारंट��नह��है।�इस�लए� , भारत�को�' �वनाशकारी�रा�य��का
एक�अ�वनाशी�संघ� ' कहा�जाता�है।�क� ��सरकार�रा�य��को�न��कर�सकती�है� , जब�क�रा�य�सरकार�संघ�को�न��नह��कर�सकती।�संयु��रा�य
अमे�रका�म�� , �सरी�तरफ�, सं�वधान��ारा�रा�य�क���े�ीय�अखंडता�या��नरंतर�अ��त�व�क��गारंट��द��जाती�है।�अमे�रक��संघीय�सरकार
संबं�धत�रा�य��क��सहम�त�के ��बना�नए�रा�य��को�नह��बना�सकती�है�या�मौजूदा�रा�य��क��सीमा��को�बदल�सकती�है।�यही�कारण�है��क
अमरीका�को�' अ�वनाशी�रा�य��का�एक�अ�वनाशी�संघ� ' कहा�जाता�है।�

इसके �अलावा�, सं�वधान� (अनु�छे द� 4) कहा�गया�है��क�नये�रा�य��के ��वेश�या��थापना� (अनु�छे द� 2 के �तहत); नए�रा�य��का��नमा�ण�और


मौजूदा�रा�य��के ��े��� , सीमा��या�नाम��के �प�रवत�न� (अनु�छे द� 3 के �तहत) अनु�छे द� 368 के �तहत�सं�वधान�संशोधन�नह��माना�जाता�है।
इसका�मतलब�है��क�ऐसे�कानून�साधारण�ब�मत�से�और�साधारण��वधायी����या�के ��ारा�पा�रत��कए�जा�सकते�ह�।�

QUESTION 13.
�न�न�ल�खत�म��से��कस�आयोग�ने�रा�य��के �पुनग�ठन�के ��लए�एक�आधार�के ��प�म�� ' भाषा�' को��वीकार��कया�?

IASbaba
Web: http://ilp.iasbaba.com/ Score:
Email: ilp@iasbaba.com 4.00 / 200
Page 95
Exam Title : Test 21-Revision: Polity & ...
Email
Contact :

a) धर�आयोग�
b) जेवीपी�आयोग�
c) फजल�अली�आयोग�
d) उपरो��म��से�कोई�नह��
Correct Answer: C
Your Answer:
Explanation

Solution (c)

फजल�अली�आयोग�

आं���दे श�के ��नमा�ण�ने�भाषाई�आधार�पर�रा�य��के �गठन�के ��लए�अ�य��े���क��मांग�को�तेज�कर��दया।�इसने�सरकार�को�इसके �स�ब�ध�म�


पुन : जांच�के ��लए�फजल�अली�क��अ�य�ता�म��तीन�सद�यीय�रा�य�पुनग�ठन�आयोग� (�दसंबर� 1953 म� ) �नयु��करने�के ��लए�मजबूर��कया।
इसके �दो�अ�य�सद�य�के. एम. प�ण�कर�और�एच. एन. कु �ज��थे।�उसने�अपनी��रपोट� ��सतंबर� 1955 म����तुत�क��और�इस�बात�को�मोटे
तौर�पर��वीकार��कया�गया��क�रा�य��के �पुनग�ठन�म��भाषा�को�मु�य�आधार�बनाया�जाना�चा�हए।�ले�कन� , इसने� ' एक�रा�य-एक�भाषा’ के
�स�ांत�को�खा�रज�कर��दया।�इसका��वचार�था��क�दे श�क���कसी�भी�राजनी�तक�इकाइय��के �पुन�न�मा�ण�म��भारत�क��एकता�को��ाथ�मकता
�दया�जाना�चा�हए।�इसम��चार��मुख�कारक��क��पहचान�क��गई�है��ज�ह��रा�य��के �पुनग�ठन�क���कसी�भी�योजना�म���यान�म��रखा�जा�सकता�है :

a) दे श�क��एकता�और�सुर�ा�का�संर�ण�और�अनुर�ण।�

b) भाषाई�और�सां�कृ �तक�एक�पता�

c) �व�ीय�, आ�थ�क�और��शास�नक��वचार�

d) ��येक�रा�य�म��और�साथ�ही�पूरे�दे श�म��लोग��के �क�याण�क��योजना�और�इसका�सवध�न।�

आयोग�ने�सलाह��दया��क�मूल�सं�वधान�के �तहत�रा�य��के �चार�आयामी�वग�करण�का�उ�मूलन��कया�जाए�और� 16 रा�य��और�3 क� ��य


�शा�सत��दे श��के ��नमा�ण��कया�जाए।�भारत�सरकार�ने�कु छ�प�रवत�न��के �साथ�इन��सफा�रश��को��वीकार��कया।�रा�य�पुनग�ठन�अ�ध�नयम
(1956) और�7व��सं�वधान�संशोधन�अ�ध�नयम� (1956) के ��ारा�, पाट� -ए�और�पाट� -बी�रा�य��के �बीच�का�अंतर�समा�त�कर��दया�गया�और
पाट� -सी�रा�य��को�समा�त�कर��दया�गया।�इनम��से�कु छ�को�आस��रा�य��म���वलय�कर��दया�गया�और�कु छ�अ�य�को�क� ��शा�सत��दे श��के ��प
म��पुनः��था�पत��कया�गया।�नतीजतन� , 1 नवंबर� , 1956 को�14 रा�य��और�6 क� ��शा�सत��दे श��का��नमा�ण��आ।�

�वचार�कर��

· रा�य��के �पुनग�ठन�

QUESTION 14.
भारत�म��नाग�रकता�के �स�दभ��म���न�न�ल�खत�कथन��पर��वचार�क��जए :

1. �वदे शी���त�न�धय��के �भारत�म��पैदा��ए�ब�चे�भारतीय�नाग�रकता��ा�त�करते�ह�।�

2. 2018 म��भारत�के �बाहर�पैदा��आ�एक�ब�चा , वंश�के �आधार�पर��वत: भारत�क��नाग�रकता��ा�त�कर�लेगा ; य�द�उसके �माता-�पता�दोन�


ही�भारतीय�हो।�

उपरो��कथन��म��से�कौन�सा /से�सही�है� ?

a) के वल�1
b) के वल�2
c) दोन��1 और�2
d) न�तो�1 और�न�ही�2
Correct Answer: D
Your Answer:

IASbaba
Web: http://ilp.iasbaba.com/ Score:
Email: ilp@iasbaba.com 4.00 / 200
Page 96
Exam Title : Test 21-Revision: Polity & ...
Email
Contact :

Explanation

Solution (d)

भारत�म��नाग�रकता�

नाग�रकता�अ�ध�नयम�1955; नाग�रकता��ा�त�करने�के �पांच�शत�� , जैसे�ज�म� , वंश� , पंजीकरण� , �ाकृ �तककरण�और��े��का�समावेशन�को


�नधा��रत�करता�है :

ज�म�से : भारत�म�� 26 जनवरी�1950 को�या�उसके �बाद�पर�तु� 1 जुलाई� 1947 से�पूव��ज�मा�����अपने�माता -�पता�के �ज�म�क��रा�ीयता
के �बावजूद�भारत�का�नाग�रक�होगा।�

भारत�म�� 1 जुलाई�को�या�उसके �बाद�ज�मा�कोई�����भारत�का�नाग�रक�माना�जाएगा�, य�द�उसके �ज�म�के �समय�उसके �माता-�पता�म��से


कोई�एक�ज�म�के �समय�भारत�का�नाग�रक�हो।�

इसके �अलावा�, य�द��कसी�����का�ज�म�3 �दसंबर� 2004 को�या�उसके �बाद�भारत�म���आ�हो�तो�वह�उसी�दशा�म��भारत�का�नाग�रक�माना


जाएगा�, य�द�उनके �माता-�पता�दोन��उसके �ज�म�के �समय�भारत�के �नाग�रक�ह��या�माता�या��पता�म��से�एक�उस�समय�भारत�का�नाग�रक�हो
और��सरा�अवैध��वासी�न�हो।�

भारत�म��पद�थ��वदे शी�राजन�यक�एवं�श�ु�दे श�के �ब�च��को�भारतीय�नाग�रकता��ा�त�करने�का�अ�धकार�नह��नह��है।�

वंश�के �आधार�पर: कोई������जसका�ज�म�26 जनवरी�1950 या�उसके �बाद�पर�तु� 10 �दसंबर� 1992 के �पहले�भारत�के �बाहर��आ�हो,
वह�वंश�के �आधार�पर�भारत�का�नाग�रक�बन�सकता�है� , य�द�उनके �ज�म�के �समय�उसका��पता�भारत�का�नाग�रक�हो।�

य�द�10 �दसंबर� 1992 को�या�उसके �बाद�य�द��कसी�����का�ज�म�दे श�के �बाहर��आ�हो�तो�व�तभी�भारत�का�नाग�रक�बन�सकता�है�य�द


उसके �ज�म�के �समय�उसके �माता-�पता�म��से�कोई�एक�भारत�का�नाग�रक�हो।�

3 �दसंबर� 2004 के �बाद�, भारत�से�बाहर�ज�मा�कोई�����वंश�के �आधार�पर�भारत�का�नाग�रक�नह��होगा�, य�द�उसके �ज�म�के �एक�वष��के


भीतर�भारतीय�वा�ण�य��तावास�म��या�क� ��सरकार�क��सहम�त�से�उ��अव�ध�के �बाद�पंजीकरण�न��आ�हो।�इस��कार�के �ब�चे�का�भारतीय
वा�ण�य��तावास�म��पंजीकरण�कराते�समय�आवेदन�प��म��माता -�पता�को�इस�आशय�का�शपथ�प��दे ना�होगा��क�उनके �ब�चे�के �पास��कसी
अ�य�दे श�का�पासपोट� �नह��है।�

�वचार�कर��

· नाग�रकता��ा�त�करने�के �अ�य�तरीके �

QUESTION 15.
�न�न�ल�खत�म��से�कौन�सा /से�कथन�मूल�अ�धकार��के �बारे�म��सही�ह�� ?

1. ये�च�र��म��नकारा�मक�होते�ह�� , अथा�त�ये��ा�धकारी�पर���तबंध�लगाते�ह�।�

2. य�द�इन�अ�धकार��का�उ�लंघन�होता�है�तो�पी�ड़त�सीधे�उ�चतम��यायालय�जा�सकता�ह�।�

3. मूल�अ�धकार��को�संसद��ारा�संशो�धत�नह���कया�जा�सकता�है।�

नीचे�कू ट�से�चयन�क��जए :

a) 1 और�2
b) 2 और�3
c) 1 और�3
d) उपरो��सभी�
Correct Answer: D
Your Answer:
Explanation

IASbaba
Web: http://ilp.iasbaba.com/ Score:
Email: ilp@iasbaba.com 4.00 / 200
Page 97
Exam Title : Test 21-Revision: Polity & ...
Email
Contact :

Solution (d)

मौ�लक�अ�धकार��क���वशेषताएं�

मूल�अ�धकार��को�सं�वधान�म���न�न�ल�खत��वशेषता��के �साथ�सु�न��त��कया�गया�ह� :

1. उनम��से�कु छ�के वल�नाग�रक��के ��लए�उपल�ध�ह�� , जब�क�अ�य�सभी����य��के ��लए�उपल�ध�ह��चाहे�नाग�रक�� , �वदे �शय��या�कानूनी


���य��जैसे��क��नगम�या�कं प�नयाँ।�

2. ये�असी�मत�नह��है�ले�कन�वादयो�य�होते�ह��रा�य�उन�पर�यु��यु����तबंध�लगा�सकता�है।�हालां�क� , इस�तरह�के ���तबंध�उ�चत�ह��या�नह��


, इसका��नण�य��यायालय��के ��ारा��कया�जाता�है।�इस��कार� , ये����गत�अ�धकार��एवं�पुरे�समाज�के �बीच�संतुलन�कायम�करते�ह�।�

3. उनम��से�अ�धकतर�रा�य�के �मनमाने�रवैये�के ��खलाफ�उपल�ध�ह�� , जैसे -रा�य�के ��खलाफ�कोई�काय�वाही�और����य��के ��खलाफ�कु छ


काय�वाही।�जब�अ�धकार�रा�य�कार�वाई�के ��खलाफ�ह��और��कसी�����के ��ारा�इसका�उ�लंघन��कया�जा�रहा�हो�तो�वे�कोई�संवैधा�नक�उपाय
नह��ह��ब��क�के वल�सामा�य�कानूनी�उपाय�ह�।�

4. उनम��से�कु छ�च�र��म��नकारा�मक�ह�� , अथा�त� , रा�य�के ��ा�धकार�को�सी�मत�करने�से�स�बं�धत�ह�� , जब�क�अ�य��वभाव�सकारा�मक�ह�� ,


जैसे����य��के ��लए�कु छ��वशेष�सु�वधा��का��ावधान।�

5. वे��यायो�चत�ह�� , ये����य��को�अदालत�जाने�क��अनुम�त�दे ते�है� , जब�भी�इनका�उ�लंघन�होता�है।�

6. उ�ह��सव��च��यायालय��ारा�सुर�ा�और�गारंट��द��जाती�है।�इस�लए� , पी�ड़त�����सीधे�उ�चतम��यायालय�जा�सकता�ह�।यह�आव�यक
नह��है��क�के वल�उ�च��यायालय��के �फै सले�के ��खलाफ�अपील�के �ज�रए�जाया�जाये।�

7. ये��थायी�नह��ह�।�संसद�इनमे�कटौती�या��नर�त�कर�सकती�है� , ले�कन�के वल�एक�संवैधा�नक�संशोधन�अ�ध�नयम�के ��ारा�और�एक�साधारण


अ�ध�नयम�के ��ारा�नह�।�इसके �अलावा�, यह�सं�वधान�के �' बु�नयाद��ढांचे� ' को��भा�वत��कए��बना�भी��कया�जा�सकता�है।�

8. उ�ह��रा�ीय�आपात�के �संचालन�के �दौरान��नलं�बत��कया�जा�सकता�है� , (अनु�छे द 20 और�21 म��व�ण�त�अ�धकार��को�छोड़कर )।��अनु�छे द


19 म��गारंट�कृ त�छह�अ�धकार��को�के वल�तभी��नलं�बत��कया�जा�सकता�है�जब�यु��या�बाहरी�आ�मण�के �आधार�पर�आपातकाल�घो�षत
�कया�जाता�है� (यानी�, बा��आपातकाल) और�सश����व�ोह�(यानी�, आंत�रक�आपातकाल ) के �आधार�पर�नह�।�

9. अनु�छे द� 31क(संप���के �अ�ध�हण�पर�कानून��क��र�ा�आ�द ) , अनु�छे द� 31ख�(9व��अनुसूची�म��शा�मल�कु छ�कृ �य�) और�अनु�छे द


31ग�(कु छ��न��त��स�ांत��पर��भाव�डालने�वाले�कानून��क���ावृ�त ) क��मा�यता��ारा�सी�मत�है।�

10. सश���बल��, अध�सै�नक�बल�� , पु�लस�बल�� , खु�फया�एज��सय��और�ऐसे�ही�सेवा��से�स�बं�धत�सेवा��के ���या�वयन�पर�संसद


��तबंध�आरो�पत�कर�सकती�है (अनु�छे द� 33)।�

11. �कसी�भी�ऐसे��े��म��इसका���या�वयन�रोका�जा�सकता�है�जहाँ�माश�ल�लॉ�लागू�है।�माश�ल�लॉ�का�अथ�� ' सै�य�शासन� ' से�है�जो


असामा�य�प�र��थ�तय��म��लगाया�जाता�है� (अनु�छे द� 34)।�यह�रा�ीय�आपातकाल�लागू�करने�से��भ��है।�

12. इनम��से��यादातर��वयं��व�त�त�ह��जब�क�उनम��से�कु छ�को�कानून�के �आधार�पर�लागू��कया�जा�सकता�है।�ऐसा�कानून�दे श�क��एकता�के


�लए�के वल�संसद��ारा�ही�बनाया�जा�सकता�है� , रा�य��वधा�यका���ारा�नह��, ता�क�पूरे�दे श�म��एक�पता�बनी�रहे� (अनु�छे द� 35)।�

QUESTION 16.
�न�न�ल�खत�म��से�कौन�सा�मूल�अ�धकार�के वल�भारतीय�नाग�रक��के ��लए�उपल�ध�ह�� ?

1. लोक��नयोजन�म��अवसर�क��समानता�

2. धम�� , जा�त�, मूल�वंश� , �ल�ग�या�ज�म��थान�के �आधार�पर�कोई�भेदभाव�नह��

3. भाषा�और��ल�प�का�संर�ण�

4. शै��क�सं�थान��क���थापना�और��शासन�का�अ�धकार�

नीचे�कू ट�से�चयन�क��जए� ?

IASbaba
Web: http://ilp.iasbaba.com/ Score:
Email: ilp@iasbaba.com 4.00 / 200
Page 98
Exam Title : Test 21-Revision: Polity & ...
Email
Contact :

a) 1,2 और�3
b) 2,3 और�4
c) 1,3 और�4
d) उपरो��सभी�
Correct Answer: D
Your Answer:
Explanation

Solution (d)

मूला�धकार�जो�के वल�नाग�रक��के ��लए�ही�उपल�ध�है� , �वदे �शय��के ��लए�नह��

· धम�� , मूलवंश� , जा�त�, �ल�ग�या�ज�म��थान�के �आधार�पर��वभेद�का���तषेध (अनु�छे द� 15)।�

· लोक��नयोजन�के �मामले�म��अवसर�क��समानता� (अनु�छे द� 16)

· �वतं�ता�के �संबंध�म��छह�अ�धकार��का�संर�ण : ( i) वाक् �एवं�अ�भ���� , (ii) स�मेलन� , (iii) संघ� , (iv) संचरण� , (v) �नवास�, और�(
vi) �वसाय�एवं�वृ�� (अनु�छे द� 19)।�

· अ�पसं�यक��क��भाषा� , �ल�प�और�सं�कृ �त�का�संर�ण� (अनु�छे द� 29)।�

· शै��क�सं�थान��क���थापना�और��शासन�का�अ�पसं�यक��का�अ�धकार� (अनु�छे द� 30)।�

QUESTION 17.
सं�वधान�के �अनु�छे द� 17 ने� ' अ�पृ�यता� ' का�उ�मूलन�कर��दया�है।�अनु�छे द� 17 के �स�दभ��म���न�न�ल�खत�कथन��म��से�कौन�सा /से�सही�है�

1. यह��प���प�से�अ�पृ�यता�को�प�रभा�षत�करता�है।�

2. इसम��कु छ����य��का�सामा�जक�ब�ह�कार�या�धा�म�क�अनु�ान��से�उनका�ब�ह�करण�शा�मल�नह��है।�

नीचे�कू ट�से�चयन�क��जए :

a) के वल�1
b) के वल�2
c) दोन��1 और�2
d) न�तो�1 और�न�ही�2
Correct Answer: B
Your Answer:
Explanation

Solution (b)

अनु�छे द� 17 ' अ�पृ�यता� ' का�उ�मूलन�करता�है�और��कसी�भी��प�म��इसके �आचरण�को��न�ष��करता�है।�अ�पृ�यता�से�उ�प��होने�वाली


�कसी�भी��नय��यता�को�लागू�करना��व�ध�के �अनुसार�दं डनीय�होगा।�

1976 म�� , अ�पृ�यता� (अपराध) अ�ध�नयम�, 1955 को��ापक��प�से�संशो�धत��कया�गया�है�और�इसे�नाग�रक�अ�धकार�संर�ण


अ�ध�नयम�, 1955 के ��प�म��ना�मत��कया�गया�है�ता�क�इसके �दायरे�को�बड़ा��कया�जा�सके �और�दं ड��ावधान��को�अ�धक�स�त�बनाया�जा
सके ।�

' अ�पृ�यता� ' श�द�को�न�तो�सं�वधान�म��और�न�ही�अ�ध�नयम�म��प�रभा�षत�नह���कया�गया�है।�हालां�क� , मैसूर�उ�च��यायालय�ने�अनु�छे द� 17


के �मामले�म��कहा�है��क�अपनी�शा��दक�या��ाकर�णक�अथ��से�परे�इसका��योग�ऐ�तहा�सक�है� , ले�कन�यह��व�श��मामल��म��कु छ�वग��के
लोग��को�उनके �ज�म�एवं�उनके �जा�तय��के �आधार�पर��सामा�जक��नय��यता�को�दशा�ता�है।�इस�लए� , इसम��कु छ����य��के �सामा�जक
ब�ह�कार�या�धा�म�क�सेवा��से�अपवज�न�शा�मल�नह��है।�

IASbaba
Web: http://ilp.iasbaba.com/ Score:
Email: ilp@iasbaba.com 4.00 / 200
Page 99
Exam Title : Test 21-Revision: Polity & ...
Email
Contact :

अ�ध�नयम��न�न�ल�खत�कृ �य��को�अपराध��के ��प�म��घो�षत�करता�है :

1. �कसी�भी�����को�साव�ज�नक�पूजा��थल�म���वेश�से�रोकना�या�कह��भी�पूजा�करने�से�रोकना� ;

2. परंपरागत� , धा�म�क� , दाश��नक�या�अ�य�आधार�पर�अ�पृ�यता�को��यायो�चत�ठहराना� ;

3. �कसी�भी��कान�, होटल�या�साव�ज�नक�मनोरंजन�के ��थान��म���वेश�से�इनकार�करना� ;

4. अ�पृ�यता�के �आधार�पर�अनुसू�चत�जा�त�के ��कसी�����का�अपमान�करना�;

5. साव�ज�नक�लाभ�के ��लए��था�पत�अ�पताल��, शै��क�सं�थान��या�हॉ�टल��म�����य��के ��वेश�से�रोकना� ;

6. ��य��या�अ��य���प�से�अ�पृ�यता�का��चार�करना� ; तथा�

7. �कसी�भी�����को�माल�बेचने�या�सेवा��को���तुत�करने�से�रोकना।�

सु�ीम�कोट� �ने�अनु�छे द� 17 के �तहत�कहा�है��क�यह�अ�धकार��नजी�����और�रा�य�का�संवैधा�नक�दा�य�व�होगा��क�इस�अ�धकार�के �उ�लंघन


को�रोकने�के ��लए�ज�री�कदम�उठाय�।�

QUESTION 18.
भारतीय�सं�वधान�ने�नाग�रक��को�संग�ठत�होने�का�अ�धकार��दया�है।�इनम��से�कौन�से��कथन�सह��ह�� ?

1. ��येक�नाग�रक�को�शां�तपूव�क�और�ह�थयार��के ��बना�संग�ठत�होने�का�अ�धकार�है।�

2. नाग�रक��को�साव�ज�नक�बैठक��म��भाग�लेने� , �दश�न�करने�और�जुलुस��नकलने�का�अ�धकार�है।�

3. इसम��हड़ताल�करने�का�अ�धकार�भी�शा�मल�है।�

नीचे�कू ट�से�चयन�क��जए :

a) 1 और�2
b) 2 और�3
c) 1 और�3
d) उपरो��सभी�
Correct Answer: A
Your Answer:
Explanation

Solution (a)

शां�तपूव�क�स�मलेन�क���वतं�ता�

�कसी�भी�नाग�रक�को��बना�ह�थयार�के �शां�तपूव�क�संग�ठत�होने�का�अ�धकार�है।�इसम��साव�ज�नक�बैठक�� , �दश�न��का�आयोजन�एवं�जुलूस�म�


भाग�लेने�का�अ�धकार�शा�मल�है।�इस��वतं�ता�का�उपयोग�के वल�साव�ज�नक�भू�म�पर��बना�ह�थयार�के ��कया�जा�सकता�है।�यह��ावधान��ह�सा
, अ�व�था�, साव�ज�नक�शां�त�का�उ�लंघन�या�ह�थयार��से�जुड़ा��आ�है� , का�संर�ण�नह��करता�है।�इस�अ�धकार�म��हड़ताल�करने�का
अ�धकार�शा�मल�नह��है।�

रा�य�दो�आधार��पर�स�मलेन�के �अ�धकार�के ��योग�पर�उ�चत���तबंध�लगा�सकता�है� , - पहला. भारत�क��एकता�और�अखंडता�तथा��सरा


लोक��व�था��जसम��संबं�धत��े��म��यातायात�के �रखरखाव�भी�शा�मल�है।�

�वचार�कर��

· ज�लयांवाला�बाग�

IASbaba
Web: http://ilp.iasbaba.com/ Score:
Email: ilp@iasbaba.com 4.00 / 200
Page 100
Exam Title : Test 21-Revision: Polity & ...
Email
Contact :

QUESTION 19.
�न�न�ल�खत�कथन��पर��वचार�क��जए:

1. �कसी�����को�एक�ही�अपराध�के ��लए�एक�बार�से�अ�धक�अ�भयो�जत�और�दं �डत�नह���कया�जाएगा।�

2. पूव��ापी�दं ड�से�रोकथाम�के वल�आपरा�धक�कृ �य�के �मामले�म��है� , �स�वल�मामल��म��नह�।�

3. कोई��व-अ�भशंसन�नह� , के वल�आपरा�धक�काय�वाही�तक��व�तृत�है�तथा�नाग�रक�काय�वाही�तक�नह�।�

उपरो��कथन��म��से�कौन�सा /से�सही�है� ?

a) 1 और�2
b) 2 और�3
c) 1 और�3
d) उपरो��सभी�
Correct Answer: D
Your Answer:
Explanation

Solution (d)

अनु�छे द� 20

अनु�छे द� 20 �कसी�भी�अ�भयु��या�दोषी�क़रार�����को�मनमानी�और�अ�य�धक�दं ड�के ��खलाफ�सुर�ा��दान�करता�है� , चाहे�वह�नाग�रक


या��वदे शी�या��कसी�कं पनी�या��नगम�का�कानूनी�����हो।�इस�स�ब�ध�म��तीन��ावधान�ह� :

1. कोई�पूव� -पद-�भाव�कानून�नह� : कोई�����( i) �कसी�भी�अपराध�के ��लए�तब�तक�दोषी�नह��ठहराया�जाएगा�, जब�तक��क�उसने�ऐसा


कोई�काय��करने�के �समय�जो�अपराध�के ��प�म��आरो�पत�है , �कसी��वृ���व�ध�का�अ�त�मण�नह���कया�है ; या�( ii) उससे�अ�धक�श���का
भागी�नह��होगा, जो�उस�अपराध�के ��लए�जाने�के �समय�म���वृ���व�ध�के �अधीन�अ�धरो�पत�क��जा�सकती�थी।�

2. दोहरी���त�नह�: �कसी�����को�एक�ही�अपराध�के ��लए�एक�बार�से�अ�धक�अ�भयो�जत�और�दं �डत�नह���कया�जाएगा।�

3. �व- अ�भशंसन�नह� : �कसी�भी�अपराध�के ��लए�अ�भयु���कसी�����को��वयं�अपने��व���सा�ी�बनने�के ��लए�मजबूर�नह���कया


जाएगा।�

एक�पूव��पद��भाव�कानून�वह�है�जो�दं ड�को�पूव��ापी� (पूव��ापी ) , जो��क�पहले�से��कए�गए�कृ �य��पर�या�जो�ऐसे�कृ �य��के ��लए�दं ड�को


बढ़ाता�है� , पर�लगाया�जाता�है।�अनु�छे द� 20 के �पहले��ावधान�के �अंतग�त�इस�तरह�के ���या�वयन�पर�रोक�है।�हालां�क� , यह�सीमा�के वल
आपरा�धक�कानून��पर�ही�लागू�होती�है� , न��क�सामा�य��स�वल�अ�धकार�या�कर�कानून��म�।��सरे�श�द��म�� , एक�जनउ�रदा�य�व�या�एक�कर
को�पूव��ापी��प�से�लगाया�जा�सकता�है।�इसके �अलावा�, इस�तरह�क���व�था�अपराध�दोष�या�सजा�सुनाए�जाने�के �मौके �पर�आपरा�धक
कानून��पर��भावी�रहती�है।�अंत�म�� , इस��ावधान�के �तहत�सुर�ा��व�था�के �तहत�बचाव�के �मामले�म��एक�����क��सुर�ा�क��मांग�के �आधार
पर�नह���कया�जा�सकता�है।�

दोहरी���त�के ��खलाफ�सुर�ा�के वल�एक��यायालय�या��या�यक��याया�धकरण�म��ही�उठाया�जा�सकता�है।��सरे�श�द��म�� , यह��वभागीय�या


�शास�नक�सुनवाई�म��लागू�नह��हो�सकता�है��य��क�ये��या�यक��कृ �त�के �नह��ह�।�

�व-अ�भशंसन�के �स�ब�ध�म��मौ�खक�और�द�तावेजी�सा�य�दोन��से�संर�ण��ा�त�है।�

हालां�क� , यह��व�ता�रत�नह���कया�जा�सकता- ( i) भौ�तक�व�तु��के �आव�यक�उ�पादन��पर�, (ii) अंगूठे�के ��नशान�, ह�ता�र�, एवं�र�


जांच�क��अ�नवाय�ता�पर�और� ( iii) �नकाय�के �अ�नवाय���दश�न�पर।�इसके �अलावा�, इसका��व�तार�के वल�आपरा�धक�काय�वाही�तक�फै ली��ई
है� , न��क��स�वल�काय�वाही�या�ऐसे�काय�वाही�जो��क�आपरा�धक��कृ �त�के �नह��ह�।�

�वचार�कर��

· �नवारक��हरासत�

IASbaba
Web: http://ilp.iasbaba.com/ Score:
Email: ilp@iasbaba.com 4.00 / 200
Page 101
Exam Title : Test 21-Revision: Polity & ...
Email
Contact :

QUESTION 20.
�न�न�ल�खत�अ�धकार��म��से��कनको�अनु�छे द� 21 के �दायरे�के �तहत�लाया�गया�है� ?

1. �नजता�का�अ�धकार�

2. मु�त��व�धक�सहायता�पाने�का�अ�धकार�

3. �वदे श�जाने�का�अ�धकार�

4. सूचना�का�अ�धकार�

नीचे�कू ट�से�चयन�क��जए� ?

a) 1 और�4
b) 2,3 और�4
c) 1,3 और�4
d) उपरो��सभी�
Correct Answer: D
Your Answer:
Explanation

Solution (d)

अनु�छे द� 21

सु�ीम�कोट� �ने�मेनका�मामले�म��अपने�फै सले�को��बारा��था�पत��कया�है।�इसम��अनु�छे द� 21 के �भाग�के ��प�म���न�न�ल�खत�अ�धकार��क�


घोषणा�क��है :

a) मानवीय���त�ा�के �साथ�जीने�का�अ�धकार�

b) �व�छ�पया�वरण - ��षण�र�हत�जल�एवं�वायु�म��जीने�का�अ�धकार�एवं�हा�नकारक�उ�ोग��के ��व���सुर�ा।�

c) आजी�वका�का�अ�धकार�

d) �नजता�का�अ�धकार�

e) आ�य�का�अ�धकार�

f) �वा��य�का�अ�धकार�

g) 14 वष��तक�क��उ��के ��लए�मु�त��श�ा�का�अ�धकार।�

h) �नशु�क�कानूनी�सहायता�का�अ�धकार�

i) अके ले�कारावास�म��बंद�होने�के ��व���अ�धकार�

j) �व�रत�सुनवाई�का�अ�धकार�

k) हथकड़ी�लगाने�के ��खलाफ�अ�धकार�

l) अमानवीय��वहार�के ��खलाफ�अ�धकार�

m) �वलं�बत�फांसी�के ��खलाफ�अ�धकार�

n) �वदे श�या�ा�का�अ�धकार�

IASbaba
Web: http://ilp.iasbaba.com/ Score:
Email: ilp@iasbaba.com 4.00 / 200
Page 102
Exam Title : Test 21-Revision: Polity & ...
Email
Contact :

o) बंधुआ��म�के ��खलाफ�अ�धकार।�

p) �हरासत�के �उ�पीड़न�के ��खलाफ�अ�धकार�

q) आपातकालीन��च�क�सा�सहायता�का�अ�धकार�

r) सरकारी�अ�पताल�म��समय�पर�उ�चत��च�क�सा�का�अ�धकार�

s) रा�य�के �बाहर�न�जाने�का�अ�धकार�

t) �न�प��सुनवाई�का�अ�धकार�

u) कै द��के ��लए�जीवन�क��आव�यकता��का�अ�धकार�

v) म�हला��के �साथ�आदर�और�स�मानपूव�क��वहार�करने�का�अ�धकार�

w) साव�ज�नक�फांसी�के ��खलाफ�अ�धकार�

x) सुनवाई�का�अ�धकार�

y) सूचना�का�अ�धकार�

z) ��त�ा�का�अ�धकार�

QUESTION 21.
�नवारक��नरोध�के �स�दभ��म���न�न�ल�खत�म��से�कौन�सा�कथन�सही�ह�� ?

1. �नरोध�तीन�महीने�से�अ�धक�नह��हो�सकता , जब�तक��क�कोई�सलाहकार�बोड���नरोध�के �पया��त�कारण�क���रपोट� �न�करे।�

2. नजरबंद��कये�गए�����को��गर�तारी�का�कारण�बताया�जाना�आव�यक�नह��है।�

3. नजरबंद��कये�गए�����को��नरोध�आदे श�के ��खलाफ�अपना�प��रखने�का�एक�अवसर��दान��कया�जाना�चा�हए।�

नीचे�कू ट�से�चयन�क��जए :

a) 1 और�2
b) 2 और�3
c) 1 और�3
d) उपरो��सभी�
Correct Answer: C
Your Answer:
Explanation

Solution (c)

अनु�छे द� 22 के �दो�भाग�ह� - पहला�भाग�साधारण�कानूनी�मामल��से�संबं�धत�है�जब�क��सरा�भाग��नवारक��नरोध�कानून�के �मामल��से�संबं�धत


है।�

अनु�छे द� 22 का��सरा�भाग�एक��नवारक��नरोध�कानून�के �तहत��गर�तार�या��हरासत�म���लया�गया����य��को�सुर�ा��दान�करता�है।�यह


सुर�ा�दोन��नाग�रक��के �साथ�ही��वदे �शय��के ��लए�भी�उपल�ध�है�और�इसम���न�न�ल�खत�शा�मल�ह� :

1. ����क���हरासत�तीन�महीने�से��यादा�नह��बढ़ाई�जा�सकती�है� , जब�तक��क�सलाहकार�बोड��इस�बारे�म��पया��त�कारण�नह��बताता�है।
बोड��उ�च��यायालय�के ��यायाधीश��से��मलकर�बनेगा।�

2. स�बं�धत�����को��गर�तारी�का�आधार�बताया�जाना�चा�हए ; हालां�क� , साव�ज�नक��हत��के ��व���इसे�बताया�जाना�आव�यक�नह��है।�

IASbaba
Web: http://ilp.iasbaba.com/ Score:
Email: ilp@iasbaba.com 4.00 / 200
Page 103
Exam Title : Test 21-Revision: Polity & ...
Email
Contact :

3. �नरोध��कये�गए�����को��नरोध�आदे श�के ��खलाफ�अपना�प��रखने�का�एक�अवसर��दान��कया�जाना�चा�हए।�

अनु�छे द� 22 संसद�को�भी�यह�बताने�के ��लए�अ�धकृ त�करता�है��क :

I. एक�सलाहकार�बोड��क��राय��ा�त��कए��बना , �कसी�भी�����को��कन�प�र��थ�तय��के �अधीन�और��कस�वग��या�वग��के �मामल��म���नवारक


�नरोध�कानून�के �तहत�तीन�माह�से�अ�धक�समय�तक��न����कया�जा�सकता�है� ;

II. और��कस�वग��या�वग��के �मामल��म���कतनी�अ�धकतम�अव�ध�के ��लए��कसी�����को��नवारक��नरोध�कानून�के �तहत��हरासत�म���लया�जा


सकता�है� ; तथा�

III. जांच�म��सलाहकार�बोड���ारा�अनुसरण�क��जाने�वाली����या�

44व��संशोधन�अ�ध�नयम� 1978 के ��ारा��नरोध�क��अव�ध�को��बना�सलाहकार�बोड��क��राय�के �तीन�से�दो�माह�कर��दया�गया�है।�हालां�क� , य


ह��ावधान�अब�तक�लागू�नह���आ�है� , इस�लए�तीन�महीने�क��मूल�अव�ध�अभी�भी�जारी�है।�

QUESTION 22.
�न�न�ल�खत�शै��णक�सं�थान��म��से��कसम��धा�म�क��श�ा�पूण�तया��न�ष��है� ?

1. सं�थान��जनका�पूरी�तरह�से�रखरखाव�रा�य�के ��ारा��कया�जाता�है�

2. सं�थान�जो�रा�य��ारा��शा�सत��क�तु���ट��ारा��था�पत�है�

3. रा�य��ारा�मा�यता��ा�त�सं�थान�

4. रा�य�से�सहायता��ा�त�सं�थान�

नीचे�कू ट�से�चयन�क��जए :

a) के वल�1
b) 2,3 और�4
c) 1,2 और�4
d) उपरो��सभी�
Correct Answer: A
Your Answer:
Explanation

Solution (a)

अनु . 28 चार��कार�के �शै��क�सं�थान��म���वभेद�करता�है :

a) ऐसे�सं�थान��जनका�पूरी�तरह�से�रखरखाव�रा�य�करता�है�

b) ऐसे�सं�थान , �जनका��शासन�रा�य�करता�है�ले�कन�उनक���थापना��कसी��यास�या���ट�के �तहत�हो।�

c) रा�य��ारा�मा�यता��ा�त�सं�थाएं�

d) रा�य�से�सहायता��ा�त�सं�थान�

�ावधान�(a) धा�म�क��श�ा�पूरी�तरह�से��न�ष��है�जब�क� (b) म�� , धा�म�क��श�ा�क��अनुम�त�है।� (c) और�(d) म�� , धा�म�क��श�ा�को


�वै��छक�आधार�पर�अनुम�त�है।�

QUESTION 23.
�न�न�ल�खत�म��से�कौन�कौन�से�कथन�उ�चतम��यायालय�और�उ�च��यायालय�के ��रट��े�ा�धकार�के �म�य�उ�चत��प�से�भेद�करते�है� ?

IASbaba
Web: http://ilp.iasbaba.com/ Score:
Email: ilp@iasbaba.com 4.00 / 200
Page 104
Exam Title : Test 21-Revision: Polity & ...
Email
Contact :

1. उ�चतम��यायालय�के वल�मूल�अ�धकार�को�लागू�करने�के ��लए��रट�जारी�कर�सकता�है� , जब�क�उ�च��यायालय�अ�य��योजन��के ��लए�भी


�रट�जारी�कर�सकता�है।�

2. उ�चतम��यायालय�पुरे�दे श�म���रट�जारी�कर�सकता�है� , जब�क�उ�च��यायालय�के वल�अपने�रा�य�म���रट�जारी�कर�सकता�है।�

3. उ�चतम��यायालय�अपने��रट�का�अ�धकार��े��का��योग�करने�से�इंकार�नह��कर�सकता� , जब�क�उ�च��यायालय�इनकार�कर�सकता�है।�

नीचे�कू ट�से�चयन�क��जए :

a) 1 और�2
b) 2 और�3
c) 1 और�3
d) उपरो��सभी�
Correct Answer: D
Your Answer:
Explanation

Solution (d)

उ�चतम��यायालय�का��रट��े�ा�धकार��न�न�ल�खत�तीन�मामल��म� , उ�च��यायालय�से��भ��है :

1. उ�चतम��यायालय�के वल�मौ�लक�अ�धकार��को�लागू�करने�के ��लए��रट�जारी�कर�सकता�है� , जब�क�उ�च��यायालय�न�के वल�मूल�अ�धकार�


को�लागू�करने�के ��लए�ब��क�अ�य��कसी�भी�उ�े �य�के ��लए�भी��रट�जारी�कर�सकता�है।� ‘�कसी�अ�य�उ�े �य�के ��लए�' श�द�का�आ�भ�ाय��कसी
सामा�य�कानूनी�अ�धकार�के �संदभ��म��भी�है।�इस��कार� , उ�चतम��यायालय�का��रट��े�ा�धकार� , इस�संबंध�म�� , उ�च��यायालय�क��तुलना�म�
कम��व�तृत�है।�

2. उ�चतम��यायालय�भारत�के �पूरे�इलाके �म���कसी�����या�सरकार�के ��खलाफ��रट�जारी�कर�सकता�है� , जब�क�उ�च��यायालय�के वल


स�बं�धत�रा�य�के �����या�अपने��े��के �रा�य�को�या�मामला��सरे�रा�य�से�स�बं�धत�हो�तो�उसके ��खलाफ�ही�जारी�कर�सकता�है।�इस��कार� ,
�रट�जारी�करने�के �स�ब�ध�म��उ�चतम��यायालय�का��या�यक��े�ा�धकार�उ�च��यायालय�क��तुलना�म��अ�धक��व�तृत�है।�

3. अनु�छे द� 32 के �तहत, उपचार�अपने�आप�म��एक�मूल�अ�धकार�है�और�इस�लए� , सु�ीम�कोट� �अपने��रट�अ�धकार��े��का��योग�करने�से


इनकार�नह��कर�सकता�है।��सरी�ओर� , अनु�छे द� 226 के �तहत�उपचार��ववेकाधीन�है�और�इस�लए� , एक�उ�च��यायालय�अपनी��रट�स�ब�धी
�याय�े��के ���या�वयन�का�इ�तेमाल�करने�से�इनकार�कर�सकता�है।�अनु�छे द� 32 के वल�उ�चतम��यायालय�को�मूल�अ�धकार��के �स�ब�ध�म�
उ�च��यायालय�को��ा�त�अनु . 226 के �तहत�श����दान�नह��करता�है।�इस��कार�उ�चतम��यायालय�को�मूल�अ�धकार��का�र�क�और�गारंटर
बनाया�गया�है।�

QUESTION 24.
�न�न�ल�खत�म��से�कौन -सा/से�नी�त��नद� शक�त�व��कृ �त�म��समाजवाद��है /ह�� ?

1. �न:शु�क��व�धक�सहायता��दान�करना�

2. समान�काय��के ��लए�समान�वेतन�

3. �ाम�पंचायत�का�गठन�

4. कु ट�र�उ�ोग��को�बढ़ावा�दे ना�

नीचे�कू ट�से�चयन�क��जए� ?

a) 1 और�2
b) 3 और�4
c) 1,2 और�3
d) उपरो��सभी�

IASbaba
Web: http://ilp.iasbaba.com/ Score:
Email: ilp@iasbaba.com 4.00 / 200
Page 105
Exam Title : Test 21-Revision: Polity & ...
Email
Contact :

Correct Answer: A
Your Answer:
Explanation

Solution (a)

डीपीएसपी�म��समाजवाद���स�ांत�

ये��स�ांत�समाजवाद�क���वचारधारा�को�दशा�ते�ह�।�वे�एक�लोकतां��क�समाजवाद��रा�य�क���परेखा�तैयार�करते�ह�� , �जसका�ल�य�सामा�जक
और�आ�थ�क��याय��दान�करना�है� , और�क�याणकारी�रा�य�क���थापना�के ��लए�माग���श�त�करना�है।�

वे�रा�य�को��नद� श�दे ते�ह���क :

1. लोक�क�याण�क��अ�भवृ���के ��लए�सामा�जक�, आ�थ�क�और�राजनी�तक��याय�के ��ारा�सामा�जक��व�था�सु�न��त�करना�और�आय� , �


�त�ा�, सु�वधा��और�अवसर��क��असमानता��को�कम�करना (अनु�छे द� 38)।�

2. सुर��त�करना� (a) सभी�नाग�रक��के ��लए�आजी�वका�के �पया��त�साधन��ा�त�करने�का�अ�धकार� ; ( b) सामू�हक��हत�के ��लए�समुदाय�के


भौ�तक�संसाधन��का��यायसंगत��वतरण� ; ( c) धन�और�उ�पादन�के �साधन��का�संके ��ण�रोकना�; ( d) पु�ष��और�म�हला��के ��लए�समान
काय��के ��लए�समान�वेतन� ; ( e) ��मक��के ��वा��य�और�श���तथा�ब�च��के �सुकु मार�अव�था�का�संर�ण� ; और�(f) ब�च��के ��व�थ��वकास
के ��लए�अवसर�(अनु�छे द� 39)।�

3. समान��याय�और�गरीब��को��न:शु�क��व�धक�सहायता�उपल�ध�कराना� (अनु�छे द� 39 ए)।�

4. काम�पाने�का , �श�ा�पाने�के �और�बेकारी�बुढ़ापा�बीमारी�और��वकलांगता�के �मामल��म��लोक�सहायता�पाने�के �अ�धकार�को�संर��त�करना


(अनु�छे द� 41)।�

5. काय��क���यायसंगत�और�मानवो�चत�दशा��का�तथा��सू�त�सहायता�का�उपबंध�करना (अनु�छे द� 42)।�

6. सभी�कामगार��के ��लए��नवा�ह�मज�री� , �श��जीवन��तर�तथा�सामा�जक�और�सां�कृ �तक�अवसर�(अनु�छे द� 43)।�

7. उ�ोग��के ��बंधन�म��कामगार��क��भागीदारी�को�सुर��त�करने�के ��लए�कदम�उठाना�(अनु�छे द� 43 ए)।�

8. पोषण��तर�और�लोग��के �जीवन��तर�को�ऊँ चा�करना�और�लोक��वा��य�म��सुधार� (अनु�छे द� 47)।�

नोट: 3 और�4 गांधीवाद���स�ांत��पर�आधा�रत�ह�।�

QUESTION 25.
अनु�छे द� 368 के �अंतग�त�संशोधन����या�के �स�दभ��म���न�न�ल�खत�म��से�कौन�सा�कथन�सही�है� ?

1. संसद�के ���येक�सदन�और�रा�य��वधानसभा�म��संशोधन��वधेयक�आर�भ��कया�जा�सकता�है।�

2. इसे�एक�मं�ी��ारा�पेश��कया�जा�सकता�है� , �नजी�सद�य�के ��ारा�नह�।�

3. य�द�रा�यसभा�के ��ारा��वधेयक�को�पा�रत�नह���कया�जाता�है�तो�संयु��बैठक�बुलाई�जाती�है।�

उपरो��कथन��म��से�कौन�सा /से�सही�है� ?

a) 1 और�2
b) 2 और�3
c) के वल�3
d) उपरो��सभी�
Correct Answer: D
Your Answer:
Explanation

IASbaba
Web: http://ilp.iasbaba.com/ Score:
Email: ilp@iasbaba.com 4.00 / 200
Page 106
Exam Title : Test 21-Revision: Polity & ...
Email
Contact :

Solution (d)

संशोधन����या�

अनु�छे द� 368 म��व�ण�त�संशोधन�क�����या�इस��कार�है :

1. सं�वधान�संशोधन�का�आरंभ�संसद�के ��कसी�भी�सदन�म��इस�उ�े �य�हेतु��वधेयक�पुर :�था�पत�करके �ही��कया�जा�सके गा�, तथा�रा�य


�वधा�यका��म��नह�।�

2. �वधेयक , या�तो�एक�मं�ी�या��कसी��नजी�सद�य��ारा�पुर :�था�पत��कया�जा�सकता�है�और�इसके ��लए�रा�प�त�क��पूव��अनुम�त�क�


आव�यकता�नह��है।�

3. �वधेयक�को���येक�सदन�म���वशेष�ब�मत�से�पा�रत��कया�जाना�चा�हए� , अथा�त�ब�मत� (जो��क�50 ��तशत�से�अ�धक�है ) सदन�क��कु ल


सद�यता�और�सदन�म��उप��थत�सद�य��का�दो -�तहाई�ब�मत�या�मतदान��ारा�होना�चा�हए।�

4. ��येक�सदन�म���वधेयक�को�अलग -अलग�पा�रत�कराना�होगा।�दोन��सदन��के �बीच�असहम�त�के �मामले�म�� , दो�सदन��क��संयु��बैठक


आयो�जत�करने�का�कोई��ावधान�नह��है।�

5. य�द��वधेयक�सं�वधान�के �संघीय��ावधान��म��संशोधन�करना�चाहता�है� , तो�इसे�आधे�रा�य�वधान�मंडल��के ��ारा��साधारण�ब�मत�से


�वीकृ �त�द��जानी�चा�हए�, जो�सदन�म��उप��थत�सद�य��के �मतदान�के �तहत�हो।�

6. संसद�के �दोन��सदन���ारा��व�धवत��प�से�पा�रत�होने�एवं�रा�य�के ��वधानसभा��के ��ारा�अनुमो�दत��कए�जाने�के �बाद�, जहां�आव�यक


हो�, इस��वधेयक�रा�प�त�क��सहम�त�के ��लए���तुत��कया�जाता�है।�

7. रा�प�त�को��वधेयक�को�अपनी�सहम�त�दे नी�होती�है�वह�न�तो��वधेयक�को�अपने�पास�रख�सकते�ह��और�न�ही�संसद�के �पुन�व�चार�के ��लए


�वधेयक�को�वापस�लौटा�सकते�ह�।�

8. रा�प�त�क��मंजूरी�के �बाद�, �वधेयक�एक�अ�ध�नयम�बन�जाता�है� (यानी�, एक�संवैधा�नक�संशोधन�अ�ध�नयम ) और�सं�वधान�म��अ�ध�नयम


के �समान�इसे�शा�मल�कर��लया�जाता�है।�

QUESTION 26.
�न�न�ल�खत�म��से�कौन�सा /से�संघीय�सरकार�क���वशेषताएं�ह�� ?

1. �ै ध�राजप��त�

2. �ल�खत�सं�वधान�

3. ��सदनीय��वधा�यका�

4. �वतं���यायपा�लका�

नीचे�कू ट�से�चयन�क��जए� ?

a) 1 और�4
b) 2 और�3
c) 1,2 और�3
d) उपरो��सभी�
Correct Answer: D
Your Answer:
Explanation

Solution (d)

संघीय�सरकार�के �ल�ण�

IASbaba
Web: http://ilp.iasbaba.com/ Score:
Email: ilp@iasbaba.com 4.00 / 200
Page 107
Exam Title : Test 21-Revision: Polity & ...
Email
Contact :

· �ै ध�राजप��त /दोहरी�सरकार(अथा�त्� , रा�ीय�सरकार�और��े�ीय�सरकार )

· �ल�खत�सं�वधान�

· क� ��और�रा�य�के �बीच�श��य��का��वभाजन�

· सरकार�

· सं�वधान�क��सव��चता�

· कठोर�सं�वधान�

· �वतं���यायपा�लका�

· ��सदनीय��वधानमंडल�

QUESTION 27.
य�द�रा�य�सभा�इस�संदभ��म��एक���ताव�पा�रत�करती�है�तो�संसद�रा�य�सूची�के ��वषय��पर�भी�कानून�बना�सकती�है।�इस�स�दभ��म�
�न�न�ल�खत�कथन��म��से�कौन�सा /से�सह��है� ?

1. इस���ताव�को�उप��थत�तथा�मतदान�करने�वाले�सद�य��के �दो��तहाई�से�सम�थ�त�होना�चा�हए।�

2. यह���ताव�एक�वष��के ��लए�लागू�रहता�है�तथा�इसके �बाद�उसे�नवीनीकृ त�करना�होता�है।�

3. यह���ताव�रा�य��वधानम�डल�को�उसी�मामले�पर�कानून�बनाने�के ��लए���तबं�धत�करता�है।�

नीचे�कू ट�से�चयन�क��जए :

a) 1 और�2
b) 2 और�3
c) 1 और�3
d) उपरो��सभी�
Correct Answer: A
Your Answer:
Explanation

Solution (a)

रा�य��े��म��संसद�य��वधान�

क� ��और�रा�य��के �बीच��वधायी�श��य��के ��वतरण�क��उपरो��योजना�सामा�य�काल�के ��लए�है।�ले�कन� , असामा�य�कल�म�� , �वतरण�क�


योजना�या�तो�संशो�धत�क��जाती�है�या��नलं�बत�कर�द��जाती�है��सरे�श�द��म�� , सं�वधान�संसद�को��न�न�ल�खत�पांच�असाधारण�प�र��थ�तय�
म��रा�य�सूची�म���कसी�भी�मामले�म��कानून�बनाने�क��श����दान�करता�है :

जब�रा�य�सभा�एक�संक�प�पा�रत�करती�है�

य�द�रा�यसभा�यह�घोषणा�करती�है��क�रा�ीय��हत�म��यह�आव�यक�है��क�संसद�को�रा�य�सूची�के ��कसी�मामले�पर�कानून�बनाना�चा�हए� , तो
संसद�उस�मामले�पर�कानून�बनाने�के ��लए�स�म�हो�जाती�है।�इस�तरह�के ���ताव�को�उप��थत�सद�य��के �दो-�तहाई�ब�मत��ारा�सम�थ�त�होना
चा�हए।�संक�प�एक�वष��के ��लए�लागू�रहता�है� ; इसे�असं�य�बार�नवीनीकृ त��कया�जा�सकता�है� , ले�कन�एक�बार�म��एक�साल�से�अ�धक�के
�लए�नह��बढाया�जा�सकता�है।�

यह��ावधान��कसी�रा�य�वधान�मंडल�को�उसी�मामले�पर�कानून�बनाने�से���तबं�धत�नह��करता�है।�ले�कन� , एक�रा�य�कानून�और�एक�संसद�य
कानून�के �बीच�असंग�त�के �मामले�म�� , उ�रा�� ��व�था�मा�य�होगी।�

IASbaba
Web: http://ilp.iasbaba.com/ Score:
Email: ilp@iasbaba.com 4.00 / 200
Page 108
Exam Title : Test 21-Revision: Polity & ...
Email
Contact :

रा�ीय�आपातकाल�के �दौरान�

रा�ीय�आपातकाल��भावी�हो�जाये , तो�संसद�को�रा�य�सूची�म��मामल��के �संबंध�म��कानून�बनाने�क��श����ा�त�हो�जाती�है।�आपातकाल�के


संचालन�को�समा�त�होने�के �छह�महीने�बाद�तक�यह��व�था��भावी�रहेगी।�

यहां�भी� , एक�रा�य��वधा�यका�क��उसी�मामले�पर�कानून�बनाने�क��श�����तबं�धत�नह��होती�है।�ले�कन� , रा�य�कानून�और�संसद�य�कानून


के �बीच���तकू ल�प�र��थ�त�म�� , उ�रा�� �का��बल�होना�है।�

जब�रा�य�एक�अनुरोध�कर��

जब�दो�या�अ�धक�रा�य��के ��वधान�मंडल�रा�य�सूची�म��एक�मामले�पर�कानून�बनाने�के ��लए�संसद�से�अनुरोध�करते��ए�संक�प�पा�रत�कर�� , तो


संसद�उस�मामले�को��व�नय�मत�करने�के ��लए�कानून�बना�सकती�है।�

ऐसा�कानून�जो�के वल�उन�रा�य��पर�लागू�होता�है��ज�ह�ने���ताव��को�पा�रत��कया�है�हालां�क� , बाद�म��कोई�भी�अ�य�रा�य�को�उसके


�वधानमंडल�म��इस�स�दभ��म��एक���ताव�पा�रत�करके �इसे�लागु�सकता�है।�इस�तरह�के �कानून�को�संसद��ारा�ही�संशो�धत�या��नर�त��कया�जा
सकता�है� , संबं�धत�रा�य��के ��वधा�यका��के ��ारा�नह�।�उपरो���ावधान�के �तहत�संसद�एक�ऐसे�मामले�के �संबंध�म��भी�कानून�बना�सकती�है
�जन�पर�उसे�सीधे�श����द��नह��क��गयी�है।��सरी�ओर� , ऐसे�मामले�म��रा�य��वधानमंडल�क��कानून�बनाने�क��श���समा�त�हो�जाती�है।
रा�य���ारा�इस�तरह�के ���ताव�पा�रत�करने�का�अ�भ�ाय�यह�है��क�उ�ह�ने�अपने��वधानमंडल��नमा�ण�क��श���को��थ�गत�या�सम�प�त�कर
�दया�है�तथा�सब�कु छ�संसद�के �हाथ��म��स�प��दया�है।�

उपरो���ावधान�के �तहत�पा�रत�कानून�के �कु छ�उदाहरण�है - पुर�कार���तयो�गता�अ�ध�नयम� , 1955 ; व�य�जीव�(संर�ण ) अ�ध�नयम�, 19


72 ; जल�(��षण��नयं�ण�और��नवारण ) अ�ध�नयम�, 1974 ; शहरी�भू�म�

(अ�धकतम�सीमा�और��व�नयमन) अ�ध�नयम�, 1976 ; और�मानव�अंग��के ���यारोपण�अ�ध�नयम�, 1994।

अंतरा��ीय�समझौते�को�लागू�करने�के ��लए�

संसद�अंतररा�ीय�सं�धय�� , समझौत��या�स�मेलन��को�लागू�करने�के ��लए�रा�य�सूची�म���कसी�भी�मामले�पर�कानून�बना�सकती�है।�यह��ावधान


क� ��सरकार�को�अपने�अंतररा�ीय�दा�य�व��और���तब�ता��को�पूरा�करने�म��स�म�बनाता�है।�

उपरो���ावधान��के �तहत�लागू��कए�गए�कु छ�कानून��के �उदाहरण�है : संयु��रा�� (�वशेषा�धकार�और�

उ�मु�� ) अ�ध�नयम�, 1947 ; �जनेवा�क�व�शन�अ�ध�नयम� , 1960 ; अपहरण�के ��खलाफ�अ�ध�नयम�, 1982 और�पया�वरण�से�संबं�धत


कानून�और����स।�

रा�प�त�शासन�के �दौरान�

जब��कसी�रा�य�म��रा�प�त�शासन�लागू�होता�है� , तो�संसद�को�उस�रा�य�के �संबंध�म��रा�य�सूची�म���कसी�भी�मामले�के �संबंध�म��कानून�बनाने


का�अ�धकार��ा�त�होता�है।�रा�प�त�शासन�के �उपरा�त�भी�संसद��ारा�बनाया�गया�कानून��भावी�रहता�है।�इसका�अथ��यह�है��क�इस�कानून�के
�व�त�त�होने�क��अव�ध�रा�प�त�शासन�क��अव�ध�से��वतं��है� , ले�कन� , इस�तरह�के �कानून�को�रा�य��वधान�मंडल��ारा��नर�त�या�प�रव�त�त
या��फर�से�लागू��कया�जा�सकता�है।�

QUESTION 28.
सं�वधान�म��अनु�छे द� 280 के �तहत��व��आयोग�का��व�था��कया�गया�है।��व��आयोग�के �स�दभ��म���न�न�ल�खत�म��से�कौन�सा /से�कथन�सही
है� ?

1. रा�प�त��ारा���येक�पांच�वष��म��या�इसके �पहले�भी�इसका�गठन��कया�जाता�है।�

2. यह�के ���और�रा�य�के �बीच�कर��के ��वतरण�क���सफा�रश�करता�है।�

3. �व��आयोग�क���सफा�रश��सरकार�पर�बा�यकारी�ह�।�

नीचे�कू ट�से�चयन�क��जए :

a) 1 और�2

IASbaba
Web: http://ilp.iasbaba.com/ Score:
Email: ilp@iasbaba.com 4.00 / 200
Page 109
Exam Title : Test 21-Revision: Polity & ...
Email
Contact :

b) 2 और�3
c) 1 और�3
d) उपरो��सभी�
Correct Answer: A
Your Answer:
Explanation

Solution (a)

�व��आयोग�

अनु�छे द� 280 एक�अध���या�यक��नकाय�के ��प�म���व��आयोग�क���व�था�करता�है ; रा�प�त��ारा�हर�पांच�वष��म��या�उससे�पहले�भी�इसका


गठन��कया�जाता�है�यह��न�न�ल�खत�मामल��पर�रा�प�त�को��सफा�रश��करता�है :

I. के ���और�रा�य��के �बीच�साझा��कए�जाने�वाले�कर��के �शु��आय�का��वतरण� , और�ऐसी��ा��तय��का�रा�य��के �बीच��ह�सेदारी�का��नधा�रण।

II. वे��स�ांत��जसके �तहत�रा�य�क� ��(अथा�त्�भारत�क��सं�चत��न�ध�से ) आ�थ�क�अनुदान�लेकर�काय��करता�है।�

III. रा�य��व��आयोग�के ��ारा�क��गई��सफा�रश��के �आधार�पर�रा�य�म��पंचायत�और�नगर�पा�लका��के ��ोत��क��पू�त��के ��लए�रा�य�के


सं�चत��न�ध�को�बढ़ाने�के ��लए�आव�यक�उपाय�सुझाना।�

IV. �कसी�भी�अ�य�मामले�के �स�ब�ध�म��रा�प�त�के ��ारा�स�पे�गए�अ�य�काय�।�

1960 तक�, आयोग�ने�असम� , �बहार�, उड़ीसा�और�प��म�बंगाल�रा�य��के ��लए�जूट�और�जूट�उ�पाद��पर��नया�त�शु�क�के �एवज�म����तवष�


�दान��कये�जाने�वाले�अनुदान�के �स�ब�ध�म��सरकार�को�सुझाव��दया।�

सं�वधान�म� , �व��आयोग�को�भारत�म���व�ीय�संघवाद�के �संतुलन�प�हये�के ��प�म��शा�मल��कया�गया�है।�हालां�क� , क� �-रा�य�राजकोषीय


संबंध��म��इसक��भू�मका�को�योजना�आयोग� , एक�गैर -संवैधा�नक�और�गैर -सां�व�धक��नकाय�के �उ�व�ने�कमजोर�कर��दया�है।�

QUESTION 29.
के ���रा�य�संबंध��क��परी�ण�के ��लए��न�न�ल�खत�म��से�कौन�से�आयोग�का�गठन��कया�गया�था� ?

1. धर�आयोग�

2. राजम�ार�आयोग�

3. पूंछ��आयोग�

4. सरका�रया�आयोग�

नीचे�कू ट�से�चयन�क��जए� ?

a) 1,2 और�3
b) 2,3 और�4
c) 1,2 और�4
d) उपरो��सभी�
Correct Answer: B
Your Answer: Unanswered
Explanation

Solution (b)

1960 के �दशक�के �म�य�से�क� �-रा�य�संबंध��म��मु���पर��वचार��कया�जा�रहा�है।�इस��दशा�म�� , �न�न�ल�खत�कमीशन�का�गठन��कया�गया�है :

IASbaba
Web: http://ilp.iasbaba.com/ Score:
Email: ilp@iasbaba.com 4.00 / 200
Page 110
Exam Title : Test 21-Revision: Polity & ...
Email
Contact :

· �शास�नक�सुधार�आयोग�

· राजम�ार�स�म�त�

· आनंदपुर�सा�हब�संक�प�

· प��म�बंगाल��ापन�

· सरका�रया�आयोग�

· पूंछ��आयोग�

QUESTION 30.
रा�ीय�आपातकाल�के �स�दभ��म���न�न�ल�खत�म��से�कौन�सा /से�कथन�सही�है� ?

1. अनु�छे द� 359 रा�ीय�आपातकाल�के �मामले�म��मूल�अ�धकार��को��नलं�बत�कर�दे ता�है।�

2. रा�ीय�आपातकाल�के �दौरान�अनु�छे द� 20 और�21 ��तबं�धत�ह�।�

नीचे�कू ट�से�चयन�क��जए :

a) के वल�1
b) के वल�2
c) दोन��1 और�2
d) न�तो�1 और�न�ही�2
Correct Answer: D
Your Answer:
Explanation

Solution (d)

अनु�छे द� 358 और�359 के �बीच��वभेद :

अनु�छे द� 358 और�359 के �बीच��न�न�ल�खत�अंतर�है :

1. अनु�छे द� 358 के वल�अनु�छे द� 19 के �तहत�मौ�लक�अ�धकार��तक�सी�मत�है� , जब�क�अनु�छे द� 359 उन�सभी�मौ�लक�अ�धकार��से


स�बं�धत�ह���जनके ��वत�न�को�रा�प�त�के �आदे श��ारा��नलं�बत�कर��दया�जाता�है।�

2. अनु�छे द� 358 �वत: ही�आपातकाल�क��घोषणा�होने�पर�अनु�छे द� 19 के �तहत�मौ�लक�अ�धकार��को��नलं�बत�कर�दे ता�है।��सरी�ओर� , अ


नु�छे द� 359 �कसी�भी�मौ�लक�अ�धकार�को��वत: �नलं�बत�नह��करता�है�यह�रा�प�त�को�यह�श���दे ता�है��क�वह�मूल�अ�धकार��के ��नलंबन
को�लागू�कर�।�

3. अनु�छे द� 358 के वल�बा��आपातकाल�(अथा�त� , जब�आपातकाल�यु��या�बा��आ�मण�के �आधार�पर�घो�षत��कया�गया�है ) म��लागू


होता�है�न��क�आंत�रक�आपातकाल�के �मामले�म�� (अथा�त� , जब�सश����व�ोह�के �आधार�पर�आपातकाल�घो�षत��कया�जाता�है )।��सरी�ओर�,
अनु�छे द� 359 , बा��और�आंत�रक�दोन��आपातकाल�के �मामले�म��लागू�होता�ह�।�

4. अनु�छे द� 358 , अनु�छे द� 19 के �अंतग�त�शा�मल�मूल�अ�धकार��को�आपातकाल�क��संपूण��अव�ध�के ��लए��नलं�बत�कर�दे ता�है� , जब�क


अनु�छे द� 359, रा�प�त��ारा��न�द���अव�ध�तक�के ��लए, मूल�अ�धकार��के ��वत�न�को��नलं�बत�कर�दे ता�है� , जो��क�या�तो�स�पूण�
आपातकालीन�अव�ध�या�अ�पाव�ध�हो�सकती�है।�

5. अनु�छे द� 358, स�पूण��दे श�म��तथा�अनु�छे द� 359, स�पूण��दे श�अथवा�इसके �कु छ��ह�से�म��लागू�हो�सकता�है।�

6. अनु�छे द� 358, अनु�छे द� 19 को�पूरी�तरह�से��नलं�बत�कर�दे ता�है� , जब�क�अनु�छे द� 359, अनु�छे द� 20 और�21 के ��नलंबन�को�लागु
नह��करता�है।�

IASbaba
Web: http://ilp.iasbaba.com/ Score:
Email: ilp@iasbaba.com 4.00 / 200
Page 111
Exam Title : Test 21-Revision: Polity & ...
Email
Contact :

7. अनु�छे द� 358, रा�य�को�अनु�छे द� 19 के �तहत�मौ�लक�अ�धकार�से�सा�य�नह��रखने�वाले��नयम�बनाने�अथवा�काय�कारी�कदम�उठाने�का


अ�धकार�दे ता�है ; जब�क�अनु�छे द� 359 के वल�उ�ही�मूल�अ�धकार��के �स�ब�ध�म��ऐसे�काय��करने�का�अ�धकार�दे ता�है��ज�ह��रा�प�त�के �आदे श
के ��ारा��नलं�बत��कया�गया�है।�

अनु�छे द� 358 और�अनु�छे द� 359 के �बीच�समानता�भी�है।�ये�दोन��उ�ह���व�धय��को�चुनौती�से�उ�मु����दान�करते�ह��जो��क�आपातकाल�से


संबं�धत�ह��और�उनको�नह��जो�आपातकाल�से�संबं�धत�नह��ह�।�इसके �अलावा�, ऐसे��व�धय��के �तहत�क��गई�काय�कारी�कार�वाई�को�भी�दोन�
अनु�छे द�संर�ण�दे ते�है।�

44व��संशोधन�अ�ध�नयम� 1978, अनु�छे द� 359 के �दायरे�को�दो�तरीके �से���तबं�धत�करता�है� -

· रा�प�त�अनु�छे द� 20 तथा�21 के �अंतग�त��दए�गए�अ�धकार��को�लागू�करने�के ��लए��यायालय�म��जाने�के �अ�धकार��नलं�बत�नह��कर�सकता


ह�।��सरे�श�द��म�� , अपराध��के �दोष�स���से�संर�ण (अनु�छे द� 20) और��ाण�एवं�दै �हक��वतं�ता�का�अ�धकार� (अनु�छे द� 21) आपातकाल�क
दौरान�भी�लागू�रहते�ह�।�

· �सरे� , के वल�उ�ही��व�धय��को�चुनौती�दे ने�से�संर�ण��ा�त�है� , जो�आपातकाल�से�स�ब��धत�है , न��क�अ�य��व�धय��और�काय��को�नह��जो


इसके �तहत�बनाए�गए�है।�

QUESTION 31.
भारत�म��रा�प�त�चुनाव�लड़ने�के ��लए��न�न�ल�खत�म��से�कौन -कौन�सी�अह�ताएं�होनी�आव�यक�है� ?

1. एक�����को�ज�म�से�भारत�का�नाग�रक�होना�चा�हए।�

2. ����ने� 35 वष��क��आयु�पूरी�कर�ली�हो।�

3. ����को�लोकसभा�के �सद�य�के ��प�म���नवा��चत�होने�के ��लए�अ�ह�त�होना�चा�हए।�

4. वह�लाभ�का�कोई�पद�धारण�न�करता�हो।�

नीचे�कू ट�से�चयन�क��जए� ?

a) 1,2 और�3
b) 2,3 और�4
c) 1,3 और�4
d) उपरो��सभी�
Correct Answer: B
Your Answer:
Explanation

Solution (b)

रा�प�त�के ��प�म���नवा�चन�के ��लए�अह�ताएं :

रा�प�त�पद�के �चुनाव�के ��लए�����को��न�न�ल�खत�अह�ताएं�पूण��करना�आव�यक�है :

· वह�भारत�का�नाग�रक�हो।�

· उसने� 35 साल�क��आयु�पूरी�कर�ली�हो।�

· वह�लोकसभा�का�सद�य��नवा��चत�होने�के ��लए�अ�ह�त�है।�

· वह�क� ��सरकार�या��कसी�रा�य�सरकार�या��कसी��थानीय��ा�धकरण�या��कसी�अ�य�साव�ज�नक��ा�धकरण�के �तहत�लाभ�का�पद�धारण�न


करता�हो।�एक�पदे न�रा�प�त�या�उपरा�प�त�या��कसी�भी�रा�य�का�रा�यपाल�और�संघ�अथवा�रा�य�का�मं�ी�लाभ�का�पद�नह��माना�जाता�है
और�इस�लए�वह�रा�प�त�पद�के ��लए�अह�क�उ�मीदवार�होता�है।�

IASbaba
Web: http://ilp.iasbaba.com/ Score:
Email: ilp@iasbaba.com 4.00 / 200
Page 112
Exam Title : Test 21-Revision: Polity & ...
Email
Contact :

इसके �अ�त�र��, रा�प�त�पद�के �चुनाव�के ��लए�उ�मीदवार�के �नामांकन�के ��लए�कम�से�कम� 50 ��तावक�व�50 अनुमोदक�होने�चा�हए।


��येक�उ�मीदवार�को��रज़व��ब�क�ऑफ�इं�डया�म�� 15 , 000 �पये�क��जमानत�रा�श�जमा�करनी�होगी।�य�द�उ�मीदवार�कु ल�डाले�गये�मत��का
1/6 भाग��ा�त�करने�म��असमथ��रहता�है�तो�यह�रा�श�ज�त�हो�जाती�है।�

�वचार�कर��

· उपरा�प�त�के �चुनाव�के ��लए�अह�ताएं :

QUESTION 32.
भारत�के �रा�प�त�के ��ारा��न�न�ल�खत�वीटो�श���म��से��कसका��योग�नह���कया�जाता�है� ?

a) �नरपे��वीटो�
b) �वशे�षत�वीटो�
c) �नलं�बत�वीटो�
d) पॉके ट�वीटो�
Correct Answer: B
Your Answer:
Explanation

Solution (b)

आधु�नक�रा�य��के �काय�कारी��मुख��क��वीटो�श��य��को��न�न�ल�खत�चार��कार��म��वग�कृ त��कया�जा�सकता�है :

· आ�यं�तक�वीटो� , अथा�त्��वधा�यका��ारा�पा�रत��वधेयक�पर�अपनी�राय�सुर��त�रखना।�

· �वशे�षत�वीटो� , जो��वधा�यका��ारा�उ�च�ब�मत��ारा��नर�त(ओवरराइड) �कया�जा�सकता�है।�

· �नलंबनकारी�वीटो� , जो��वधा�यका��ारा�साधारण�ब�मत��ारा��नर�त��कया�जा�सकता�है।�

· पॉके ट�वीटो�, �वधा�यका��ारा�पा�रत��वधेयक�पर�कोई��नण�य�नह��करना।�

उपरो��चार�म��से� , भारत�के �रा�प�त�को�तीन�श��यां - आ�यं�तक�वीटो� , �नलंबनकारी�वीटो�और�पॉके ट�वीटो��न�हत�है।�भारत�के �रा�प�त�के


मामले�म���वशे�षत�वीटो�मह�वहीन�है� ; तथा�यह�अमे�रक��रा�प�त�के ��ारा��योग��कया�जाता�है।�

QUESTION 33.
रा�प�त�और�रा�यपाल�के ��मादान�क��श���के �म�य��न�न�ल�खत�म��से�कौन�सा�कथन��प���प�से��भ��है� ?

1. रा�प�त�सै�य��यायालय (कोट� �माश�ल ) �ारा��दए�गए�सजा�को�माफ�कर�सकते�ह�� , जब�क�रा�यपाल�नह��कर�सकता�है।�

2. रा�प�त�मृ�युदंड�को��मा�कर�सकते�ह� , जब�क�रा�यपाल�नह��कर�सकता�है।�

नीचे�कू ट�से�चयन�क��जए :

a) के वल�1
b) के वल�2
c) दोन��1 और�2
d) न�तो�1 और�न�ही�2
Correct Answer: C
Your Answer:
Explanation

Solution (c)

IASbaba
Web: http://ilp.iasbaba.com/ Score:
Email: ilp@iasbaba.com 4.00 / 200
Page 113
Exam Title : Test 21-Revision: Polity & ...
Email
Contact :

�मादान�श��य��म���भ�ता :

रा�यपाल�क���मादान�श��यां , रा�प�त�क��तुलना�म��अलग�है :

1. रा�प�त�सै�य��यायालय��ारा�द��गयी�सजा�को�माफ़�कर�सकते�ह�� , जब�क�रा�यपाल�नह�।�

2. रा�प�त�मृ�युदंड�को��मा�कर�सकते�ह��पर�तु�रा�यपाल�नह�।�यहां�तक� �क�रा�य��व�ध�के �तहत�मृ�युद�ड�क��सजा�सजा�को��मा�करने�क�


श���रा�प�त�म���न�हत�है� , न��क�गवन�र�म�।�हालां�क� , रा�यपाल�मृ�युद�ड�को��नलं�बत� , दं डाव�ध�कम�कर�सकता�है।��सरे�श�द��म�� , मृ�युदण्
ड�के ��नलंबन� , दं डाव�ध�को�कम�करने�और�द�ड�के ��व�प�बंदलने�के �स�ब�ध�म��म��रा�यपाल�और�रा�प�त�दोन��क��श��यां�समान�है।�

QUESTION 34.
संसद�के �ऊपरी�सदन�के �संदभ��म���न�न�ल�खत�कथन��पर��वचार�क��जए :

1. रा�य��के ���त�न�धय��को�रा�य��वधानसभा��के ��नवा��चत�सद�य���ारा�चुना�जाता�है।�

2. सभी�रा�य��को�रा�यसभा�म��एकसमान���त�न�ध�व��दया�गया�है।�

3. संघ�रा�य��े���को�रा�यसभा�म����त�न�ध�व�नह���दया�गया�है।�

उपरो��कथन��म��से�कौन�सा /से�सही�ह�� ?

a) के वल�1
b) 2 और�3
c) 1 और�3
d) उपरो��सभी�
Correct Answer: A
Your Answer:
Explanation

Solution (a)

रा�यसभा�क��संरचना�

रा�यसभा�क��अ�धकतम�सं�या� 250 तय�क��गई�है� , �जसम��से� 238 सद�य�रा�य��और�संघ�शा�सत��दे श��के ���त�न�ध�(अ��य���प�से�चुने


गए) होते�है�और� 12 सद�य�रा�प�त��ारा�ना�मत��कए�जाते�ह�।�

वत�मान�म�� , रा�य�सभा�म�� 245 सद�य�ह�।�इनम��से� 229 सद�य�रा�य��का���त�न�ध�व�करते�ह�� , 4 सद�य�संघ�शा�सत��दे श��का���त�न�ध�व


करते�ह��और� 12 सद�य��को�रा�प�त��ारा�ना�मत��कया�जाता�है।�

सं�वधान�क��चौथी�अनुसूची�रा�यसभा�म��रा�य��और�क� ��शा�सत��दे श��म��सीट��के �आवंटन�से�संबं�धत�है।�

1. रा�य��का���त�न�ध�व�

रा�य�सभा�म��रा�य��के ���त�न�धय��को�रा�य��वधान�सभा��के ��नवा��चत�सद�य���ारा�चुना�जाता�है।�चुनाव , आनुपा�तक���त�न�ध�व��णाली


के �अनुसार�एकल�ह�तांतरणीय�मत�के �मा�यम�से�आयो�जत��कया�जाता�है।�रा�यसभा�के ��लए, रा�य��क��सीट��का�बंटवारा�उनक��जनसं�या�के
आधार�पर�आवं�टत��कया�जाता�ह�।�इस�लए� , रा�य�के ���त�न�धय��क��सं�या�अलग -अलग�रा�य�म���भ� -�भ��होती�है।�उदाहरण�के ��लए�, उ�
र��दे श�म�� 31 सद�य�ह��जब�क���पुरा�के �के वल�1 सद�य�ह�।�

हालां�क� , संयु��रा�य�अमे�रका�म�� , सभी�रा�य��को�उनक��आबाद��के �बावजूद�सीनेट�म��समान���त�न�ध�व��दया�जाता�है।�संयु��रा�य


अमे�रका�म�� 50 रा�य�ह��और���येक�रा�य�से�सीनेट�म��कु ल�100 सीट� �ह��तथा���येक�रा�य�को� 2 सीट� ��ा�त�ह�।�

2. संघ�रा�य��े���का���त�न�ध�व�

IASbaba
Web: http://ilp.iasbaba.com/ Score:
Email: ilp@iasbaba.com 4.00 / 200
Page 114
Exam Title : Test 21-Revision: Polity & ...
Email
Contact :

रा�यसभा�म��संघ�रा�य��े��का���येक���त�न�ध�परो���प�से�इस�उ�े �य�के ��लए�ग�ठत�एक��नवा�चक�म�डल (इले��ोराल�कॉलेज ) के �सद�य�


�ारा��नवा��चत��कया�जाता�है।�आनुपा�तक���त�न�ध�व��णाली�के �अनुसार�एकल�ह�तांतरणीय�मत�के �मा�यम�से�आयो�जत��कया�जाता�है।
सात�संघ�शा�सत��दे श��म��से�के वल�दो�(�द�ली�और�पुडुचेरी ) को�रा�यसभा�म����त�न�ध�व��दया�गया�ह�।�अ�य�पांच�क� ��शा�सत��दे श��क�
आबाद��रा�य�सभा�म��उनके ���त�न�ध�व�के ��लए�ब�त�कम�है।�

3. नामां�कत�सद�य�

रा�प�त, रा�य�सभा�म�� 12 ऐसे�सद�य��को�नामां�कत�करता�है� , �जनके �पास�कला�, सा�ह�य�, �व�ान�और�सामा�जक�सेवा�म���वशेष��ान�या


�ावहा�रक�अनुभव�हो।�ऐसे����य��के �नामांकन�के �पीछे �का�तक� �है��क�नामी�या���स��लोग��बना�चुनाव�के �रा�यसभा�म��जा�सक� ।�यहां��यान
�दया�जाना�चा�हए��क�अमे�रक��सीनेट�म��कोई�नामां�कत�सद�य�नह��होता�ह�।�

QUESTION 35.
लोकसभा�का�अ�य���कसे�संबो�धत�कर�अपने�पद�से�इ�तीफा�दे �सकता�ह� ?

a) रा�प�त�
b) उपरा�प�त�
c) �धानमं�ी�
d) उपा�य��
Correct Answer: D
Your Answer:
Explanation

Solution (d)

अ�य�(�पीकर) का�काय�काल�

अ�य��आमतौर�पर�लोकसभा�के �जीवनकाल�तक�पद�धारण�करता�है।�हालां�क� , उनका�पद��न�न�ल�खत�तीन�मामल��म��इससे�पहले�भी


समा�त�हो�सकता�है :

· य�द�वह�लोकसभा�का�सद�य�नह��रहता�है� ;

· य�द�वह�उपा�य�(�ड�ट���पीकर) को�संबो�धत�अपने�ह�ता�र�स�हत�लेख��ारा�पद��याग�कर�� ;

· तथा�य�द�लोकसभा�के �त�कालीन�सम�त�सद�य��के �ब�मत�से�पा�रत�एक�संक�प��ारा�उ�ह��हटा��दया�गया�है।�इस�तरह�का�कोई�एक�संक�प ,


14 �दन�क��अ��म�नो�टस�दे ने�के �बाद�ही�लाया�जा�सकता�है।�

जब�अ�य��हो�हटाने�के ��लए�एक�संक�प�सदन�म���वचाराधीन�है� , तो�वह�सभा�क��बैठक�म��पीठासीन�नह��होगा।�हालां�क� , वह�सदन�क�


काय�वाही�म��भाग�ले�सकते�ह��और�बोल�सकते�ह��और�पहली�बार�वोट�दे ते�ह�� , हालां�क�वोट��क��समानता�के �मामले�म��नह�।�

�वचार�कर��

· रा�य�सभा�के �अ�य��का�हटाया�जाना�

QUESTION 36.
स�टर�फॉर�डेवेलपम�ट�ऑफ�एडवांस�क��यू�ट�ग� (सी-डेक ) ने�मोबाइल�आधा�रत�ए�लीके शन�' दश�क� ' को��वक�सत��कया�है।��न�न�ल�खत�म��से
कौन�सा�कथन�इस�ए�लीके शन�के �स�दभ��म��सही�है� ?

1. इसका�उ�े �य��डफर�टली -एब�ड�लोग��के ��यू�जयम�दश�न�के �अनुभव�को�सुधारना�है।�

2. यह��रयल�टाइम��यू�जयम�आगंतुक��को�एक� QR कोड�को��कै न�करके �व�तु��या�कलाकृ �तय��के �बारे�म��सभी��ववरण�एक��करने�क�


अनुम�त�दे ता�है।�

नीचे�कू ट�से�चयन�क��जए :

IASbaba
Web: http://ilp.iasbaba.com/ Score:
Email: ilp@iasbaba.com 4.00 / 200
Page 115
Exam Title : Test 21-Revision: Polity & ...
Email
Contact :

a) के वल�1
b) के वल�2
c) दोन��1 और�2
d) न�तो�1 और�न�ही�2
Correct Answer: C
Your Answer:
Explanation

Solution (c)

दश�क�

यह�एक�मोबाइल�आधा�रत�ए�लीके शन�है।�इसका�उ�े �य��डफर�टली�एब�ड�लोग��के ��यू�जयम�दश�न�के �अनुभव�को�सुधार�करना�है।�

यह��रयल�टाइम��यू�जयम�आगंतुक��को�एक� QR कोड�को��कै न�करके �व�तु��या�कलाकृ �तय��के �बारे�म��सभी��ववरण�एक��करने�क��अनुम�त


दे ता�है।�

जतन�

इस�सॉ�टवेयर�का�उ�े �य�सं�हालय��म��संर��त�सभी�व�तु��का�एक��ड�जटल�इं���ट�तैयार�करना�है।�यह�शोधकता��� , �यूरेटर�और��े��म�


��च�रखने�वाले�लोग��क��भी�मदद�करेगा।�

�वचार�कर��

· सी-डैक�

· भारत�म��पय�टन�

QUESTION 37.
क� ��सरकार�ने� ' अटल�भूजल�योजना� ' नामक�एक�नई�योजना�क��शु�आत�क��है।�योजना�के �स�दभ��म���न�न�ल�खत�म��से�कौन�सा /से�कथन
सही�ह�� ?

1. इसका�उ�े �य��थानीय�भागीदारी�के �साथ�नद��के �जल�का�कु शल��बंधन�करना�है।�

2. लागत�का�आधा��ह�सा��व��ब�क�के ��ारा�ऋण�के �मा�यम�से��व��पो�षत�होगा�तथा�पचास���तशत��ह�सा�क� ��सरकार�के ��ारा��व��पो�षत


�कया�जाएगा।�

नीचे�कू ट�से�चयन�क��जए :

a) के वल�1
b) के वल�2
c) दोन��1 और�2
d) न�तो�1 और�न�ही�2
Correct Answer: B
Your Answer: Unanswered
Explanation

Solution (b)

अटल�भूजल�योजना�

· इसका�उ�े �य�जल�संसाधन��के �कु शल��बंधन�और�सामुदा�यक�भागीदारी�के �मा�यम�से�पुन�न�मा�ण�तं��को�मजबूत�बनाना�है।�

· �थानीय��तर�पर�लोग��को�शा�मल�करते��ए , इस�योजना�का�जोर�पुनभ�रण� (भूजल��ोत��के ) और�पानी�के �कु शल�उपयोग�पर�होगा।�

IASbaba
Web: http://ilp.iasbaba.com/ Score:
Email: ilp@iasbaba.com 4.00 / 200
Page 116
Exam Title : Test 21-Revision: Polity & ...
Email
Contact :

· अटल�भूजल�योजना�गुजरात� , महारा��, ह�रयाणा�, कना�टक� , राज�थान�, उ�र��दे श�और�म�य��दे श�म��शु��क��जाएगी।�

· अटल�भूजल�योजना�नाम�क��इस�क� ��य�योजना�के �कु ल�लागत�का�आधा�(50%) �व��ब�क�के ��ारा�ऋण�के ��प�म���दया�जायेगा�जब�क�शेष


आधा�(3 , 000 करोड़��पये ) सरकार��ारा��व��पो�षत��कया�जाएगा।�

· �ाम�जल��बंधन�म��ल�य�हा�सल�करने�के ��लए��ो�साहन�के ��प�म���ाम�पंचायत��स�हत�रा�य��को�सरकार� 50% धन�दे ने�क��योजना�बना


रही�है।�यह�सामुदा�यक�भागीदारी�और��वहार�प�रवत�न�को��ो�सा�हत�करने�के ��लए�पहला�कदम�है।�

· शेष� 50% धनरा�श�सं�थागत��व�था��को�मजबूत�करने�के ��लए�रा�य��को�द��जाएगी�जैसे�भूजल�के ��थायी��बंधन�को�पूरा�करने�म��मदद


करने�के ��लए�एक�मजबूत�डेटाबेस�और�वै�ा�नक����कोण�उपल�ध�करवाना।�

· �ाम�पंचायत�जो�जल�सुर�ा�क��योजना�तैयार�करते�ह��और�जल�क��आपू�त��बढ़ाने�के ��लए�आधारभूत�ढांचा�रखते�ह�� , उ�ह���ो�साहन��मलेगा।


आपू�त��म��वृ���करने�के ��लए�एक�रा�य��जतने�अ�धक�कदम�उठाएंगे� , उ�ह��उतना�ही�अ�धक��ो�साहन��मलेगा।�यह�समुदाय�भागीदारी�के
मा�यम�से�आ�म -�नयमन(से�फ�रेगुलेशन ) क��अवधारणा�को�लाने�का�एक��वचार�है।�

�या�आप�जानते�है� ?

· 2016-17 के �बजट�घोषणाप��के ���तउ�र�म��मं�ालय�ने��ारंभ�म��रा�ीय�भूजल��बंधन�सुधार�योजना� (एनजीएमआईएस) क��अवधारणा


�दया�था।�

· हालां�क� , मई�2017 म���य��व��स�म�त�ने�एनजीएमआईएस�को�ख़ा�रज�कर��दया�और��सफा�रश��कया��क�मं�ालय�एक�सम����ताव�के


साथ�योजना�को�पुन : �था�पत�करे।�

· इसके �बाद�इस�योजना�का�पुन�था�पन��कया�गया�है�और�इसका�नाम�बदलकर�अटल�भूजल�योजना�कर��दया�गया।�

https://timesofindia.indiatimes.com/home/environment/centre-pushes-rs-6000cr-plan-to-tackle-
water-depletion/articleshow/62923993.cms

QUESTION 38.
हाल�ही�म��मेला�सडीन ( Malacidins ) के �नाम�से�एंट�बायो�टक�दवा��के �एक�नए�वग��क��खोज�क��गई�है।��न�न�ल�खत�म��से�कौन�सा /से
कथन�इस�दवा�के �संबंध�म��सही�है� ?

1. इ�ह��मृदा�और�गंदगी�म��रहने�वाले�सू�मजीव��के ��ारा�उ�पा�दत��कया�जाता�है।�

2. ये�ब��दवा���तरोधी�रोगज़नक़��के ��व���स��य�ह�।�

नीचे�कू ट�से�चयन�क��जए :

a) के वल�1
b) के वल�2
c) दोन��1 और�2
d) न�तो�1 और�न�ही�2
Correct Answer: C
Your Answer:
Explanation

Solution (c)

मैला�स�डन ( Malacidins )

· ये�एंट�बायो�ट�स�क���व�श��वग��है , जो�आमतौर�पर�मृदा�के �सू�मजीव��म��एनकोडेड�होते�ह�।�संवध�न -आधा�रत�NP ( �ाकृ �तक�उ�पाद)


खोज��यास��म��इनक��कभी�भी�सूचना�नह���मली�है।�

IASbaba
Web: http://ilp.iasbaba.com/ Score:
Email: ilp@iasbaba.com 4.00 / 200
Page 117
Exam Title : Test 21-Revision: Polity & ...
Email
Contact :

· पशु�घाव�मॉडल�म��मे�थ�स�लन -��तरोधी��टे �फलोकोकस�ऑ�रयस� ( MRSA) �वचा�सं�मण�को�बा�धत�करते�ह��और��योगशाला���थ�तय�


म����तरोध�के ��लए�चयन�नह��करते�ह�।� MRSA �वचा�के �सं�मण�के �साथ�चूह��पर�मैला�स�डन�का�परी�ण��कया�गया�था।�हालत�ठ�क�हो�गई
थी�, और�मैला�स�डन�के �साथ�जारी�संपक� �के �20 �दन��के �बाद�भी�, कृ �तक��को��कसी����भाव�का�अनुभव�नह���आ।�

· ये�ब�औष�ध (म�ट��ग)-��तरोधी�रोगजनक��के ��खलाफ�स��य�ह�� , एक�पशु�वोउंड�मॉडल�म��मे�थ�स�लन -��तरोधी��टे �फलोकोकस�ऑ�रयस


(एम.आर.एस.ए.) �वचा�के �सं�मण�को�बा�धत�करते�ह��और��योगशाला���थ�तय��के �तहत���तरोध�(अथा�त्���तरोध�को���गर�नह��करते ) का
चयन�नह��करते�है।�

· मैला�स�डन�के वल��ाम�पॉ�ज�टव�बै�ट��रया�को�ल��त�करते�ह�।�

�या�आप�जानते�है� ?

· �ाम-नेगे�टव�बै�ट��रया�हैजा� , �यूमो�नया� , यौन�संचा�रत�बीमा�रय��और��लेग�के �कारण�हो�सकता�है।�

· �ाम-नेगे�टव�को�शका�क��द�वार��म��एक�पतली�पे�टाइडो�लाइके न�परत�(ट�कोइ�कक�ए�सड�के ��बना) होती�है�जो�एक�मोट���ला�मा��झ�ली�से


�घरा�होता�है।�

· �ाम-पॉ�ज�टव�बै�ट��रया�उनक��मोट��पे�टाइडो�लाइके न�को�शका��भ���के �कारण�ब�गनी��नशान� दे गा।�

�वचार�कर��

· सुपरबग ( Superbugs )

http://indianexpress.com/article/explained/hunt-for-new-antibiotics-hits-pay-dirt-5064385/

QUESTION 39.
हाल�ही�म��भारत�का�सव��थम�रे�डयो�उ�सव��न�न�ल�खत��थान��म��से�कहाँ�आयो�जत��आ�था� ?

a) हैदराबाद�
b) चे�ई�
c) ब�गलोर�
d) �द�ली�
Correct Answer: D
Your Answer:
Explanation

Solution (d)

भारत�का�पहला�रे�डयो�उ�सव�हाल�ही�म���द�ली�म��आयो�जत��कया�गया।�यूने�को�के �सहयोग�से�रे�डयो�और�टे ली�वजन�म��अ�तरा��ीय�म�हला


संघ�के ��ारा��योहार�आयो�जत��कया�।�

�वचार�कर��

· �सार�भारती�

· सामुदा�यक�रे�डयो�

QUESTION 40.
�न�न�ल�खत�म��से�कौन�सा�कथन� ‘ �वएन�टआन��वजन�( Vientiane Vision)' को�सही�ढं ग�से�प�रभा�षत�करता�है� ?

a) यह�आ�सयान�के �साथ�जापान�के �र�ा�सहयोग�के ��लए�एक�माग�दश���स�ांत�है।�


b) यह��वयतनाम�को�शा�मल�करने�क��चीन�क��नी�त�है।�
c) यह�एस.सी.ओ. का�माग�दश���स�ांत�है।�

IASbaba
Web: http://ilp.iasbaba.com/ Score:
Email: ilp@iasbaba.com 4.00 / 200
Page 118
Exam Title : Test 21-Revision: Polity & ...
Email
Contact :

d) यह�सी.आई.एस. के �साथ�आ�थ�क�संबंध��म��सुधार�करने�के ��लए�चीन�क��आ�थ�क�नी�तय��को��दया�गया�नाम�है।�


Correct Answer: A
Your Answer:
Explanation

Solution (a)

�वएन�टआन��वजन�( Vientiane Vision)

आ�सयान�के �साथ�जापान�के �र�ा�सहयोग�के ��लए�यह�एक�माग�दश�क��स�ांत�है� , 16 नवंबर� 2016 को��वयतनाम�, लाओ�पीडीआर�म�


आयो�जत��सरे�आ�सयान -जापान�र�ा�मं��य��क��अनौपचा�रक�बैठक�म��र�ा�मं�ी�इनाडा�के ��ारा�जापान�क��अपनी�पहल�के ��प�म��घो�षत
�कया�गया।�

यह��वज़न�पहली�बार�, पारदश��ढं ग�से� , �ाथ�मकता�वाले��े���म��आ�सयान�के �साथ�र�ा�सहयोग�क��भ�व�य�क���दशा�क��पूरी�त�वीर��दखाती


है।�

QUESTION 41.
�लोबल�एंट�माइ�ो�बयल�रे�सट� स��नगरानी��णाली� (�लास) ��नया�भर�म��एंट�माइ�ो�बयल�रे�सट� स�पर�वै��क�डेटा�साझा�करने�के ��लए�एक�मंच
है।��न�न�ल�खत�संगठन��म��से��कसने�इसे�लॉ�च��कया�है� ?

a) यू�नसेफ�
b) ड��यू .एच.ओ.
c) �व��आ�थ�क�मंच�
d) रेड��ॉस�सोसाइट��
Correct Answer: B
Your Answer:
Explanation

Solution (b)

�लोबल�एंट�माइ�ो�बयल�रे�सट� स��नगरानी��णाली� (�लास)

· �लोबल�एंट�माइ�ो�बयल�रे�सट� स��नगरानी��णाली� (�लास) ��नया�भर�म��एंट�माइ�ो�बयल�रे�सट� स�पर�वै��क�डेटा�साझा�करने�के ��लए�एक


मंच�है।�

· यह��व���वा��य�संगठन��ारा , �लोबल�ए�शन��लान�पर�एंट��म�ो�बयल�रे�ज़�ट� स� (एएमआर) के ���या�वयन�के �भाग�के ��प�म��शु���कया


गया�है।�

· तैयार��कए�गए�डेटा�से�रा�ीय� , �े�ीय�तथा�वै��क��नण�य�लेने� , रणनी�तय��और�प�समथ�न�को�अनु�मा�णत�करने�म��मदद��मलेगी।�

· इस��णाली�का�उ�े �य�रा�ीय�एएमआर��नगरानी��णाली�को�बढ़ावा�दे ना�है�और���नया�भर�म��भाग�लेने�वाले�दे श��के ��ारा�अ�धकृ त


एंट��म�ो�बयल���तरोध�पर�आकड़��के �सं�हण� , एक�कृ त��व�ेषण�और�मानक�करण�और�वै�करण�को�साझा�करने�के ��लए�स�म�करना�है।�

· जनसां��यक�य�और�सू�मजीव�व�ानी�सूचना�एक��त�क��जाएगी�ता�क�जनसं�या�पर�एएमआर�का��व�तार�एवं��भाव�को�समझने�के ��लए�,
�झान��पर�नजर�रखने�के ��लए�, उभरती���तरोध�का�पता�लगा�सके �और�एएमआर�को��नयं��त�करने�के ��लए�ह�त�ेप�क���भावशीलता�को
मापने�के ��लए�जोड़ा�जा�सके ।�

लाभ�

· रा�ीय�एएमआर��नगरानी�के ��लए��मता��नमा�ण�

· �नगरानी�सॉ�टवेयर�स�हत�एक�काया��वयन�पैके ज�

· डेटा�साझाकरण� , डेटा��बंधन�और��रपो�ट�ग�के ��लए�वेब -आधा�रत��लेटफ़ॉम��तक�प�ंच�

IASbaba
Web: http://ilp.iasbaba.com/ Score:
Email: ilp@iasbaba.com 4.00 / 200
Page 119
Exam Title : Test 21-Revision: Polity & ...
Email
Contact :

· �नगरानी�और�मू�यांकन�के �साथ�सहायता�

· WHO सहयोग�क� ���के �वै��क�नेटवक� �से�सहायता�

· वै��क�एएमआर���थ�त�और��वृ��य��पर��नय�मत��रपोट� �

�वचार�कर��

· एएमआर�पर��लोबल�ए�शन��लान�

· एंट�माइ�ो�बयल���तरोध�पर�साम�रक�और�तकनीक��सलाहकार�समूह� (एसट�एजी)

QUESTION 42.
�लोबल�इंटरब�क�फाइन��सयल�टे लीक�यु�नके शन�( ��व�ट) के �स�दभ��म���न�न�ल�खत�कथन��पर��वचार�क��जए :

1. यह�एक�संदेश��णाली�है , जो�ब�क���ारा���नया�भर�म��एक�सुर��त�चैनल�के �मा�यम�से�ए����ट��कए�गए��ा�प�म��जानकारी�और��नद� श


भेजने�के ��लए�उपयोग�क��जाती�है।�

2. यह�अ�पाव�धक�अंतरब�क�ऋण�के ��लए�दै �नक�धन�अंतरण�म��सहायता�करती�है।�

3. इसका�मु�यालय�बे��जयम�म��है�

उपरो��कथन��म��से�कौन�सा /से�सह��है� ?

a) 1 और�2
b) 2 और�3
c) 1 और�3
d) उपरो��सभी�
Correct Answer: C
Your Answer:
Explanation

Solution (c)

सोसाइट��फॉर�व�ड�वाइड�इंटरब�क�फाइन��शयल�टे लीक�यु�नके शंस� (��व�ट:SWIFT)

· यह�एक�सुर��त�माग� (चैनल ) के ��ारा�एक�ए����टे ड�फॉम�ट�म��जानकारी�और��नद� श�भेजने�के ��लए���नया�भर�म��ब�क��के ��ारा�उपयोग�क�


जाने�वाली�एक�संदेश��णाली�है।�

· यह�एक�संदेश��णाली�है� , तथा�यह�न�ही�मु�ा�धारण�करती�और�न�ही�खात��को��बं�धत�करती�है।�

· यह�धन�ह�तांतरण�क��सु�वधा�नह��दे ता�है� , ब��क�, यह�भुगतान�आदे श�भेजता�है� , �जसे�स�ब��धत�खात���ारा��व��थत��कया�जाना�चा�हए


, जो�सं�थान�एक��सरे�के �साथ�रखते�ह�।�

· इसका�मु�यालय�बे��जयम�म��है�

· ��व�ट�, हर�साल�एक�वा�ष�क�स�मेलन�आयो�जत�करता�है� , �जसे��सबोस�कहा�जाता�है� , जो��वशेष��प�से��व�ीय�सेवा�उ�ोग�के �उ�े �य�से


आयो�जत�होता�है।�

काय���णाली�

· मान�ली�जए��क�एक��ाहक�कहती�है�वह��क�नई��द�ली�म���टे ट�ब�क�ऑफ�इं�डया�शाखा�से�लंदन�म���सट�ब�क�शाखा�म��अपने�दो�त�को�पैसा
भेजना�चाहती�है।�

IASbaba
Web: http://ilp.iasbaba.com/ Score:
Email: ilp@iasbaba.com 4.00 / 200
Page 120
Exam Title : Test 21-Revision: Polity & ...
Email
Contact :

· नई��द�ली�म��उस�म�हला�को�एस .बी.आई. शाखा�म��अपने�दो�त�का�खाता�सं�या�और�उसके �लंदन�शाखा�के ��लए��सट�ब�क�का�अनूठा� SWI


FT कोड��दान�करना�होगा।�एसबीआई, सुर��त���व�ट�नेटवक� �पर��सट��ब�क�शाखा�को , भुगतान��ांसफर� SWIFT संदेश�भेज�दे गा।�

· �सट�ब�क�आने�वाले�भुगतान�के �बारे�म�� SWIFT संदेश��ा�त�करने�के �बाद�, इसे���लयर�करेगा�और�धन�को�लंदन�वाले��म��के �खाते�म��जम


कर�दे गा।�

�या�आपको�पता�है� ?

· एक�कोर�ब��क�ग��णाली�एक�ऐसा�सॉ�टवेयर�है�जो�दै �नक�अंतरण�तथा�आंत�रक��प�से��कसी�ब�क�के �भीतर�खात��का�समथ�न�करता�है।�यह


�ाहक��को�उस�ब�क�के ��कसी�भी�शाखा�काया�लय�से�बे�सक�लेनदे न�करने�दे ता�है।�

http://indianexpress.com/article/india/17-months-before-pnb-scandal-rbi-had-cautioned-against-
swift-abuse-at-union-bank-just-in-time-nirav-modi-5070570/

QUESTION 43.
�व��सूचना��ौ�ो�गक��कां�ेस� (ड��यू .आई.ट�.सी.) 2018, भारत�म��आयो�जत�होने�जा�रहा�है।�इसक���थापना�के �बाद�यह�पहली�बार�है ,
जब�भारत�इसे�हो�ट�करेगा।��न�न�ल�खत�म��से�कौन�सा /से�कथन� WITC के �स�दभ��म��सही�है� ?

1. इसका�उ�े �य�आईट���वशेष��� , नी�तय��और��नण�य��नमा�ता��और���नया�भर�के �सरकारी�अ�धका�रय��को�एक�साथ�एक�मंच��दान�करना


है , ता�क�वे��व�भ��चुनौ�तय��और�संभा�वत�समाधान��पर�चचा��कर�सक� ।�

2. यह�1978 से�आयो�जत�होने�वाला�एक�वा�ष�क�काय��म�है।�

नीचे�कू ट�से�चयन�क��जए :

a) के वल�1
b) के वल�2
c) दोन��1 और�2
d) न�तो�1 और�न�ही�2
Correct Answer: A
Your Answer:
Explanation

Solution (a)

�व��सूचना��ौ�ो�गक��कां�ेस� (ड��यूआईट�सी ) 2018

यह�एक���वा�ष�क�इव�ट�है�और�अपने�तरह�का�सबसे�बड़ा�आयोजन�माना�जाता�है।�इसका�उ�े �य�आईट���वशेष��� , नी�तय��और��नण�य


�नमा�ता��और���नया�भर�के �सरकारी�अ�धका�रय��को�एक�साथ�एक�मंच��दान�करना�है।�ता�क�वे��व�भ��चुनौ�तय��और�संभा�वत�समाधान�
पर�चचा��कर�सक� ।�

यह�पहली�बार�1978 म��आयो�जत��कया�गया�था�और�तब�से�हर�दो�साल�बाद�आयो�जत��कया�जाता�है।�

कहां� - हैदराबाद�

थीम�- Future Enterprises

यह�पहली�बार�है��क�भारत�ड�लूआईट�सी�क��मेजबानी�कर�रहा�है।�यह��सरी�बार�होगा��क�मले�शया�म��कु आलालंपुर�म��आयो�जत�होने�के �बाद


ए�शया�म��आयोजन�होगा।�

नासकॉम, ड�लूसीआईट��क��मेजबानी�कर�रहा�है।�

http://www.business-standard.com/article/economy-policy/modi-to-address-world-congress-on-
information-technology-in-hyderabad-today-118021800207_1.html

IASbaba
Web: http://ilp.iasbaba.com/ Score:
Email: ilp@iasbaba.com 4.00 / 200
Page 121
Exam Title : Test 21-Revision: Polity & ...
Email
Contact :

QUESTION 44.
महानद��जल��ववाद�को��व��थत�करने�के ��लए�मं��मंडल�ने�हाल�ही�म�����यूनल�के �गठन�को�मंजूरी�द��है।�यह��ववाद��कनके �म�य�है ?

a) म�य�दे श�और�छ�ीसगढ़�
b) छ�ीसगढ़�और�ओ�डशा�
c) ओ�डशा�और�झारखंड�
d) झारखंड�और�छ�ीसगढ़�
Correct Answer: B
Your Answer:
Explanation

Solution (b)

महानद��जल��ववाद��नपटाने�के ��लए����यूनल�

मं��प�रषद�ने�महानद��के �जल�को�साझा�करने�के ��लए�ओ�डशा�और�छ�ीसगढ़�के �बीच�एक�सीमा�तय�करने�के ��लए�एक����यूनल�क���थापना


को�मंजूरी�द�।�

हाइलाइट�

· स�पूण��महानद��बे�सन�म��पानी�क��सम��उपल�धता�के �आधार�पर�, ��येक�रा�य�का�योगदान� , ��येक�रा�य�म��जल�संसाधन�क��वत�मान


उपयोग�और�भ�व�य�के ��वकास�क���मता�के �आधार�पर����यूनल�को�बे�सन�रा�य��के �बीच�जल�साझाकरण�को��नधा��रत�करने�क��उ�मीद�है।�

· अंतररा�यीय�नद��जल��ववाद� (आईएसआरड��यूडी ) अ�ध�नयम�, 1956 के ��ावधान��के �अनुसार� , ���यूनल�म��भारत�के �मु�य��यायाधीश


के ��ारा, सव��च��यायालय�या�उ�च��यायालय�के ��यायाधीश��म��से�ना�मत�अ�य��और�दो�अ�य�सद�य�शा�मल�ह�गे।�

· आईएसआरड��यूडी�अ�ध�नयम� , 1956 के ��ावधान��के �अनुसार� , ���यूनल�को�तीन�साल�क��अव�ध�के �भीतर�अपनी��रपोट� �और��नण�य


��तुत�करने�क��आव�यकता�है� , �जसे�अप�रहाय��कारण��से� 2 वष��से�अ�धक�नह��बढ़ाया�जा�सकता�है।�

�वचार�कर��

· अंतररा�यीय�नद��जल��ववाद� (संशोधन ) �वधेयक�

IASbaba
Web: http://ilp.iasbaba.com/ Score:
Email: ilp@iasbaba.com 4.00 / 200
Page 122
Exam Title : Test 21-Revision: Polity & ...
Email
Contact :

http://pib.nic.in/PressReleseDetail.aspx?PRID=1521034

QUESTION 45.
वै���क���ाचार�धारणा�सूचकांक� (जी.सी.पी.आई.) म��भारत�को� 81वां��थान��दया�गया�है।�जी .सी.पी.आई. के �स�दभ��म���न�न�ल�खत�म��से
कौन�सा�कथन�सही�ह�� ?

1. इसे�ए�ने�ट��इंटरनेशनल�के ��ारा�जारी��कया�गया�है।�

2. यह�सूचकांक� 0 से� 100 के �पैमाने�का�उपयोग�करता�है� , जहां� 0 अ�यंत��व�छ�है�और� 100 अ�य�धक����है।�

नीचे�कू ट�से�चयन�क��जए :

a) के वल�1
b) के वल�2
c) दोन��1 और�2
d) न�तो�1 और�न�ही�2
Correct Answer: D
Your Answer:
Explanation

Solution (d)

वै��क���ाचार�बोध�सूचकांक�

· �ांसपेर�सी�इंटरनेशनल��ारा , जारी��कए�गए�वै��क���ाचार�बोध�सूचकांक�म��भारत�को� 81वां��थान��दया�गया�है।�

· ए�शया��शांत��े��म����ाचार�और��ेस��वतं�ता�के �मामले�म��भारत�को� "worst offenders" के ��ेणी�म��रखा�गया�है।�

· सूचकांक� 0 से� 100 के �पैमाने�का�उपयोग�करता�है� , जहां� 0 का�अथ��अ�य�धक����है�और� 100 अ�यंत��व�छ�छ�व�है।�हालां�क� , नवीनतम


र��क�ग�म��भारत�का��कोर� 40 पर, अप�रव�त�त�रहा�है।�

�वचार�कर��

· �ांसपेर�सी�इंटरनेशनल�

http://indianexpress.com/article/india/india-ranks-81st-in-global-corruption-perception-
index-5073800/

QUESTION 46.
��ल�नकल� ��त�ान�(पंजीकरण�और��व�नयमन ) अ�ध�नयम�, 2010 के �स�दभ��म���न�न�ल�खत�कथन��पर��वचार�क��जए :

1. इस�अ�ध�नयम�के �तहत, सभी���ल�नकल� ��त�ान��(सेना�को�छोड़कर ) को��वयं�को�पंजीकृ त�करने�और�सामा�य�रोग��तथा�दशा��के ��लए


मानक�उपचार��दशा�नद� श��का�एक�सेट��दान�करना�आव�यक�है।�

2. सभी�रा�य��और�संघ�रा�य��े���के ��लए, इस�अ�ध�नयम�को�लागू�करना�अ�नवाय��है।�

3. यह�अ�ध�नयम�साव�ज�नक�और��नजी��े���से���ल�नकल� ��त�ान��के �सभी��कार��(थेरापयू�टक�और�डाय�नो��टक� �कार��दोन�) पर�लागू


होता�है� , �जसम��एकल��च�क�सक (�स�गल�डॉ�टर ) �ली�नक�स�हत, �च�क�सक�क��सभी�मा�यता�ा�त��णाली�शा�मल�है।�

उपरो��कथन��म��से�कौन�सा /से�सह��है� ?

a) 1 और�2
b) 2 और�3

IASbaba
Web: http://ilp.iasbaba.com/ Score:
Email: ilp@iasbaba.com 4.00 / 200
Page 123
Exam Title : Test 21-Revision: Polity & ...
Email
Contact :

c) 1 और�3
d) उपरो��सभी�
Correct Answer: C
Your Answer:
Explanation

Solution (c)

�ली�नकल���त�ान�(पंजीकरण�और��नयमन ) अ�ध�नयम�, 2010

· क� ��सरकार��ारा�दे श�म��सभी���ल�नकल� ��त�ान��के �पंजीकरण�और��व�नयमन�के ��लए�अ�ध�नय�मत��कया�गया�है�ता�क�उनके ��ारा


उपल�ध�कराए�जाने�वाले�सु�वधा��और�सेवा��के ��यूनतम�मानक��को��नधा��रत��कया�जा�सके ।�

· इस�अ�ध�नयम�के �तहत, सभी���ल�नकल� ��त�ान��को��वयं�को�र�ज�टर�करने�और�सामा�य�रोग��और�दशा��के ��लए�मानक�उपचार


�दशा�नद� श��का�एक�सेट��दान�करना�आव�यक�है।�

· इस�अ�ध�नयम�को�भारत�के �सभी�रा�य��पर�सीधे�लागू�नह���कया�जा�सकता�है।�रा�य��को�एक���ताव�पा�रत�कर�इसे�अपनाने�या�इसी�तरह
का�एक��वधेयक�पा�रत�करने�का��वक�प��दया�गया�है।�

· अपवाद�व�प�सै�य�बल��के �तहत�आने�वाले���त�ान��के �अलावा�, आयुष���त�ान��स�हत�सभी�साव�ज�नक�और��नजी���त�ान��को


पंजीकरण�कराना�आव�यक�है।�

· यह�अ�ध�नयम�साव�ज�नक�और��नजी��े���से���ल�नकल� ��त�ान��के �सभी��कार��(थेरापयू�टक�और�डाय�नो��टक� �कार��दोन�) पर�लागू


होता�है� , �जसम��एकल��च�क�सक (�स�गल�डॉ�टर ) �ली�नक�स�हत, �च�क�सक�क��सभी�मा�यता�ा�त��णाली�शा�मल�है।�

· यह�अ�ध�नयम, नेशनल�काउं�सल�फॉर���ल�नकल�ए�टा��ल�म�ट�नामक�एक�प�रषद��नकाय�क���थापना�क���व�था�करता�है , जो��क


नैदा�नक� ��त�ान�के ��ारा�तथा��यूनतम�मानक��और�उनके �आव�धक�समी�ा�को��वक�सत�करने�के ��ारा�उ�चत��वा��य�दे खभाल�सु�न��त
करने�के ��लए�मानक��था�पत�करने�के ��लए��ाथ�मक��प�से��ज�मेदार�है।�

�या�आप�जानते�है� ?

· ��ल�नकल���त�ान�(पंजीकरण�और��व�नयमन ) अ�ध�नयम�, 2010 चार�रा�य�- अ�णाचल��दे श� , �हमाचल��दे श� , �मजोरम�और


�स��कम�तथा��द�ली�के �अलावा�सभी�संघ�रा�य��े���म�� 1 माच�� 2012 से�लागू�ह�।�

· उ�र��दे श� , उ�राखंड� , राज�थान�, �बहार�, झारखंड�और�असम�ने�सं�वधान�के �अनु�छे द� 252 के �खंड� (1) के �तहत�इस�अ�ध�नयम�को
अपनाया�है।�

· सं�वधान�का�अनु�छे द� 47 साव�ज�नक��वा��य�म��सुधार�लाने�के �उ�े �य�से�रा�य�पर�एक��ज�मेदारी�आरो�पत�करता�है�तथा��वशेष��प�से


अपनी��ाथ�मक�कत����के ��प�म��इस��ज�मेदारी�को��यान�म��रखेगा� , रा�य�नशीली�पेय�और�दवा��जो��वा��य�के ��लए�हा�नकारक�ह� , के
औषधीय�उ�े �य��को�छोड़कर�इसके �उपभोग�पर���तबंध�लगाने�का��यास�करेगा।�

�वचार�कर��

· सं�वधान�का�अनु�छे द� 252

http://www.livemint.com/Politics/jVlkSNP683orAm5qDe4POK/Complaints-over-medical-bills-
Centre-seeks-states-response.html

QUESTION 47.
राजनी�तक�पाट��के ��व�पोषण�को��व�नय�मत�करने�के ��लए�चुनावी (इले�टोरल ) बांड�योजना�शु��क��गई�है।�इले�टोरल�बांड�के �स�दभ��म���न�न
�ल�खत�म��से�कौन�सा�कथन�सही�ह�� ?

1. यह�उस�����के ��ारा�खरीदा�जा�सकता�ह�� , जो�भारत�का�नाग�रक�है�या�भारत�म���नग�मत�या��था�पत�है।�

2. एक�पा��राजनी�तक�दल�के ��ारा�के वल�अ�धकृ त�ब�क�के �साथ�एक�ब�क�खाते�के �मा�यम�से�ही�बॉ�ड�को��ा�त��कया�जाएगा।�

IASbaba
Web: http://ilp.iasbaba.com/ Score:
Email: ilp@iasbaba.com 4.00 / 200
Page 124
Exam Title : Test 21-Revision: Polity & ...
Email
Contact :

3. इले�टोरल�बॉ�ड , जारी�होने�क��तारीख�से�प��ह��दन��के ��लए�वैध�होगा�और�वैधता�अव�ध�क��समा��त�के �बाद�बॉ�ड�जमा�होने�पर��कसी�


भी�आदाता�पॉ�लसी�पाट��को�कोई�भुगतान�नह���कया�जाएगा।�

नीचे�कू ट�से�चयन�क��जए :

a) 1 और�2
b) 2 और�3
c) 1 और�3
d) उपरो��सभी�
Correct Answer: D
Your Answer:
Explanation

Solution (d)

इले�टोरल�बॉ�ड��क�म� 2018

· इले�टोरल�बॉ�ड��कसी������ारा�खरीदे �जा�सकते�ह�� , जो�भारत�का�नाग�रक�है�या�भारत�म���नग�मत�या��था�पत�है।�

· एक�����या�तो�एकल��प�से�या�संयु���प�से�अ�य����य��के �साथ�इले�टोरल�बांड�खरीद�सकता�है।�जन��त�न�ध�व�अ�ध�नयम� , 195


1 (1951 के �43) क��धारा�29A के �तहत�के वल�पंजीकृ त�राजनी�तक�दल�और�जो��क�लोकसभा�या�रा�य��वधानसभा�के ��वगत�आम�चुनाव
म��मतदान��कए�गए�वोट��का�कम�से�कम�एक���तशत��ा�त�करते�ह�� , चुनाव�बांड��को��ा�त�करने�के ��लए�पा��ह�गे।�

· एक�पा��राजनी�तक�दल�के ��ारा�के वल�अ�धकृ त�ब�क�के �साथ�ब�क�खाते�के �मा�यम�से�ही�बॉ�ड�को��ा�त��कया�जाएगा।�

· यह�नोट��कया�जा�सकता�है��क��नवा�चन�बॉ�ड�जारी�होने�क��तारीख�से�प��ह��दन��के ��लए�वैध�होगा�और�वैधता�अव�ध�समा�त�होने�के �बाद


बॉ�ड�जमा�होने�पर��कसी�भी�आदाता�पॉ�लसी�पाट��को�कोई�भुगतान�नह���कया�जाएगा।�

· �कसी�भी�यो�य�राजनी�तक�दल��ारा�उसके �खाते�म��जमा�बांड�उसी��दन�जमा��कया�जाएगा।�

· चुनावी�बॉ�ड�क��पहली��ब��� 01 माच�� 2018 से�शु��होकर� 10 �दन��क��अव�ध�के ��लए�10 माच�� 2018 तक�होगी�

�वचार�कर��

· हा�लया�चुनाव�सुधार�

QUESTION 48.
सऊद��अरब�क��यह�नी�त�उसके ��म�मं�ालय��ारा�लागू�क��गई�है� , �जसम��सऊद��कं प�नय��और�उ�म��को�अपने�कम�चा�रय��के �कु छ�भाग�के
�प�म��सउद��नाग�रक��को��नयो�जत�करना�आव�यक�है ?

a) �नताकत��णाली�
b) �लयाकत��णाली�
c) �हमाकत��णाली�
d) नज़ाकत��णाली�
Correct Answer: A
Your Answer:
Explanation

Solution (a)

�नताकत��णाली�

सऊद�करण�, आ�धका�रक�तौर�पर�सऊद��रा�ीयकरण�योजना�या�अरबी�म���नताकत��णाली� , �म�मं�ालय��ारा�लागू�सऊद��अरब�क��नी�त�है�


, �जसम��सऊद��कं प�नय��और�उ�म��को�अपने�कम�चा�रय��के �कु छ�भाग�के ��प�म��सउद��नाग�रक��को��नयो�जत�करना�आव�यक�है।�

IASbaba
Web: http://ilp.iasbaba.com/ Score:
Email: ilp@iasbaba.com 4.00 / 200
Page 125
Exam Title : Test 21-Revision: Polity & ...
Email
Contact :

QUESTION 49.
इ�कडना�सु�ढ़�होते�ह��तथा��भ� -�भ��वातावरण�म��पाए�जाते�ह�।��न�न�ल�खत�म��से�कौन�सा /से�कथन�इ�कडना�के �स�दभ��म��सही�ह�� ?

1. आम�तौर�पर�अ�य��तनपाय��क��तुलना�म��इनके �शरीर�का�तापमान�ब�त�कम�होता�ह�।�

2. इनके �प��पाद�पीछे �क��ओर�से�नुक�ले�होते�ह��जो�उनको��बल /मांद�बनाने�म��मदद�करते�ह�।�

3. ये�अंडे�दे ते�ह� , ले�कन�अपने�ब�च��को�मां�क��थैली�म��रखते�ह�।�

नीचे�कू ट�से�चयन�क��जए :

a) 1 और�2
b) 2 और�3
c) 1 और�3
d) उपरो��सभी�
Correct Answer: D
Your Answer:
Explanation

Solution (d)

इ�कडना�

· इ�कडना�सु�ढ़�होते�ह��तथा�रे�ग�तान�से�लेकर�बफ़�ले��े���तक�अलग -अलग�वातावरण�म��पाए�जाते�ह�� , अ�य�सभी��तनधा�रय��के �मुकाबले


इनके �शरीर�का�तापमान�ब�त�कम- लगभग�30C होता�है , एक��दन�म��इसम�� 10C तक�का�उतारचढ़ाव�हो�सकता�है।�

· इ�कडना�वृहद��री�क��या�ा�कर�सकते�ह� - सामा�यत: एक��दन�कई��कलोमीटर�तक�

· इ�कडना�अंडे�दे ते�ह�� , ले�कन�इन�न�ह� -पु�गल�को�माँ�के �पाउच�म��रखते�ह��

· इ�कडना�के �प��पाद�पीछे �क��से�नुक�ले�होते�ह��जो�उनको�ग�े�करने�म��मदद�करते�ह��

· वे�ऑ��े �लया�और��यू��गनी�जैसे��थान��म��पाए�जाते�ह��

· इ�कडना���नया�के �सबसे�पुराने�जी�वत��तनधा�रय��म��से�एक�ह��

· वे��माट� �ह�� , �जनम��सभी��तनधा�रय�� (मनु�य��स�हत ) के �शरीर�के �आकार�के �संबंध�म��सबसे�बड़ा��ं टल�कॉट� �स�होता�है।�

�वचार�कर��

· �ले�टपस�

http://www.thehindu.com/todays-paper/tp-in-school/whats-threatening-echidnas/
article22830002.ece

QUESTION 50.
�मलन�अ�यास�के �स�दभ��म���न�न�ल�खत�म��से�कौन�सा�कथन�सही�है� ?

1. यह�भारतीय�नौसेना�के ��ारा�आयो�जत�होने�वाला�समु���नौसेना�का�एक�समागम�है।�

2. यह�अंडमान�और��नकोबार���प�समूह�म��आयो�जत��कया�जा�रहा�है।�

3. यह�हर�पांच�साल��म��एक�बार�आयो�जत��कया�जाता�है।�

नीचे�कू ट�से�चयन�क��जए :

IASbaba
Web: http://ilp.iasbaba.com/ Score:
Email: ilp@iasbaba.com 4.00 / 200
Page 126
Exam Title : Test 21-Revision: Polity & ...
Email
Contact :

a) 1 और�2
b) 2 और�3
c) 1 और�3
d) उपरो��सभी�
Correct Answer: A
Your Answer:
Explanation

Solution (a)

�मलन�2018

· यह�अंडमान�और��नकोबार���प�समूह�म��भारतीय�नौसेना��ारा�दो�वष��म��आयो�जत�होने�वाला�समु���नौसेना�का�एक�समागम�है।�

· यह�अंडमान�और��नकोबार���प�समूह�म��आयो�जत��कया�जा�रहा�है।�

· अंडमान��नकोबार�कमान�के �त�वावधान�म��भारतीय�नौसेना��ारा�इस�काय��म�का�आयोजन��कया�जा�रहा�है।�

· यह�पहली�बार�1995 म��आयो�जत��कया�गया�था�और�चार�समु���नौसेना��क��भागीदारी�दे खी�गई�थी।�

· �वषय�- ‘�� ड�शप�अ�ॉस�द�सी�'

· इस�साल�के ��मलन�इंटरनेशनल�मैरीटाइम�से�मनार�का��वषय�है� ' इन�परसूट�ऑफ़�मैरीटाइम�गुड�ऑड�र - �ापक�जानकारी�साझाकरण


उपकरण�क��आव�यकता�'

· सहभागी�दे श� - ऑ��े �लया� , मले�शया� , मालद�व�, मॉरीशस�, �यांमार� , �यूजील�ड� , ओमान�, �वयतनाम�, थाईल�ड� , तंजा�नया� , �ीलंका� ,
�स�गापुर� , बां�लादे श� , इंडोने�शया� , के �या�और�कं बो�डया।�

�वचार�कर��

· आई.ओ.आर.-ए.आर.सी�.

http://www.thehindu.com/news/national/navy-to-host-milan-from-march-6-to-13/
article22852221.ece

QUESTION 51.
रा�प�त�का���य���नवा�चन�नह��होता�ह�� , पर�तु��नवा�चक�म�डल (इले�टोरल�कॉलेज ) के �सद�य��के ��ारा��कया�जाता�है।�म�डल�म��शा�मल
होते�ह� ?

1. संसद�के �दोन��सदन��के ��नवा��चत�सद�य।�

2. रा�य��वधानम�डल�के ��नवा��चत�सद�य।�

3. क� ��शा�सत��दे श��के ��वधानम�डल��के ��नवा��चत�सद�य।�

नीचे��दए�गए�कू ट�से�सही�उ�र�चु�नए ?

a) 1 और�2
b) 2 और�3
c) 1 और�3
d) 1,2 और�3
Correct Answer: C
Your Answer:
Explanation

IASbaba
Web: http://ilp.iasbaba.com/ Score:
Email: ilp@iasbaba.com 4.00 / 200
Page 127
Exam Title : Test 21-Revision: Polity & ...
Email
Contact :

Solution (c)

रा�प�त�का��नवा�चन�जनता�के ��ारा���य���प�से�नह���कया�जाता�ह�� , ब��क�एक��नवा�चक�मंडल�के �सद�य��के ��ारा�उसका��नवा�चन��कया


जाता�है :

· संसद�के �दोन��सदन��के ��नवा��चत�सद�य�

· रा�य��वधानसभा��के ��नवा��चत�सद�य�

· संघ�शा�सत��दे श���द�ली�और�पुडुचेरी�के ��वधानसभा��के ��नवा��चत�सद�य�

इस��कार�, संसद�के �दोन��सदन��के �मनोनीत�सद�य�, रा�य��वधानसभा��के �मनोनीत�सद�य�, रा�य��वधान�प�रषद��के �सद�य�(दोन�


�नवा��चत�और�मनोनीत ) तथा��द�ली�और�पुडुचेरी��वधानसभा��के �मनोनीत�सद�य, रा�प�त�के �चुनाव�म��भाग�नह��लेते।�

नोट: रा�य��वधानमंडल�म��रा�यपाल� , �वधानसभा�और��वधान�प�रषद�शा�मल�ह�।�

�वचार�कर� !

· रा�प�त�के ��नवा�चन�क��री�त�

· रा�प�त�के �पद�क��अह�ताएं�

QUESTION 52.
सं�वधान�म���कया�गया��न�न�ल�खत�म��से�कौन�सा�प�रवत�न , अनु�छे द� 368 के �दायरे�से�बाहर�ह�� ?

1. नए�रा�य��का��वेश�या��थापना�

2. संसद�म��रा�य��का���त�न�ध�व�

3. संसद�के �चुनाव�म��प�रवत�न�

4. पांचव��अनुसूची�के �अंतग�त�अनुसू�चत��े���म��प�रवत�न�

नीचे��दए�गए�कू ट�से�सह��उ�र�चु�नए ?

a) 1,3 और�4
b) 1,2 और�3
c) 2 और�4
d) 1,2 ,3 और�4
Correct Answer: A
Your Answer:
Explanation

Solution (a)

सं�वधान�के �कई��ावधान�संसद�के �दोन��सदन��क��साधारण�ब�मत�से�संशो�धत��कए�जा�सकते�ह�।�ये��व�थाएं�अनु�छे द� 368 क��सीमा�से


बाहर�ह�।�इन��ावधान��म��शा�मल�ह� :

· नए�रा�य��का��वेश�या��थापना।�

· संसद�और�रा�य��वधानसभा��के �चुनाव�

· पांचव��अनुसूची -अनुसू�चत��े���और�अनुसू�चत�जनजा�तय��का��शासन�।�

IASbaba
Web: http://ilp.iasbaba.com/ Score:
Email: ilp@iasbaba.com 4.00 / 200
Page 128
Exam Title : Test 21-Revision: Polity & ...
Email
Contact :

संसद�म��रा�य��के ���त�न�ध�व�म��संशोधन , संसद�के ��वशेष�ब�मत�है�और�साथ�ही�आधे�रा�य��के ��वधानसभा��के �साधारण�ब�मत��ारा


सहम�त�के �साथ��कया�जाता�है।�

�या�आप�जानते�है� ?

· साधारण��वधेयक�के ��वपरीत�, सं�वधान�संशोधन��वधेयक�म��ग�तरोध�के �मामले�म��दो�सदन��क��संयु��बैठक�आयो�जत��कये�जाने�का�कोई


�ावधान�नह��है।�

�वचार�कर� !

· ब�मत�के ��कार�

· भारतीय�सं�वधान�न�तो�कठोर�और�न�ही�लचीला�है� , ले�कन�दोन��का�संतुलन�है�

· मूल�ढांचे�का��स�ांत�

QUESTION 53.
भारतीय�सं�वधान�के �अनुसार� , पया�वरण�संर�ण�इनम��से��कसका��ह�सा�है ?

1. मूल�अ�धकार�

2. रा�य�के �नी�त��नद� शक��स�ांत�

3. मूल�कत���

नीचे��दए�गए�कू ट�से�सह��उ�र�चु�नए :

a) 1 और�2
b) 2 और�3
c) 1 और�3
d) 1,2 और�3
Correct Answer: D
Your Answer: Unanswered
Explanation

Solution (d)

सं�वधान�के �अनु�छे द� 21 के �अनुसार� , " �कसी�����को�उसके ��ाण�या�दै �हक��वतं�ता�से��व�ध��ारा��था�पत����या�के �अनुसार�ही�वं�चत


�कया�जायेगा�अ�यथा�नह� "

मेनका�गांधी�बनाम�यू�नयन�ऑफ�इं�डया�के �अनुसार� , सव��च��यायालय�ने�अनु�छे द� 21 के �भाग�के ��प�म���व�थ�पया�वरण�का�अ�धकार�को


शा�मल��कया�है।�

रा�य�नी�त�के ��नद� शक��स�ांत��के �तहत�सं�वधान�के �अनु�छे द� 48-ए�म��कहा�गया�ह�� , " रा�य�पया�वरण�का�संर�ण��एवं�संवध�न�तथा�वन�एवं


व�य�जीव��क��सुर�ा�का��यास�करेगा "।�

मूल�कत����के �तहत�, अनु�छे द� 51-ए(जी) म��कहा�गया�ह�� , " वन��, झील��, न�दय��और�व�य�जीव��स�हत��ाकृ �तक�पया�वरण�क��सुर�ा�और
सुधार�करना�तथा��ा�णमा��के ���त�दया�भाव�रखना, भारत�के ���येक�नाग�रक�का�यह�कत���होगा। "

अ�धक�जानकारी�के ��लए:

http://pib.nic.in/newsite/PrintRelease.aspx?relid=105411

�वचार�कर� !

IASbaba
Web: http://ilp.iasbaba.com/ Score:
Email: ilp@iasbaba.com 4.00 / 200
Page 129
Exam Title : Test 21-Revision: Polity & ...
Email
Contact :

· मेनका�गांधी�वाद�और�संवैधा�नक��ा�या�पर�इसके ��न�हताथ��

· भारत�म��पया�वरण�संर�ण�कानून�

QUESTION 54.
अंतरा��यीय�प�रषद् इं(टर��टे ट�काउं�सल ) के �संदभ��म���न�न�ल�खत�कथन��पर��वचार�क��जए ?

1. यह�एक�संवैधा�नक��नकाय�है।�

2. सरकार��के �बीच��कसी�भी�कानूनी��ववाद�के �मामले�म�� , प�रषद�का��नण�य�अं�तम�और�बा�यकारी�होता�है।�

3. सं�वधान�के वल�अ�खल�भारतीय�प�रषद��के ��नमा�ण�क��अनुम�त�दे ता�है , तथा��े�ीय�प�रषद��के ��लए�नह�।�

ऊपर��दए�गए�कथन��म��से�कौन�सा /से�सही�है� ?

a) के वल�1
b) 1 और�2
c) के वल�3
d) 1,2 और�3
Correct Answer: A
Your Answer:
Explanation

Solution (a)

अनु�छे द� 263, रा�य��के �म�य�तथा�क� ��और�रा�य��के �म�य�सम�वय�के ��लए�एक�अंतरा��यीय�प�रषद�के �गठन�क���व�था�करता�है।�

प�रषद, �कसी�भी��ववाद�चाहे�वो�कानूनी�अथवा�गैर�कानूनी�हो� , का��न�पादन�कर�सकता�है�ले�कन�इसका�काय��सलाहकारी�है�न��क��यायालय


क��तरह�अ�नवाय���प�से��नण�य�दे ता�है।�

�ब���कर�हेतु�उ�री� , पूव�� , प��मी�और�द��णी��े��के ��लए�चार��े�ीय�प�रषद� �पहले�ही�इस�अनु�छे द�के �तहत�रा�प�त��ारा�ग�ठत�क��जा


चुक��है।�

�या�आप�जानते�है� ?

· सरका�रया�आयोग�क���सफा�रश��पर�अनु�छे द� 263 के �अंतग�त� 1990 म���थायी�अंतरा��यीय�प�रषद�का�गठन��कया�गया�था।�

�वचार�कर� !

· सहकारी�संघवाद�म��अंतरा��यीय�प�रषद�क��भू�मका�

QUESTION 55.
सं�वधान�भारत�को� ' रा�य��का�संघ� ' के ��प�म��व�ण�त�करता�है��य��क�

1. यह�संघ (फे डरेशन ), रा�य��के �बीच�समझौते�का�प�रणाम�है।�

2. रा�य��को, संघ (फे डरेशन ) से�अलग�होने�का�कोई�अ�धकार�नह��है।�

नीचे��दए�गए�कू ट�से�सही�उ�र�चु�नए ?

a) के वल�1
b) के वल�2
c) दोन��1 और�2

IASbaba
Web: http://ilp.iasbaba.com/ Score:
Email: ilp@iasbaba.com 4.00 / 200
Page 130
Exam Title : Test 21-Revision: Polity & ...
Email
Contact :

d) न�तो�1 और�न�ही�2
Correct Answer: B
Your Answer:
Explanation

Solution (b)

सं�वधान�के �अनु�छे द� 1 म��भारत�को� ' रा�य��का�संघ� ' कहा�गया�है।�

डॉ. बी. आर. अ�बेडकर�के �अनुसार� , ' रा�य��का�संघ� ' उ���को�संघीय�रा�य�के ��थान�पर�मह�व�दे ने�के �दो�कारण�है :

1. भारतीय�संघ , अमे�रक��संघ�के �समान�रा�य��के �बीच�समझौते�का�प�रणाम�नह��है।�

2. रा�य��को�संघ�से�अलग�होने�का�कोई�अ�धकार�नह��है।�

�या�आप�जानते�है� ?

· एका�मक�क��और�झुके �होने�के �कारण�भारतीय�संघ�को�कभी -कभी�अध� -संघीय�भी�कहा�जाता�है।�

�वचार�कर� !

· भारतीय�सं�वधान�क��एका�मक�और�संघीय��वशेषताएं�

· एस�आर�बो�माई�बनाम�यू�नयन�ऑफ�इं�डया�

QUESTION 56.
सु�ीम�कोट� �के �अनुसार� , �न�न�ल�खत�अ�धकार��म��से�कौन�से�अ�धकार , वाक् �एवं�अ�भ����क���वतं�ता�का�अ�धकार�के �भाग�है� ?

1. �ेस�क���वतं�ता�

2. सरकारी�ग�त�व�धय��के �बारे�म��जानने�का�अ�धकार�

3. शां�त�का�अ�धकार�

4. हड़ताल�का�अ�धकार�

नीचे��दए�गए�कू ट�से�सही�उ�र�चु�नए ;

a) 1 और�3
b) 1,2 और�3
c) 2,3 और�4
d) 1,2 और�4
Correct Answer: B
Your Answer:
Explanation

Solution (b)

उ�चतम��यायालय�के �अनुसार�वाक् �एवं�अ�भ����क���वतं�ता�म���न�न�ल�खत�शा�मल�ह� :

1. अपने�या��कसी�अ�य�के ��वचार��को��चा�रत�करने�का�अ�धकार�

2. �ेस�क���वतं�ता�

3. वा�ण��यक��व�ापन��क���वतं�ता�

IASbaba
Web: http://ilp.iasbaba.com/ Score:
Email: ilp@iasbaba.com 4.00 / 200
Page 131
Exam Title : Test 21-Revision: Polity & ...
Email
Contact :

4. टे लीफ़ो�नक�वाता�लाप�के �टै �प�ग�के ��व���

5. �सा�रत�करने�का�अ�धकार� , अथा�त� , सरकार�का�इले��ॉ�नक�मी�डया�पर�कोई�एका�धकार�नह��है�

6. एक�राजनै�तक�पाट��या�संगठन��ारा�आयो�जत�बंद�के ��खलाफ�अ�धकार�

7. सरकारी�ग�त�व�धय��के �बारे�म��जानने�का�अ�धकार�

8. शां�त�का�अ�धकार�

9. अखबार�पर�पूव��स�सर�शप�लगाने�के ��खलाफ।�

10. �दश�न�या�धरने�का�अ�धकार� , ले�कन�हड़ताल�करने�का�अ�धकार�नह��है।�

�या�आप�जानते�है� ?

· य��प�भारतीय�सं�वधान�एसो�सएशन�या���मक�संघ��के �गठन�क��गारंट��का�मूल�अ�धकार��दान�करता�है�ले�कन�हड़ताल�पर�जाने�का�कोई
मौ�लक�अ�धकार�नह��है।�वा�तव�म�� , यह�औ�ो�गक��ववाद�अ�ध�नयम�, 1947 के �तहत�एक�सशत��अ�धकार�है� , और�कु छ�पूव� -शत��पूरी�होने
के �बाद�ही�उपल�ध�होता�है।�

�वचार�कर� !

· साइबर��न�दा�बनाम�वाक् �एवं�अ�भ����क���वतं�ता�

· राज�ोह�कानून�बनाम�वाक् �एवं�अ�भ����क���वतं�ता�

QUESTION 57.
भारतीय�सं�वधान�म��मूल�कत����के �संदभ��म���न�न�ल�खत�कथन��पर��वचार�क��जए ?

1. यह�जापानी�सं�वधान�से��े�रत�ह�।�

2. यह�के वल�नाग�रक��पर�ही�लागू�होता�ह�।�

3. रा�य�नी�त�के ��नद� शक��स�ांत��के ��वपरीत�, ये�गैर -�यायो�चत�ह�।�

ऊपर��दए�गए�कथन��म��से�कौन�सा /से�सही�है� ?

a) 1 और�2
b) के वल�2
c) 2 और�3
d) 1,2 और�3
Correct Answer: B
Your Answer:
Explanation

Solution (b)

भारतीय�सं�वधान�म��मूल�कत����को�पूव��सो�वयत�संघ�के �सं�वधान�से��भा�वत�होकर��लया�गया�है।�

मूल�कत����को�के वल�नाग�रक��तक�ही�सी�मत�रखा�जाता�है�और��वदे �शय��तक��व�तार�नह��होता�है।�

�नद� शक��स�ांत��क��तरह� , मौ�लक�कत���भी�गैर -�यायसंगत�ह�।�

�वचार�कर� !

IASbaba
Web: http://ilp.iasbaba.com/ Score:
Email: ilp@iasbaba.com 4.00 / 200
Page 132
Exam Title : Test 21-Revision: Polity & ...
Email
Contact :

· �वण���स�ह�स�म�त�क���सफा�रश�

· मूल�कत����क��आलोचना�

QUESTION 58.
�न�न�ल�खत�म��से�कौन�सा /से�रा�ीय�आपातकाल�क��घोषणा�का /के �प�रणाम�है� ?

1. क� ���कसी�भी�मामले�पर�रा�य�को��नद� श�दे �सकता�है।�

2. रा�य�के ��वधान-स�ब�धी�श���को��नलं�बत�कर��दया�जाता�है।�

3. आपातकाल�के �संचालन�के �दौरान�रा�प�त�एक�अ�यादे श�जारी�नह��कर�सकता�ह�।�

नीचे��दए�गए�कू ट�से�सही�उ�र�चु�नए ?

a) के वल�1
b) 1 और�2
c) 2 और�3
d) के वल�3
Correct Answer: A
Your Answer:
Explanation

Solution (a)

रा�ीय�आपातकाल�के �दौरान�क� ���कसी�भी�मामले�पर�रा�य�को�काय�कारी��नद� श�दे ने�का�हकदार�हो�जाता�है।�इस��कार� , रा�य�सरकार��को


क� ��के �पूण���नयं�ण�म��लाया�जाता�है� , हालां�क�वे��नलं�बत�नह��होते�ह�।�

रा�ीय�आपातकाल�के �दौरान�संसद�को�रा�य�सूची�म��उ��ल�खत��कसी�भी��वषय�पर�कानून�बनाने�का�अ�धकार��ा�त�हो�जाता�है।�हालां�क
रा�य��वधान�मंडल��को��नलं�बत�नह���कया�जाता� , ले�कन�सं�वधान�संघा�मक�क��जगह�एका�मक�हो�जाता�है।�

जब�रा�ीय�आपातकाल�क��घोषणा�लागु�है� , और�संसद�स��म��नह��है� , तो�रा�प�त�रा�य�के ��वषय��पर�भी�अ�यादे श�जारी�कर�सकता�है।�

�वचार�कर� !

· मूल�अ�धकार��पर�रा�ीय�आपातकाल�का��भाव�

· रा�प�त�शासन�और�रा�ीय�आपातकाल�के �म�य�अंतर�

QUESTION 59.
सं�वधान�के �अनुसार� , एक�����को�संसद�सद�य�के ��प�म���नवा��चत�होने�के ��लए�अयो�य�घो�षत��कया�जाएगा�य�द�

1. वह�लाभ�का�पद�धारण�करता�है।�

2. वह�भारत�का�नाग�रक�नह��है।�

3. वह�चुनाव�अपराध��का�दोषी�पाया�जाता�है।�

4. अ�पृ�यता�के ��सार�के ��लए�उ�ह��दं �डत��कया�गया�है।�

नीचे��दए�गए�कोड�का�सही�उ�र�चुन��

a) 1 और�2
b) 1 और�3

IASbaba
Web: http://ilp.iasbaba.com/ Score:
Email: ilp@iasbaba.com 4.00 / 200
Page 133
Exam Title : Test 21-Revision: Polity & ...
Email
Contact :

c) 2 और�4
d) 3 और�4
Correct Answer: A
Your Answer:
Explanation

Solution (a)

सं�वधान�के �तहत�, �कसी�����को�संसद�के �सद�य�के ��प�म���नवा��चत�होने�के ��लए�अयो�य�घो�षत��कया�जाएगा�, य�द:

1. वह�क� ��या�रा�य�सरकार�के �अधीन�कोई�लाभ�का�पद�धारण�करता�है� (संसद��ारा�तय�कोई�पद�या�मं�ी�पद�को�छोड़कर )।�

2. वह��वकृ त�च��है�और��यायालय�ने�ऐसी�घोषणा�क��है।�

3. वह�एक�घो�षत��दवा�लया�है।�

4. य�द�वह�भारत�का�नाग�रक�नह��है�या��वे�छा�से��कसी��वदे शी�रा�य�क��नाग�रकता�हा�सल�कर�चुका�है�या��कसी��वदे शी�रा�य�के ���त��न�ा


को�अ�भ�वीकार��कये��ए�है।�

5. य�द�वह�संसद��ारा�बनाई�गई��कसी��व�ध��ारा��नर�ह�त��कया�गया�है।�

चुनावी�अपराध�या�चुनाव�म�����आचरण�के �तहत�दोषी�पाए�जाने�का�मानदं ड� , जन���त�न�ध�व�अ�ध�नयम�(1951) के �तहत�अपा��ह�।�

�या�आप�जानते�है� ?

· लाभ�का�पद�धारण�करना, रा�प�त�, उपरा�प�त�या�संसद�के ��कसी�भी�सद�य�के ��नह�रता�के ��लए�एक�मह�वपूण��मानदं ड�है� , ले�कन�इसे


सं�वधान�या��जन���त�न�ध�व�अ�ध�नयम� (1951) के �तहत�प�रभा�षत�नह���कया�गया�है।�

�वचार�कर� !

· दलबदल�के �आधार�पर��नह�रता�

· जन��त�न�ध�व�अ�ध�नयम�1951

QUESTION 60.
�न�न�ल�खत�मामल��म��से�कौन�सा /से��े��सव��च��यायालय�का /के �मूल��े�ा�धकार�नह��है� ?

1. दो�रा�य��के �म�य��ववाद�

2. पूव��संवैधा�नक�सं�ध�से�उ�प���ववाद�

3. अंतररा�यीय�जल��ववाद�

नीचे��दए�गए�कोड�का�सही�उ�र�चुन��

a) के वल�2
b) 2 और�3
c) के वल�1
d) के वल�3
Correct Answer: B
Your Answer:
Explanation

Solution (b)

IASbaba
Web: http://ilp.iasbaba.com/ Score:
Email: ilp@iasbaba.com 4.00 / 200
Page 134
Exam Title : Test 21-Revision: Polity & ...
Email
Contact :

�न�न�ल�खत��ववाद��के �स�दभ��म��सु�ीम�कोट� �के �पास�अन�य�मूल��े�ा�धकार�है :

I. क� ��व�एक�या�एक�से�अ�धक�रा�य��के �बीच�ह��

II. क� ��और�कोई�रा�य�या�रा�य��का�एक�तरफ�होना�एवं�एक�या�एक�से�अ�धक�रा�य��का��सरे�तरफ�होना�

III. दो�या�दो�से�अ�धक�रा�य��के �बीच�

पूव� -संवैधा�नक�सं�ध�से�उ�प��होने�वाले��ववाद��पर�उ�चतम��यायालय�का�कोई�मूल��े�ा�धकार�नह��है।�

�या�आप�जानते�है� ?

· इंटर��टे ट�वॉटर��ड��यूट्स�ए�ट� , 1956 म��अंतररा�यीय�जल��ववाद��पर�सव��च��यायालय�को�अ�धकार��े��से�बाहर�रखा�गया�है।�हालां�क�


, �व-घोषणा�से� , उ�चतम��यायालय�ने�खुद�को�ऐसे�मामल��पर�अपीलीय��े�ा�धकार��दान��कया�है।�

�वचार�कर� !

· अपीलीय��या�यक��े��

· �पेशल�लीव�पे�टशन�

QUESTION 61.
रा�य�लोक�सेवा�आयोग�के �अ�य��को��कसके ��ारा��नयु���कया�जाता�है ?

a) रा�प�त�
b) �धानमं�ी�
c) रा�यपाल�
d) कै �बनेट�स�चव�
Correct Answer: C
Your Answer:
Explanation

Solution (c)

रा�यपाल, रा�य�लोक�सेवा�आयोग�के �अ�य��और�सद�य��को��नयु��करता�है।�

हालां�क� , उ�ह��के वल�रा�प�त��ारा�हटाया�जा�सकता�है� , न��क�गवन�र��ारा�।�

�वचार�कर� !

· रा�यपाल�क��काय�कारी�श��यां�

QUESTION 62.
भारत�म���या�यक�समी�ा�के �संदभ��म���न�न�ल�खत�कथन��पर��वचार�क��जए ?

1. यह�के वल��वधायी�काय��के ��लए��कया�जा�सकता�है�तथा�काय�कारी�काय��के ��लए�नह�।�

2. ' �या�यक�समी�ा�' उ���का�उ�लेख , अनु�छे द� 13 म���कया�गया�है।�

3. नौव��अनुसूची , �या�यक�समी�ा�के �दायरे�से�बाहर�है।�

ऊपर��दए�गए�कथन��म��से�कौन -सा�सही�है� ?

a) के वल�1

IASbaba
Web: http://ilp.iasbaba.com/ Score:
Email: ilp@iasbaba.com 4.00 / 200
Page 135
Exam Title : Test 21-Revision: Polity & ...
Email
Contact :

b) के वल�2
c) के वल�3
d) उपरो��म��से�कोई�नह��
Correct Answer: D
Your Answer:
Explanation

Solution (d)

ज��टस�सैयद�शाह�मोह�मद�क़ादरी�ने��न�न�ल�खत�तीन��े�णय��म���या�यक�समी�ा�का�वग�करण��कया�है :

1. सं�वधान�संशोधन�क���या�यक�समी�ा।�

2. संसद�और�रा�य��वधान�मंडल��और�अधीन�थ��वधान��क���या�यक�समी�ा।�

3. रा�य�के �अधीन�संघ�और�रा�य�और��ा�धकरण�क���शास�नक�कार�वाई�क���या�यक�समी�ा।�

इस��कार�, �या�यक�समी�ा��वधायी�और�काय�कारी�दोन��काय��पर�लागू�होती�है।�

सं�वधान�म�� ' �या�यक�समी�ा�' उ���का�कह��भी�उपयोग�नह���कया�गया�है।�

अनु�छे द� 31 B , नौव��अनुसूची�म��शा�मल�अ�ध�नयम��और��नयम��को�चुनौती�दे ने�और��कसी�भी�मौ�लक�अ�धकार��के �उ�लंघन�के �आधार�पर


अमा�य�होने�से�बचाता�है।�

हालां�क� , आई.आर. कोलोहो�मामले� (2007) म��एक�मह�वपूण��फै सले�म��सु�ीम�कोट� �ने�फै सला�सुनाया��क� 24 अ�ैल� , 1973 के �बाद�नौव
अनुसूची�के �तहत�रखे�गए�कानून��को��यायालय�म��चुनौती�द��जा�सकती�ह��य�द�वे�अनु�छे द� 14 , 15 , 19 और�21 के �तहत�गारंट�कृ त
मौ�लक�अ�धकार��या�' सं�वधानके �मूल�ढांचे ’ का�उ�लंघन�करते�ह�।�

�या�आप�जानते�है� ?

· सु�ीम�कोट� �ने��या�यक�समी�ा�क��श���को�सं�वधान�के �मूल�ढांचे�के �एक�त�व�बताया�है।�इस�लए� , �या�यक�समी�ा�क��श���म��एक


सं�वधान�संशोधन�के ��ारा�कटौती�या�कमी�नह��जा�सकती�है।�

�वचार�कर� !

· भारत�और�यू .एस.ए. म���या�यक�समी�ा�क��श���के �बीच�का�अंतर।�

· मूल�ढांचे�का��स�ांत�

QUESTION 63.
73व��सं�वधान�संशोधन�अ�ध�नयम�के �संदभ��म���न�न�ल�खत�कथन��पर��वचार�क��जए ?

1. यह�अ�ध�नयम���येक�रा�य�म� , अ�नवाय�����तरीय�पंचायती�राज�प��त�क���थापना�करता�है।�

2. सभी��तर��पर, पंचायत�के �अ�य��अ��य���प�से�चुने�जाएंगे।�

3. यह�अ�ध�नयम, म�हला��के ��लए�कु ल�सीट��का�एक��तहाई�आर�ण�क���व�था�करता�है।�

ऊपर��दए�गए�कथन��म��से�कौन�सा /से�सही�है� ?

a) 1 और�3
b) के वल�3
c) 2 और�3
d) 1 , 2 और�3

IASbaba
Web: http://ilp.iasbaba.com/ Score:
Email: ilp@iasbaba.com 4.00 / 200
Page 136
Exam Title : Test 21-Revision: Polity & ...
Email
Contact :

Correct Answer: B
Your Answer:
Explanation

Solution (b)

यह�अ�ध�नयम���येक�रा�य�म�����तरीय�पंचायती�राज�प��त�क���थापना�करता�है� , अथा�त� , �ाम, मा�य�मक�और��जला��तर�पर�पंचायत।


हालां�क� , कोई�ऐसा�रा�य��जसक��जनसं�या� 20 लाख�से�अ�धक�न�हो , को�मा�य�मक��तर�पर�पंचायत�के �गठन�नह��करने�क��छु ट�दे ता�है।�

�ाम�, मा�य�मक��तर�और��जला��तर�पर�पंचायत��के �सभी�सद�य��को�सीधे�लोग��के ��ारा�चुना�जाता�है।�इसके �अलावा�, म�यवत��और��जला


�तर�पर�पंचायत�के �अ�य��का�चुनाव��नवा��चत�सद�य��के ��ारा�उ�ही�म��से�अ��य���प�से�होता�है।�हालां�क� , गांव�के ��तर�पर�एक�पंचायत�के
अ�य��का�चुनाव�रा�य�वधान�मंडल��ारा��व�हत�तरीके �से��कया�जाता�है�।�

यह�अ�ध�नयम�म�हला��के ��लए�कु ल�सीट��के �कम�से�कम�एक -�तहाई�आर�ण�क���व�था�करता�है� (�जसम��अनुसू�चत�जा�त�और�अनुसू�चत


जनजा�त�से�संबं�धत�म�हला��के ��लए�आर��त�सीट��क��सं�या�शा�मल�है )।�

�वचार�कर� !

· पंचायती�राज�सं�थान��के �मु�े�

QUESTION 64.
इनम��से�कौन -कौन�संवैधा�नक��प�से��था�पत��वतं���नकाय�ह�� ?

1. चुनाव�आयोग�

2. भारत�के ��नयं�क�और�महालेखा�परी�क�का�पद�

3. संयु��लोक�सेवा�आयोग�

4. रा�ीय�मानवा�धकार�आयोग�

नीचे��दए�गए�कू ट�से�सही�उ�र�चु�नए ?

a) 1 और�2
b) 1,2 और�3
c) 3 और�4
d) 1 , 2 , 3 और�4
Correct Answer: A
Your Answer:
Explanation

Solution (a)

चुनाव�आयोग�और�भारत�का��नयं�क�एवं�महालेखा�परी�क�का�पद�संवैधा�नक��प�से��था�पत��वतं���नकाय�ह�।�

संयु��लोक�सेवा�आयोग�और�रा�ीय�मानवा�धकार�आयोग�वैधा�नक��नकाय�ह�।�

�वचार�कर� !

· संवैधा�नक��नकाय�

· रा�ीय�मानवा�धकार�आयोग�

QUESTION 65.
भारत�का��नयं�क�एवं�महालेखा�परी�क� (सीएजी) �न�न�ल�खत�म��से��कसका�लेखा -परी�ा�नह��करता�है� ?

IASbaba
Web: http://ilp.iasbaba.com/ Score:
Email: ilp@iasbaba.com 4.00 / 200
Page 137
Exam Title : Test 21-Revision: Polity & ...
Email
Contact :

1. �थानीय��नकाय�

2. तेल�एवं��ाकृ �तक�गैस�आयोग�

3. रा�य�सरकार��

4. क� ��सरकार�के �अ�ध�नयम��ारा��था�पत�सभी�सं�थाएं�

नीचे��दए�गए�कू ट�से�सही�उ�र�चु�नए ?

a) के वल�1
b) 1 और�2
c) के वल�4
d) 3 और�4
Correct Answer: C
Your Answer:
Explanation

Solution (c)

कै ग��थानीय��नकाय��, रा�य�सरकार��और�क� ��सरकार��का�ऑ�डट�करता�है।�

हालां�क� , साव�ज�नक��नगम��के �मामले�म��इसक��सी�मत�भू�मका�है।�

I. सीएजी��ारा�कु छ��नगम��का�पूण��एवं���य���प�से�लेखा -परी�ण��कया�जाता�है� , उदाहरण�के ��लए�, दामोदर�घाट���नगम�, तेल�एवं


�ाकृ �तक�गैस�आयोग� , एयर�इं�डया� , इं�डयन�एयरलाइंस�कॉप�रेशन� , और�अ�य।�

II. कु छ�अ�य��नगम��का�लेखा�परी�ा��नजी�पेशेवर�लेखा�परी�क��के ��ारा��कया�जाता�है��ज�ह��क� ��सरकार��ारा�सीएजी�के �परामश��से��नयु�


�कया�जाता�है।�उदाहरण�ह�� , स��ल�वेयरहाउ�स�ग�कॉरपोरेशन� , इंड���यल�फाइन�स�कॉरपोरेशन� , तथा�अ�य।�

III. कु छ�अ�य��नगम��क��पूरी�तरह��नजी�लेखा�परी�ा�क��जाती�है।��सरे�श�द��म�� , उनके �ऑ�डट��वशेष��प�से��नजी��ावसा�यक�लेखा


परी�क��के ��ारा��कया�जाता�है�और�इसम��कै ग�क��कोई�भू�मका�नह��होती�है।�ऐसे��नगम��के �उदाहरण�ह��जीवन�बीमा��नगम� , भारतीय��रजव�
ब�क� , भारतीय��टे ट�ब�क� , भारतीय�खा���नगम�, और�अ�य।�

�या�आप�जानते�है� ?

· गु�त�सेवा��य , सीएजी�क��ऑ�ड�ट�ग�भू�मका�पर�सीमाएं�आरो�पत�करता�है।�इस�संबंध�म�� , कै ग�काय�कारी�एज��सय��के ��ारा��कए�गए��य


के ��यौरे�नह��मांग�सकता�है।�

�वचार�कर� !

· कै ग�क��भू�मका�क��आलोचना�

QUESTION 66.
�न�न�ल�खत�म��से�कौन� 42वां�संशोधन�अ�ध�नयम�का /के ��ावधान�ह�� ?

1. ��तावना�म�� ' समाजवाद��' श�द�को�शा�मल�करना।�

2. कु छ��वषय��का�रा�य�सूची�से�समवत��सूची�म���थानांतरण।�

3. संप���के �अ�धकार�को��व�धक�अ�धकार�बनाना।�

नीचे��दए�गए�कू ट�से�सही�उ�र�चु�नए ?

a) 1 और�2

IASbaba
Web: http://ilp.iasbaba.com/ Score:
Email: ilp@iasbaba.com 4.00 / 200
Page 138
Exam Title : Test 21-Revision: Polity & ...
Email
Contact :

b) के वल�1
c) 2 और�3
d) 1 , 2 और�3
Correct Answer: A
Your Answer:
Explanation

Solution (a)

42व��संशोधन�अ�ध�नयम� , 1976 के �तहत��न�न�काय���कया�गया :

· ��तावना�म�� ' समाजवाद��, धम��नरपे��और�अखंडता� ' श�द�जोड़े�गए।�

· अनु�छे द� 51-ए�के �तहत�मूल�कत����को�जोड़ा�गया।�

· रा�य�सूची�से�कु छ��वषय��का�समवत��सूची�म���थानांतरण��कया�गया।�

· नए�डीपीएसपी�शा�मल��कये�गए।�

संप���के �अ�धकार�को�44व��संशोधन�अ�ध�नयम� , 1978 के �तहत�एक�कानूनी�अ�धकार�बनाया�गया।�

�वचार�कर� !

· 44व��संशोधन�अ�ध�नयम�के �मह�वपूण���ावधान�

QUESTION 67.
�न�न�ल�खत�म��से�कौन�सी��ा��तयां�क� ��के �गैर -कर�राज�व�का�मु�य��ोत�नह��ह�� ?

a) ब��क�ग�
b) वन�
c) रेलवे�
d) �स�का�और�मु�ा�
Correct Answer: B
Your Answer: Unanswered
Explanation

Solution (b)

क� ��क��गैर -कर-राज�व�के ��मुख��ोत��न�न��ा��तयां�ह� :

i. डाक�और�टे ली�ाफ�

ii. रेल�

iii. ब��क�ग�

iv. �सारण�

v. �स�का�और�मु�ा�

vi. क� ��य�साव�ज�नक��े��के �उ�म�

vii. ए�के ट�एवं�लै�स�

वन��से��ा�त�आय , रा�य��के ��लए�गैर -कर�राज�व�के ��ो��है।�

IASbaba
Web: http://ilp.iasbaba.com/ Score:
Email: ilp@iasbaba.com 4.00 / 200
Page 139
Exam Title : Test 21-Revision: Polity & ...
Email
Contact :

�वचार�कर� !

· क� ��और�रा�य��के �बीच�कर�और�गैर -कर�राज�व�का��वतरण�

QUESTION 68.
लोकसभा�अ�य��के �संदभ��म���न�न�ल�खत�कथन��पर��वचार�क��जए ?

1. वह�मत�वभाजन(टाई) क��दशा�म��एक��नणा�यक�वोट�दे �सकता�है।�

2. वह�सदन�म��मत�नह��दे �सकता�है� , जब�उनके ��न�कासन�का�एक���ताव�सदन�म���वचाराधीन�है।�

3. लोकसभा�के ��वघटन�के �बाद�भी�वह�अपने�पद�पर�बना�रहता�है।�

ऊपर��दए�गए�कथन��म��से�कौन�सा /से�सही�है� ?

a) के वल�1
b) 1 और�3
c) 2 और�3
d) 1 , 2 और�3
Correct Answer: B
Your Answer:
Explanation

Solution (b)

सामा�य���थ�त�म��अ�य� (�पीकर) मत�नह��दे ता�है।�ले�कन�मत�वभाजन (टाई) के �मामले�म��का��टं ग�वो�ट�ग�का��योग�कर�सकता�ह�।�

जब�अ�य��को�हटाने�के ��लए�एक�संक�प�सदन�के �सम���वचाराधीन�है� , तो�वह�सभा�क��बैठक�म��अ�य�ता�नह��कर�सकता (पीठासीन


अ�धकारी�नह��होता�है ) , हालां�क�वह�उप��थत�हो�सकता�है , इस�समय�सदन�क��काय�वाही�म��भाग�ले�सकता�है।�ऐसी���थ�त�म��उसे�मत�दे ने�का
भी�अ�धकार�होगा�पर�तु�मत��के �बराबर�होने�क��दशा�म��मत�दे ने�का�अ�धकार�नह��होगा।�

जब�भी�लोकसभा�भंग�हो�जाती�है� , तो�अ�य��अपना�पद�नह��छोड़ता�ह��और�नव��नवा��चत�लोकसभा�क��बैठक�तक�पद�धारण�करता�है।�

�वचार�कर� !

· �पीकर�क��श��यां�और���थ�त�

· �पीकर�क��भू�मका�पर��ववाद�

QUESTION 69.
धन��वधेयक�के �संदभ��म���न�न�ल�खत�कथन��पर��वचार�क��जए ?

1. एक�धन��वधेयक , के वल�एक�मं�ी�के ��ारा�ही���तुत��कया�जा�सकता�है।�

2. रा�य�सभा, धन��वधेयक�को�अ�वीकार�या�संशो�धत�नह��कर�सकती�है।�

3. रा�प�त, �वधेयक�पर�अपनी�सहम�त�रोक�नह��सकते�ह�।�

ऊपर��दए�गए�कथन��म��से�कौन�सा /से�सही�है� ?

a) 1 और�2
b) 1 और�3
c) 2 और�3

IASbaba
Web: http://ilp.iasbaba.com/ Score:
Email: ilp@iasbaba.com 4.00 / 200
Page 140
Exam Title : Test 21-Revision: Polity & ...
Email
Contact :

d) 1,2 और�3
Correct Answer: A
Your Answer:
Explanation

Solution (a)

सं�वधान�के �अनु�छे द� 110 म��धन��वधेयक�क��प�रभाषा�द��गयी�है।�

ऐसे���येक��वधेयक�को�सरकारी��वधेयक�माना�जाता�है�और�एक�मं�ी��ारा�ही�पेश��कया�जा�सकता�है।�

धन��वधेयक�के �संबंध�म��रा�य�सभा�क��श��यां���तबं�धत�कर�द��गयी�ह�।�यह�एक�धन��वधेयक�को�अ�वीकार�या�संशो�धत�नह��कर�सकता�है।�

जब�एक�धन��वधेयक�रा�प�त�के �सम����तुत��कया�जाता�है� , तो�वह�या�तो�इस�पर�अपनी�सहम�त�दे �सकता�है�या��फर�इसे�रोक�कर�रख


सकता�है�ले�कन�वह��कसी�भी�दशा�म��इसे�सदन��के �पुन�व�चार�के ��लए�वापस�नह��लौटा�सकता�है।�

�वचार�कर� !

· धन��वधेयक�बनाम��व���वधेयक�

QUESTION 70.
�ा�कलन�स�म�त�के �संदभ��म���न�न�ल�खत�कथन��पर��वचार�क��जए ?

1. इस�स�म�त�म��रा�यसभा�का�कोई���त�न�ध�व�नह��होता�है।�

2. इसके �सद�य, �पीकर�के ��ारा�चुने�जाते�ह�।�

3. यह�संसद�के ��ारा, मतदान�के �प�ात्�बजट�अनुमान��क��जांच�करता�है।�

ऊपर��दए�गए�कथन��म��से�कौन�सा /से�सही�है� ?

a) 1 और�2
b) 1 और�3
c) 2 और�3
d) 1,2 और�3
Correct Answer: B
Your Answer:
Explanation

Solution (b)

�ा�कलन�स�म�त�बजट�म��शा�मल���कलन��क��जांच�करती�है�और�साव�ज�नक��य��म�� ' �मत��यता�' लाने�का�सुझाव�दे ती�है।�

इसम�� 30 सद�य�होते�ह��तथा�सभी�लोकसभा�से�ही�होते�ह�।�इस�स�म�त�म��रा�यसभा�का�कोई���त�न�ध�व�नह��है।�

इसके �सद�य��का�चुनाव���तवष��लोकसभा�के ��ारा�इसके �सद�य��म��से��कया�जाता�है�और�इसम��समानुपा�तक���त�न�ध�व�के ��स�ांत��का


पालन�एकल�ह�तांतरणीय�मत�के �मा�यम�से��कया�जाता�है।�

यह�बजट��ा�कलन��क��जांच�तभी�करता�है�जब�इसके ��लए�संसद�म��मतदान�हो�चूका�हो� , और�इससे�पहले�नह�।�

�या�आप�जानते�है� ?

· स�म�त�क��भू�मका�सी�मत�होती�है� , वह�न�तो�संसद��ारा��नधा��रत�नी�त�पर�सवाल�कर�सकती�है�और�न�ही�इसक���सफा�रश��मं�ालय��पर
बा�यकारी�होती�ह�।�

IASbaba
Web: http://ilp.iasbaba.com/ Score:
Email: ilp@iasbaba.com 4.00 / 200
Page 141
Exam Title : Test 21-Revision: Polity & ...
Email
Contact :

�वचार�कर� !

· �थायी�स�म�तयां�और�तदथ��स�म�तयां�

QUESTION 71.
उ�च��यायालय�के �संदभ��म���न�न�ल�खत�कथन��पर��वचार�क��जए ?

1. संबं�धत�उ�च��यायालय�के �मु�य��यायाधीश�के �साथ�परामश��करने�के �बाद, रा�यपाल�के ��ारा�उ�च��यायालय�के ��यायाधीश��नयु���कए


जाते�ह�।�

2. रा�प�त�काय��क��आव�यकता�के �अनुसार�समय -समय�पर�उ�च��यायालय�क��सं�या��नधा��रत�करते�है।�

ऊपर��दए�गए�कथन��म��से�कौन�सा /से�सह��है� ?

a) के वल�1
b) के वल�2
c) दोन��1 और�2
d) न�तो�1 और�न�ही�2
Correct Answer: B
Your Answer:
Explanation

Solution (b)

संबं�धत�उ�च��यायालय�के �मु�य��यायाधीश�के �साथ�परामश��करने�के �बाद�रा�प�त�के ��ारा�उ�च��यायालय�के ��यायाधीश��नयु���कए�जाते�ह�।�

सं�वधान�एक�उ�च��यायालय�क��सं�या�को��न�द���नह��करता�है�तथा�इसे�रा�प�त�के ��ववेक�पर�छोड़��दया�गया��है।�तदनुसार� , रा�प�त�काय�


क��आव�यकता�के �अनुसार�समय -समय�पर�उ�च��यायालय�क��सं�या��नधा��रत�करते�है।�

�या�आप�जानते�है� ?

· �द�ली�एकमा��क� �शा�सत��दे श�है��जसका��वयं�का�उ�च��यायालय�है।�

�वचार�कर� !

· रा�ीय��या�यक��नयु���आयोग� (एनजेएसी )

· एक�उ�च��यायालय�के ��यायाधीश�को�हटाने�क�����या�

QUESTION 72.
भाग�XXI के �अंतग�त��वशेष��ावधान , �न�न�ल�खत�रा�य��म��से��कस�के ��लए�नह���कए�गए�ह�� ?

a) नागाल�ड�
b) गोवा�
c) �स��कम�
d) पंजाब�
Correct Answer: D
Your Answer:
Explanation

Solution (d)

IASbaba
Web: http://ilp.iasbaba.com/ Score:
Email: ilp@iasbaba.com 4.00 / 200
Page 142
Exam Title : Test 21-Revision: Polity & ...
Email
Contact :

सं�वधान�के �भाग�XXI म��अनु�छे द� 371 से� 371-जे�तक�म�� 12 रा�य��अथा�त�महारा�� , गुजरात� , नागाल�ड� , असम�, म�णपुर� , आं���दे श� ,
तेलंगाना� , �स��कम�, �मजोरम�, अ�णाचल��दे श� , गोवा�और�कना�टक�के ��लए��वशेष��ावधान�ह�।�

�या�आप�जानते�है� ?

· मूल��प�से� , सं�वधान�म��इन�रा�य��के ��लए�कोई��वशेष��ावधान�नह��था।�इ�ह��बाद�के �कई�संशोधन��के ��ारा�शा�मल��कया�गया�ह�।�

�वचार�कर� !

· ज�मू�और�क�मीर�क���वशेष���थ�त�

QUESTION 73.
रा�प�त�चुनाव�म��मतदान�का�अ�धकार�है ?

a) �ाकृ �तक�अ�धकार�
b) संवैधा�नक�अ�धकार�
c) मूल�अ�धकार�
d) �व�धक�अ�धकार�
Correct Answer: D
Your Answer:
Explanation

Solution (d)

संवैधा�नक�अ�धकार�वे�ह��जो��प���प�से�सं�वधान�म��व�ण�त�होते�ह�।�

अनु�छे द� 54 म��रा�प�त�चुनाव�और��नवा�चक�म�डल�के �सद�य��के �बारे�म��बताया�गया�है� , ले�कन�उनके �मत�दे ने�के �अ�धकार�के �बारे�म��कु छ


नह��कहा�गया�है।�जन���त�न�ध�व�अ�ध�नयम�के �तहत�इसका��ावधान��कया�गया�है।�इस�लए�यह�एक�कानूनी�अ�धकार�है।�

इसके �अलावा�हाल�ही�म��महारा��सरकार�ने�जेल�म��बंद��वधायको�के ��ारा��कये�गए�मतदान�का��वरोध��कया�और�कहा��क�यह�एक�संवैधा�नक


अ�धकार�नह��है।�

आगे�और�अ�ययन�के ��लए:

http://pib.nic.in/newsite/PrintRelease.aspx?relid=167193

�वचार�कर� !

· जन���त�न�ध�व�अ�ध�नयम�

QUESTION 74.
भारतीय�सं�वधान�के ��नमा�ण�के �संदभ��म���न�न�ल�खत�कथन��पर��वचार�क��जए ?

1. सव��थम� , 1935 म��भारतीय�रा�ीय�कां�ेस�ने�सं�वधान�सभा�के �गठन�क��मांग�क��थी।�

2. ���स��मशन�योजना�म��सं�वधान�सभा�के �गठन�क��मांग�को��वीकार��कया�गया।�

3. जवाहरलाल�नेह��को�अंत�रम�सरकार�तथा�सं�वधान�सभा�का��मुख�बनाया�गया।�

ऊपर��दए�गए�कथन��म��से�कौन�सा /से�गलत�है� ?

a) 1 और�2
b) 2 और�3

IASbaba
Web: http://ilp.iasbaba.com/ Score:
Email: ilp@iasbaba.com 4.00 / 200
Page 143
Exam Title : Test 21-Revision: Polity & ...
Email
Contact :

c) 1 और�3
d) 1,2 और�3
Correct Answer: B
Your Answer:
Explanation

Solution (b)

1934 म�� , एम.एन. रॉय�ने�पहली�बार�सं�वधान�सभा�के �गठन�के ��वचार�को���तुत��कया�और� 1935 म��भारतीय�रा�ीय�कां�ेस�ने�पहली�बार


भारत�के �सं�वधान�के ��नमा�ण�के ��लए�आ�धका�रक�तौर�पर�सं�वधान�सभा�के �गठन�क��मांग�क�।�

आ�खरकार�1940 के �अग�त���ताव�म�����टश�सरकार�ने�इस�मांग�को��वीकार�कर��लया।�

अंत�रम�सरकार�के �सद�य�वाइसराय�क��काय�कारी�प�रषद�के �सद�य�थे।�वायसराय�प�रषद�के ��मुख�बने�रहे।�ले�कन� , जवाहरलाल�नेह��को


प�रषद�का�उपा�य��के ��प�म��ना�मत��कया�गया�था।�

�वचार�कर� !

· सं�वधान�सभा�क��संरचना�

· सं�वधान�सभा�क���मुख�स�म�तयां�

QUESTION 75.
बजट�के �अ�ध�नयमन�के �संबंध�म���न�न�ल�खत�म��से�कौन�सा�संवैधा�नक��ावधान�गलत�है� ?

a) संसद , कर�म��वृ���नह��कर�सकती�है।�
b) रा�य�सभा, अनुदान�क��मांग�पर�मतदान�नह��कर�सकती�है।�
c) धन��वधेयक�के ��वपरीत�, रा�यसभा�म��कराधान�से�स�बं�धत�एक��व���वधेयक���तुत��कया�जा�सकता�है।�
d) �व�ध�के ��ा�धकार�के �अलावा�कोई�कर�नह��लगाया�जाएगा।�
Correct Answer: C
Your Answer:
Explanation

Solution (c)

संसद��कसी�कर�को�कम�कर�सकती�है�या�समा�त�कर�सकती�है� , ले�कन�इसे�बढ़ा�नह��सकती�है।�

रा�य�सभा�अनुदान�क��मांग�पर�मतदान�नह��कर�सकती�है।�

एक�धन��वधेयक�या�कर�से�स�बं�धत�एक��व���वधेयक�को�रा�यसभा�म����तुत�नह���कया�जाता�है� - इसे�के वल�लोकसभा�म���कया�जा�सकता


है।�

�व�ध�के ��ा�धकार�के ��बना�कोई�कर�नह��लगाया�जाएगा।�

�वचार�कर� !

· बजट�के �अ�ध�नयम�के �चरण�

· भा�रत��य�

· क� ��सरकार�के ��व�भ���कार�क���न�धयाँ�

QUESTION 76.
�व��आयोग�के �संदभ��म���न�न�ल�खत�कथन��पर��वचार�क��जये�

IASbaba
Web: http://ilp.iasbaba.com/ Score:
Email: ilp@iasbaba.com 4.00 / 200
Page 144
Exam Title : Test 21-Revision: Polity & ...
Email
Contact :

1. यह�एक�अध���या�यक��नकाय�है।�

2. आयोग�का�अ�य��पुन�न�यु���के ��लए�पा��नह��ह�।�

3. आयोग�के �सद�य��क��यो�यताएं�सं�वधान�म���न�द���नह��है।�

ऊपर��दए�गए�कथन��म��से�कौन�सा /से�सही�है� ?

a) 1 और�2
b) 1 और�3
c) 2 और�3
d) 1,2 और�3
Correct Answer: B
Your Answer:
Explanation

Solution (b)

भारत�के �सं�वधान�का�अनु�छे द� 280 के �अंतग�त�एक�अध���या�यक��नकाय�के ��प�म���व��आयोग�क���व�था�क��गई�है।�यह�भारत�के �रा�प�त


के ��ारा�हर�पांचव��वष��या�आव�यकतानुसार�इसके �पहले�भी�इसका�गठन��कया�जाता�है।�

�व��आयोग�म��एक�अ�य��और�चार�अ�य�सद�य�होते�ह� , �जनक���नयु���रा�प�त�करते�है।�उनका�काय�काल�रा�प�त�के �आदे श�के �तहत


�न�द���होता�है।�वे�पुन�न�यु���के ��लए�भी�पा��ह�।�

सं�वधान�ने�संसद�को�आयोग�के �सद�य��क��यो�यता�और�चयन��व�ध�का��नधा�रण�करने�का�अ�धकार��दया�है।�

�या�आप�जानते�है� ?

· भारत�का�सं�वधान��व��आयोग�को�भारत�म��राजकोषीय�संघवाद�के �संतुलन�प�हया�के ��प�म��पेश�करता�है।�

�वचार�कर� !

· �व��आयोग�के �काय��

· राजकोषीय�संघवाद�के ��बना�सहकारी�संघवाद�हा�सल�नह���कया�जा�सकता�है� ?

QUESTION 77.
क� ��य��शास�नक�अ�धकरण(सीएट�) के �मूल��े�ा�धकार�म���न�न�ल�खत�म��से�कौन�शा�मल�नह��है� ?

a) अ�खल�भारतीय�सेवाएं�
b) र�ा(�डफे �स) सेवा��के �नाग�रक�कम�चारी�
c) संसद�के �स�चवालय��टाफ़�
d) क� ��के �अंतग�त��स�वल�पद�
Correct Answer: C
Your Answer: Unanswered
Explanation

Solution (c)

के ���य��शास�नक�अ�धकरण(स��ल�एड�म�न��े �टव����यूनल :सी.ए.ट�.) भत��के �संबंध�म��अपने�मूल�अ�धकार��े��के �अंतग�त�आने�वाले�लोक


सेवक��क��भत��व�सेवा�मामल��को�दे खता�है।�इसके �अ�धकार��े��म��अ�खल�भारतीय�सेवा�� , के ���य��स�वल�सेवा� , क� ��के �अंतग�त�नाग�रक
पद��और�र�ा�सेवा��के �नाग�रक�कम�चा�रय��तक��व�ता�रत�है।�हालां�क� , सै�य�सेवा��के �सद�य�व�अ�धकारी, उ�चतम��यायालय�के �कम�चारी
और�संसद�के �स�चवालय��टाफ�के �सद�य�इसके �अंतग�त�शा�मल�नह��होते�ह�।�

IASbaba
Web: http://ilp.iasbaba.com/ Score:
Email: ilp@iasbaba.com 4.00 / 200
Page 145
Exam Title : Test 21-Revision: Polity & ...
Email
Contact :

�या�आप�जानते�है� ?

· 2006 म���शास�नक��याया�धकरण�अ�ध�नयम� , 1985 म��संशोधन�के �साथ�सद�य��को�उ�च��यायालय��के ��यायाधीश��का�दजा���दया�गया


है।�

�वचार�कर� !

· अनु�छे द� 323 ए�और�अनु�छे द� 323 बी�

· �याय�का��याया�धकरण�

QUESTION 78.
भारत�के �अटॉन��जनरल�के �पद�के �संदभ��म���न�न�ल�खत�कथन��पर��वचार�क��जए ?

1. यह�दे श�का�सव��च��व�ध�अ�धकारी�होता�ह�।�

2. उसे�वह�����का�होना�चा�हए�जो�उ�चतम��यायालय�के ��यायाधीश�के ��प�म���नयु���कए�जाने�यो�य�हो।�

3. उसे�उ�चतम��यायालय�के ��यायाधीश�के �समान�री�त�से�हटाया�जा�सकता�है।�

ऊपर��दए�गए�कथन��म��से�कौन�सा /से�सही�है� ?

a) 1 और�2
b) 1 और�3
c) 2 और�3
d) 1,2 और�3
Correct Answer: A
Your Answer:
Explanation

Solution (a)

सं�वधान�म�� (अनु�छे द� 76) भारत�के �महा�यायवाद�(अटॉन��जनरल) के �पद�क���व�था�क��गई�है।�वह�दे श�का�सव��च�कानून�अ�धकारी�होता


ह�।�

अटॉन��जनरल�(एजी) क���नयु���रा�प�त�के ��ारा�क��जाती�है�वह�उस�����म��उन�यो�यता��का�होना�आव�यक�है , जो�सु�ीम�कोट� �के


�यायाधीश�क���नयु���के ��लए�होती�है।�

सं�वधान�म��उसको�हटाने�के ��लए����या�और�आधार�नह��है।�वह�रा�प�त�के ��सादपय�त�पदधारण�करता�है।�

�या�आप�जानते�है� ?

· अटॉन��जनरल�सरकार�का�पूण�का�लक�वक�ल�नह��है�और��नजी�कानूनी��ै��टस�क��अनुम�त�है।�

�वचार�कर� !

IASbaba
Web: http://ilp.iasbaba.com/ Score:
Email: ilp@iasbaba.com 4.00 / 200
Page 146
Exam Title : Test 21-Revision: Polity & ...
Email
Contact :

· भारत�के �सॉ�ल�सटर�जनरल�

· रा�य�के �महा�धव�ा�

QUESTION 79.
�न�न�ल�खत�म��से��कस�सं�वधान�संशोधन�के ��ारा��ाथ�मक��श�ा�को�मूल�अ�धकार�का�दजा���दया�गया� ?

a) 84 th
b) 91 st
c) 93 rd
d) 86 th
Correct Answer: D
Your Answer:
Explanation

Solution (d)

86व��सं�वधान�संशोधन� 2002 ने��ाथ�मक��श�ा�को�मूल�अ�धकार�बनाया।�

इसके ��ारा�अनु�छे द� 21ए�, अनु�छे द� 51ए�के �तहत�खंड (k) को�जोड़ा�गया�तथा�अनु�छे द� 45 के �मूल�पाठ�को�प�रव�त�त��कया�गया।�

�वचार�कर� !

· �श�ा�का�अ�धकार�कानून�का��व�ेषण�

QUESTION 80.
�न�न�ल�खत�म��से�कौन� ‘सं�वधान�को�बनाएं�रखने ’ का�शपथ�लेता�है� ?

a) अ�य��
b) �धान�मं�ी�
c) भारत�के �मु�य��यायाधीश�
d) संसद�सद�य�
Correct Answer: C
Your Answer:
Explanation

Solution (c)

भारत�के �मु�य��यायाधीश�और�सव��च��यायालय�तथा�उ�च��यायालय�के ��यायाधीश��ने�सं�वधान�और��व�ध�क��मया�दा�को�बनाए�रखने�के ��लए


शपथ�लेते�है।�

�या�आप�जानते�है� ?

· इन��यायाधीश��के �अलावा�, कै ग�और�सूचना�आयु��भी�इसी�तरह�क��शपथ�लेते�ह�।�

�वचार�कर� !

· रा�प�त�का�शपथ�

QUESTION 81.
युकाटन��ाय��प , पानी�के �नीचे�क��गुफा��के �दो�बड़े�नेटवक� �के ��लए�समाचार�म��था।�यह��न�न�ल�खत�म��से��कस�दे श�म����थत�है� ?

IASbaba
Web: http://ilp.iasbaba.com/ Score:
Email: ilp@iasbaba.com 4.00 / 200
Page 147
Exam Title : Test 21-Revision: Polity & ...
Email
Contact :

a) �यूज़ील�ड�
b) मे��सको�
c) चीन�
d) अज�ट�ना�
Correct Answer: B
Your Answer:
Explanation

Solution (b)

मै��सको�के �यूकाटन��ाय��प�के �अंतग�त�दो��वशालकाय�भूजल�क�दराएँ� (347 �कलोमीटर), धरती�पर��ात�सबसे�बड़ी��न�मा��जत�गुहा


�णाली��न�म�त�करते�ह�।�

�वशाल��लॉथ�के �जीवा�म�तथा�माया�दे वता�का�एक��व�तृत�मं�दर�के �भ�नावशेष�वहां�पाए�गए�ह�।�

यूकाटन��ाय��प�माया�लोग��के ��मारक�य�अवशेष��से�भरा��आ�है� , �जनके �शहर��को�सीनोट् स�के ��प�म��जाना�जाता�था।�यही�कारण�है��क


इसने�भू�मगत�जल�से�जुड़े��स�कहोल��के �एक��ापक�नेटवक� �को�अपनी�ओर�आक�ष�त��कया।�

�या�आप�जानते�है� ?

· एक��ाचीन��परा�मड�शहर�, �जसे�एंगामुको�के �नाम�से�जाना�जाता�है� , को��लडार(LIDAR) टे �नोलॉजी�का�उपयोग�करके �मै��सको�के


मु�यभू�म�म��भू�म�के �नीचे�खोजा�गया।�

�वचार�कर� !

· माया�स�यता�

· म�य�अमे�रका�के �दे श�

QUESTION 82.
�न�न�ल�खत�कथन��पर��वचार�क��जए?

1. इ�ह�ने�म�म -उल-बहरीन�क��रचना�क�।�

2. इ�ह�ने�सं�कृ त�म���लखे�उप�नषद��का�फारसी�भाषा�म��अनुवाद��कया।�

�न�न�ल�खत�म��से�कौन�ऊपर��दए�गए�कथन��का�सबसे�उपयु��वण�न�है� ?

a) �शवाजी�
b) दारा��शकोह�
c) अबुल�फजल�
d) अकबर�

IASbaba
Web: http://ilp.iasbaba.com/ Score:
Email: ilp@iasbaba.com 4.00 / 200
Page 148
Exam Title : Test 21-Revision: Polity & ...
Email
Contact :

Correct Answer: B
Your Answer: Unanswered
Explanation

Solution (b)

दारा�शकोह�ने�मु��लम��व�ान��के �अ�ययन�के ��लए�सभी�उप�नषद��का�अनुवाद�सं�कृ त�से�फारसी�म���कया�था।�

उ�ह�ने�सूफ�वाद�और�वै�दक�दश�नशा���के �बीच�सम�पता�पर��व�तार�करने�के ��लए�म�म-उल-बहरीन�(दो�समु���का�संगम ) �लखा।�

�वचार�कर� !

· दारा��शकोह�के �बारे�म��त�य�

· औरंगजेब�क��नी�तयां�

QUESTION 83.
इयान, महाक�प( eras ), क�प( periods ) , युग ( epochs ) तथा�आयु ( ages ) भूगभ�य�काल�म�के ���व�भ��ख�ड�को��न��पत
करते�ह�।�पृ�वी�के �इ�तहास�को����करने�के ��लए�इन�समय�मानक��को��था�पत�करने�के ��लए�कौन�सी�वै��क�सं�था��ज�मेदार�है� ?

a) द�इंटरनेशनल�सोसाइट��ऑफ़�अ��कलो�ाफ��
b) इंटरनेशनल�कमीशन�ऑन���े ट��ाफ��
c) इंटरनेशनल�यू�नयन�ऑफ़�बाल�नयो�ाफ��
d) द�इंटरनेशनल�कमीशन�ऑन�अथ��एंड�साइंस�
Correct Answer: B
Your Answer:
Explanation

Solution (b)

��ै ट��ाफ��पर�अंतरा��ीय�आयोग , ‘भूगभ�य��व�ान�का�अंतरा��ीय�संघ� (आईयूजीएस )’ का�सबसे�बड़ा�और�सबसे�पुराना�संघटक�वै�ा�नक


�नकाय�है।�

इसका�मु�य�उ�े �य�अंतररा�ीय��ोनो��ाट��ा�फ़क�चाट� �क��वै��क�इकाइय�� (�स�टम�, सीरीज�और��टे ज ) को�ठ�क�से�प�रभा�षत�करना�है� , जो


बदले�म�� , अंतरा��ीय�भूगभ��काल�म�के �पैमाने�क��इकाइय�� (पी�रयड�, एपोच�और�ऐज) का�आधार�है� ; इस��कार�पृ�वी�के �इ�तहास�को���
करने�के ��लए�मौ�लक�पैमाने�के ��लए�वै��क�मानक��क���थापना�करना।�

QUESTION 84.
खाद��और��ामो�ोग�आयोग�(के.वी.आई.सी.) के �संदभ��म���न�न�ल�खत�कथन��पर��वचार�क��जए।�

1. सू�म� , लघु�और�म�यम�उ�म�मं�ालय�के �अंतग�त�यह�एक�वैधा�नक��नकाय�है।�

2. इसका�एक�उ�े �य��ामीण�युवा��को�रोजगार�उपल�ध�कराना�है।�

3. यह��ामीण�उ�ोग��को��व�ीय�सहायता��दान�करती�है।�

ऊपर��दए�गए�कथन��म��से�कौन�सा /से�सह��है� ?

a) 1 और�2
b) 1 और�3
c) 2 और�3
d) 1,2 और�3
Correct Answer: D

IASbaba
Web: http://ilp.iasbaba.com/ Score:
Email: ilp@iasbaba.com 4.00 / 200
Page 149
Exam Title : Test 21-Revision: Polity & ...
Email
Contact :

Your Answer:
Explanation

Solution (d)

खाद��और��ामो�ोग�आयोग�(के वीआईसी) 1956 म��संसद�के �एक�अ�ध�नयम��ारा��था�पत�एक�सां�व�धक��नकाय�है।�

के वीआईसी�के ��ापक�उ�े �य�ह��

· रोजगार��दान�करने�का�सामा�जक�उ�े �य�

· �ब���यो�य�व�तुए�ं बनाने�का�आ�थ�क�उ�े �य�

· गरीब��को�आ�म-�नभ�र�बनाने�और�ठोस��ामीण�समुदाय�क��भावना�का��नमा�ण�करने�का��ापक�उ�े �य।�

के.वी.आई.सी. को�खाद��और��ामीण�उ�ोग��के ��वकास�और�संचालन�के ��लए�सं�थान��और����य��को��व�ीय�सहायता��दान�करने�का


काय��स�पा�गया�है।�

के.वी.आई.सी., उ�पाद��का�खरापन�सु�न��त�करने�और�गुणव�ा�के �मानक��को��नधा��रत�करने�तथा�खाद��और��ामीण�उ�ोग��के �उ�पाद��के


मानक��के �अनु�प�सु�न��त�करने�के ��लए�कदम�उठा�सकती�ह�।�

�या�आप�जानते�है� ?

के वीआईसी�अ�ध�नयम�के �अनुसार� , �ामीण��े��म����थत�कोई�भी�उ�ोग�जो��बजली�के �इ�तेमाल�के �साथ�या�इ�तेमाल�के �बगैर��कसी�भी�व�तु


का�उ�पादन�करता�है�या�कोई�सेवा��दान�करता�है , तथा��जसम����त�कारीगर�या�कामगार , एक��न��त�पूंजी��नवेश�एक�लाख��पए�से�अ�धक
नह��है� , को�एक��ामीण�उ�ोग�कहा�जाता�है।�

�वचार�कर� !

· खाद��हाट�

· �वीट�रेवोलुशन�

http://www.business-standard.com/article/pti-stories/kvic-divisional-office-comes-up-in-hubli-91-
yr-old-khadi-centre-revived-in-mysuru-118022701086_1.html

QUESTION 85.
हाल�ही�म���व��मं�ालय�ने� "ऑपरेशन��ीन " नामक�एक�पहल�क��घोषणा�क��है� , जो�नीचे��दए�गए�कथन�का�सबसे�अ�छा�वण�न�करता�है� ?

a) सभी�सरकारी�काया�लय��को�कागज�र�हत (पेपरलेस ) बनाने�के ��लए�पहल।�


b) दे श�म��फल��और�स��जय��के �उ�पादन�को�बढ़ाने�के ��लए�पहल।�
c) दे श�के �सभी�मु�य�हवाई�अ���को�काब�न�तट�थ�बनाने�के ��लए�पहल।�
d) वा�पीकरण�से�जल�का�संर�ण�करने�तथा�साथ�ही��व�ुत�उ�प��करने�के ��लए��मुख�नद�य�माग��पर�सोलर�पैनल��को��था�पत�करने�के
�लए�पहल।�
Correct Answer: B
Your Answer:
Explanation

Solution (b)

टमाटर�, �याज�और�आलू�के �उ�पादन�को�बढ़ाने�के ��लए�ऑपरेशन��लड�क��तज��पर�ऑपरेशन��ीन�क��घोषणा�क��गई�है।�

�वचार�कर� !

· कृ �ष�संकट�को�संबो�धत�करने�के ��लए��कए�गए�उपाय�

IASbaba
Web: http://ilp.iasbaba.com/ Score:
Email: ilp@iasbaba.com 4.00 / 200
Page 150
Exam Title : Test 21-Revision: Polity & ...
Email
Contact :

https://www.thehindubusinessline.com/economy/budget/rs-500-crore-allocated-for-operation-
green/article22619481.ece

QUESTION 86.
�लोबल�बीज�वॉ�ट�के �संदभ��म���न�न�ल�खत�कथन��पर��वचार�क��जए ?

1. यह��ीनल�ड�म��अव��थत�है।�

2. इसका�कोई��वा�म�व�अ�धकार(ओनर�शप�राईट) नह��है�और�यह�के वल�मानवता�के �खा��सुर�ा�के ��लए�है।�

3. भारत�ने�भी�वॉ�ट�के ��लए�बीज�भेजे�ह�।�

ऊपर��दए�गए�कथन��म��से�कौन�सा /से�सही�है� ?

a) 1 और�2
b) के वल�3
c) 1 और�3
d) 2 और�3
Correct Answer: B
Your Answer: Unanswered
Explanation

Solution (b)

�लोबल�सीड�वॉ�ट, �वालबाड��म��एक�पहाड़�के �नीचे���थत�है� , जो��क�नाव��जयन���पसमूह�म��एक�सु�रवत��आक� �टक���प�है।�इसे��वालबाड�


�लोबल�बीज�वो�ट�(एसजीएसवी) कहा�जाता�है।�

यह�एक��तजोरी�है�जो�मानवता�का�परम�खा��सुर�ा�शा�मल�है।�

यह�मानव��न�म�त�तथा��ाकृ �तक�आपदा��का�सामना�करने�के ��लए�बनाया�गया�है।�इसे�सै�ां�तक��प�से�पारंप�रक�बम��से�संर��त��कया�गया


है� , तथा�समु���तर�म��वृ���के �प�ंच�से��र�रखा�गया�है।�

नॉव��क��सरकार�इस�वॉ�ट�क��मा�लक�है�और�जमाकता���के �पास�ओनर�शप�राईट�ह�।�भारत�ने� 95 करोड़�बीज��को��वालबाड��भेजा�है।�

पमा��ो�ट (P ermafrost ) के ��पघलने�के �कारण�, वा�ट�, जलवायु�प�रवत�न�से�खतरे�का�सामना�कर�रहा�है।�

�या�आप�जानते�है� ?

· �हमालय�म��चांग�ला�के ��नकट�17 , 300 फ�ट�क��ऊं चाई�पर�भारत�का�अपना�बीज�वॉ�ट���थत�है� , जो�नेशनल��यूरो�ऑफ़��लांट�जेने�टक


�रसोस�ज� (भारतीय�कृ �ष�अनुसंधान�प�रषद�के �अंतग�त ) और��डफे �स�इं��ट�ूट�ऑफ़�उ�च�हाइ�ट�ूड��रसच�� (र�ा�अनुसंधान�और��वकास
संगठन�के �अंतग�त ) �ारा�संयु���प�से�बनाए�गए�है।�

�वचार�कर��

· Permafrost

· Cryosphere

http://www.thehindu.com/sci-tech/science/whats-inside-the-svalbard-global-seed-vault/
article22858978.ece

QUESTION 87.
हाल�ही�म��कोरोनरी��टं ट��क��क�मत��को��नयं�ण�म��लाने�स�ब�धी�सरकार�के �एक��नण�य�ने��च�क�सा�समुदाय�म��शॉक�वेव�का��नमा�ण��कया�है।
भारत�म��ऐसे�फामा��यू�टकल�उ�पाद��क��क�मत��को��नयं��त�करने�के ��लए��कस��ा�धकारी�को�आ�ाप���दया�गया�है� ?

IASbaba
Web: http://ilp.iasbaba.com/ Score:
Email: ilp@iasbaba.com 4.00 / 200
Page 151
Exam Title : Test 21-Revision: Polity & ...
Email
Contact :

a) रसायन�और�उव�रक�मं�ालय�
b) �व��मं�ालय�
c) �वा��य�और�प�रवार�क�याण�मं�ालय�
d) म�हला�एवं�बाल��वकास�मं�ालय�
Correct Answer: A
Your Answer:
Explanation

Solution (a)

रा�ीय�फामा��यु�टकल��ाइ�स�ग�अथॉ�रट��भारत�म��दवाइय��क��क�मत��को��नयं��त�करने�के ��लए��नयामक��ा�धकरण�है� , जो��क��डपाट� म�ट


ऑफ़�फामा��यू�टकल , रसायन�एवं�उव�रक�मं�ालय�के �अंतग�त�आता�है।�

�वचार�कर�� !

· �च�क�सा�उपकरण��और�दवाईय��के �मू�य��नधा�रण�के �प��और��वप��

http://www.thehindu.com/news/national/pharma-pricing-panel-chief-shunted-out/
article22897704.ece

QUESTION 88.
हाल�ही�म��समाचार�म���दखाई�दे ने�वाला�एक�सुपर���टकल�काब�न�डाइऑ�साइड��ैटन�साइ�कल�टे �ट�लूप�फै �स�लट���कससे�संबं�धत�है ?

a) �व�छ�ऊजा��
b) हाइपरलूप�
c) हाइ�ोजन�बम�
d) अंत�र��काय��म�
Correct Answer: A
Your Answer:
Explanation

Solution (a)

· भारतीय�वै�ा�नक��ने�एक�सुपर����टकल�काब�न�डाय�आ�साइड��ैटन�टे �ट�लूप�फै �स�लट��को��वक�सत��कया�है�जो�सोलर�थम�ल�समेत


भ�व�य�के ��व�ुत�संयं���से��व�छ�ऊजा��उ�प��करने�म��मददगार�होगा।�

· व�क�ग��लूइड�मटे �रयल�के ��प�म��बंद�च�� CO


2 के �साथ�नई�पीढ़��के �उ�च��मता�वाले��व�ुत्�संयं���म��भाप�आधा�रत�परमाणु�और�तापीय
�व�ुत�संयं���को���त�था�पत�करने�क���मता�है� , इस��कार�काब�न�फु ट����ट�म��पया��त��प�से�कमी�क��जा�सके गी।�

· इस�टे �ट�लूप�को��के ल�अप�S-CO2 �बजली�संयं���के �भ�व�य�म���वकास�के ��लए�आव�यक�डेटा�तैयार�करने�के ��लए��डज़ाइन��कया�गया�है


, �जसम��कई�तकनीक��चुनौ�तय��पर�काबू�पाने�क��आव�यकता�होगी� - टरबाइन�, कं �ेसर�और�हीट�ए�सच�जस��जैसे�मह�वपूण��घटक��को
�वक�सत�करना�जो�वां�छत�दाब�और�तापमान�सीमाएं�पर�काय��कर�सकते�ह��और�उन�साम��य��का�उपयोग�जो�इन���थ�तय��का�सामना�कर
सकते�ह�।�

· आज�के �थम�ल�पॉवर��लांट�ऊजा���ोत��से�गम���र�करने�और��बजली�उ�प��करने�के ��लए�टरबाइन�को�चालू�करने�के ��लए�भाप�का�इ�तेमाल


करते�ह�।�हालां�क� , यह�अ�धक�श���उ�प��कर�सकता�है� , य�द�भाप�के �बजाय�, सुपर���टकल� CO2 (S-CO 2 ) का�उपयोग��कया
जाता�है।�श�द� "सुपर���टकल ", 31 �ड�ी�से��सयस�के ��ां�तक�तापमान�तथा� 73 atm के ��ां�तक�दबाव�के �ऊपर�काब�न�डाइऑ�साइड�क�
��थ�त�को�बताता�है�जो�इसे�भाप�से��दोगुना�अ�धक�सघन�बनाता�है।�

IASbaba
Web: http://ilp.iasbaba.com/ Score:
Email: ilp@iasbaba.com 4.00 / 200
Page 152
Exam Title : Test 21-Revision: Polity & ...
Email
Contact :

http://www.thehindu.com/todays-paper/tp-national/tp-karnataka/supercritical-co2-brayton-cycle-
facility-opened-at-iisc/article22830320.ece

QUESTION 89.
हाल�ही�के �एक�पहल�के �अनुसार� , वायरल�लोड�परी�ण�का�उपयोग��कसके ��लए��कया�जाएगा?

a) र��म��एचआईवी�क��मा�ा�का�पता�लगाना�
b) एच1एन1 का��व�रत�संसूचन�
c) �ां�तक�साइबर�बु�नयाद��ढांचे�क��सुर�ा�
d) रैनसमवेयर�जैसे�साइबर�अपराध��को�रोकना�
Correct Answer: A
Your Answer:
Explanation

Solution (a)

सरकार�ने�एच .आई.वी./एड् स� (पी.एल.एच.आई.वी.) पी�ड़त�1.2 �म�लयन�लोग��के ��लए�सोमवार�को�एक�वष��म��कम�से�कम�एक�बार�मु�त


वायरल�लोड�परी�ण�उपल�ध�करवाने�के ��लए�एक�योजना�शु���कया�ह�।�

समय�के �साथ�एंट��र�ोवाइरल�थेरेपी� (एआरट�) क���भावशीलता�पर�नजर�रखने�के ��लए�वायरल�लोड�का�उपयोग��कया�जाता�है ; यह�र��म�


एचआईवी�आनुवं�शक�पदाथ�� (आरएनए) क��मा�ा�को�मापता�है�और��रपोट� �करता�है��क�वायरस�क���कतनी���तयां�मौजूद�ह�।�

�वचार�कर� !

· एंट�रे�ोवायरल�थेरेपी� (एआरट�)

· एचआईवी�परी�ण�(ए�लसा�, पीसीआर)

QUESTION 90.
हाल�ही�म����व�ट( SWIFT ) �ांजे�शन�समाचार�म��था� , यह��न�न�ल�खत�म��से��कसका�सं���त��प�है� ?

a) सोसायट��फॉर�व�ड�वाइड�इंटरब�क�फाइन��सयल��ा�सफर�
b) सोसायट��फॉर�व�ड�वाइड�इंटरब�क�फाइन��सयल�टे लीक�यु�नके शन�
c) सोसायट��फॉर�व�ड�वाइड�एंड�इंटरनल�फाइन��सयल��ा�सफर�
d) सोसायट��फॉर�व�ड�वाइड�इं��ट�ूशनल�फाइन��सयल�टे लीक�यु�नके शन�
Correct Answer: B
Your Answer:
Explanation

Solution (b)

द�सोसाइट��फॉर�व�ड�वाइड�इंटरब�क�फाइन��शयल�टे लीक�यु�नके शंस� (एसवीआईएफट�), एक�ऐसा�नेटवक� ��दान�करता�है�जो��व�ीय�सं�थान�


को�सुर��त� , मानक�कृ त�और��व�सनीय�वातावरण�म���व�ीय�लेनदे न�के �बारे�म��जानकारी�भेजने�और��ा�त�करने�के ��लए�स�म�बनाता�है।�

��व�ट��ांसफर , ��व�ट��अंतररा�ीय�भुगतान�नेटवक� �के �मा�यम�से�भेजे�जाने�वाले�अंतरा��ीय�धन�ह�तांतरण�का�एक��कार�है।�

पंजाब�नेशनल�ब�क�धोखाधड़ी , जहां�इंटर�ब�क�संदेश��णाली�के �मा�यम�से��न�ध�अंतरण�को�कोर�ब��क�ग�समाधान�को��रपोट� �नह���कया�गया�था ;


ने�अंतररा�ीय�ब��क�ग�के ��लए�SWIFT के �उपयोग�को�मह�वपूण��बना��दया�है।�

�या�आप�जानते�है� ?

IASbaba
Web: http://ilp.iasbaba.com/ Score:
Email: ilp@iasbaba.com 4.00 / 200
Page 153
Exam Title : Test 21-Revision: Polity & ...
Email
Contact :

SWIFT क���मुख�भू�मका�एक�सुर��त��ांस�मशन�चैनल��दान�करना�है�ता�क�ब�क� A को�यह�मालूम�हो��क�ब�क� B को�अपना�संदेश�ब�क� B


म��जाता�है�और�कह��और�नह�।�बदले�म�� , ब�क� B , को�मालूम�है��क�माग��पर�स�दे श�को�ब�क� A , तथा�ब�क� A के �अलावा�कोई�भी�नह��, भेज
सकता�है� , पढ़�सकता�है�या�बदल�सकता�है।�

�वचार�कर� !

· ब�क��के ��लए�बेसल��नयम�

· ब�क��के ��व�नयमन�के ��लए�आरबीआई�के �गुणा�मक�और�मा�ा�मक�उपाय�

http://www.thehindu.com/news/national/a-swift-autopsy-how-nirav-modi-defrauded-pnb/
article22844201.ece

QUESTION 91.
�व�ीय�काय�वाही�काय�बल� (एफ.ए.ट�.एफ.) के �संदभ��म���न�न�ल�खत�कथन��पर��वचार�क��जए ?

1. यह�एक�अंतरसरकारी�संगठन�है।�

2. इसके �जनादे श�म��ब��क�ग�को��व�नय�मत�करना� , मनी�लॉ���ंग�और�आतंकवाद�के ��व�पोषण�का�मुकाबला�करना�शा�मल�है।�

ऊपर��दए�गए�कथन�म��से�कौन�सा /से�सही�है� ?

a) के वल�1
b) के वल�2
c) दोन��1 और�2
d) न�तो�1 और�न�ही�2
Correct Answer: A
Your Answer:
Explanation

Solution (a)

फाइन��सयल�ए�शन�टा�क�फोस�� (एफएट�एफ), 1989 म���था�पत�एक�इंटर -गवन�म�टल�बॉडी�है।�

इसके �अ�धदे श�म��अंतररा�ीय��व�ीय��णाली�क��अखंडता�को�मनी�लॉ���ंग� , आतंकवाद���व�पोषण�और�अ�य�संबं�धत�खतर��का�सामना


करना�शा�मल�है।�

�वचार�कर� !

· एफएट�एफ�क���े -सूची�

· आ�थ�क�अपराध��का�सामना�करने�के ��लए�भारत�म��वैधा�नक�उपाय�

https://timesofindia.indiatimes.com/india/india-congratulates-china-on-financial-action-task-
force-post-hopes-beijing-will-be-balanced/articleshow/63066160.cms

QUESTION 92.
�न�न�ल�खत�दे श��म��से�कौन -कौन�'QUAD' का��ह�सा�ह�� ?

1. भारत�

2. संयु��रा�य�अमे�रका�

3. �स�

IASbaba
Web: http://ilp.iasbaba.com/ Score:
Email: ilp@iasbaba.com 4.00 / 200
Page 154
Exam Title : Test 21-Revision: Polity & ...
Email
Contact :

4. जापान�

नीचे��दए�गए�कू ट�से�सही�उ�र�चु�नए ?

a) 1 और�3
b) 2 और�4
c) 1,2 और�4
d) 1,2 , 3 और�4
Correct Answer: C
Your Answer: Unanswered
Explanation

Solution (c)

�वैड (�वा�ीलेटरल ) भारत�, यू .एस.ए. , जापान�और�ऑ��े �लया�को�संद�भ�त�करता�है।�

�वचार�कर� !

· �वाड, चीन�के �उ�कष��का�सामना�कै से�कर�सकता�है� ?

http://www.thehindu.com/todays-paper/tp-opinion/cornered-by-the-quad/article22872575.ece

QUESTION 93.
�शकार�पर�सतक� ता�बढ़ाने�तथा�व�य�जीव��से�स�बं�धत�अपराध��को��नयं��त�करने�के ��लए�, �न�न�ल�खत�म��से��कस�रा�य�म��व�यजीव�फा�ट
�ै क�कोट� ��था�पत��कया�गया� ?

a) �द�ली�
b) �बहार�
c) असम�
d) महारा��
Correct Answer: C
Your Answer:
Explanation

Solution (c)

दे श�म��पहली�बार� , असम�म�� 10 व�यजीव�फा�ट-�ै क�कोट� ��था�पत��कए�गए�ह�� , जो��वशेष��प�से�व�य�जीव��के ��खलाफ��शकार�और�अ�य


संबं�धत�अपराध��से��नपटने�के ��लए�ह�।�

�या�आप�जानते�है� ?

· असम�म��पांच�रा�ीय�उ�ान�और� 19 व�यजीव�अभयार�य�ह�।�इसके �अलावा�, यह�91% से�अ�धक�भारतीय�ग�ड��का�वास�थल�है।�

�वचार�कर� !

· व�यजीव�और�जैव��व�वधता�के �संर�ण�के ��लए��वधान�और�अंतरा��ीय�उपाय�

· असम�के �बायो��फयर��रज़व��और�रा�ीय�उ�ान��क��सूची�

http://www.thehindu.com/todays-paper/tp-national/guards-get-modern-weapons-to-fight-
poaching/article22793099.ece

QUESTION 94.

IASbaba
Web: http://ilp.iasbaba.com/ Score:
Email: ilp@iasbaba.com 4.00 / 200
Page 155
Exam Title : Test 21-Revision: Polity & ...
Email
Contact :

गोबर-धन�योजना�के �स�दभ��म���न�न�ल�खत�म��से�कौन�सा /से�इसके �उ�े �य�ह�� ?

1. गांव��क��सफाई�

2. कचरे�से�धन�

नीचे��दए�गए�कू ट�से�सही�उ�र�चु�नए ?

a) के वल�1
b) के वल�2
c) दोन��1 और�2
d) न�तो�1 और�न�ही�2
Correct Answer: C
Your Answer:
Explanation

Solution (c)

“गोबर-धन" (गै�वनाइ�ज�ग�ऑग��नक�बायो -ए�ो��रसोस�ज -धन) योजना, पशु��के �गोबर�और�खेत��म��ठोस�अप�श��के ��बंधन�और�इसे


क�पो�ट�, बायोगैस�और�जैव -सीएनजी�म��प�रव�त�त�करने�पर��यान�क� ��त�करेगी।�

इस�पहल�के �दो�उ�े �य�ह� : गांव��को�साफ�रखना�तथा�पशु�और�अ�य�कचरे�से�धन�और�ऊजा��उ�प��करना।�

�वचार�कर� !

· ठोस�एवं�तरल�कचरा��बंधन�

· अप�श��को�संपदा�को�समझने�म��चुनौ�तयां�

http://www.business-standard.com/article/opinion/govt-s-gobar-dhan-initiative-is-a-
commendable-plan-letter-to-bs-118022801336_1.html

QUESTION 95.
Rustom- 2 के �संदभ��म���न�न�ल�खत�कथन��पर��वचार�क��जए ?

1. इसे�र�ा�अनुसंधान�और��वकास�संगठन� (डी.आर.डी.ओ.) के ��ारा��वक�सत��कया�गया�है।�

2. यह�एक�मी�डयम�अ�ट�टयूट�लांग�ए��ुर�स��ोन� ( MALE) है।�

ऊपर��दए�गए�कथन��म��से�कौन�सा /से�सह��है� ?

a) के वल�1
b) के वल�2
c) दोन��1 और�2
d) न�तो�1 और�न�ही�2
Correct Answer: C
Your Answer:
Explanation

Solution (c)

��तम-2 एक�मानवर�हत�हवाई�वाहन�है��जसे�र�ा�अनुसंधान�एवं��वकास�संगठन� (डीआरडीओ) �ारा��वक�सत��कया�गया�है।�

यह�एक�मी�डयम�अ�ट�टयूट�लांग�इ��ुर�स��ोन� ( MALE) है।�

IASbaba
Web: http://ilp.iasbaba.com/ Score:
Email: ilp@iasbaba.com 4.00 / 200
Page 156
Exam Title : Test 21-Revision: Polity & ...
Email
Contact :

इस��ोन�को�भारतीय�सश���बल��के �सभी�तीन��सेवा��के �उपयोग, मु�य��प�से�खु�फया� , �नगरानी�और�टोही�(आईएसआर) संचालन�के


�लए��वक�सत��कया�गया�है।�

�या�आप�जानते�है� ?

· ��तम-2, मैनुअल�और�साथ�ही��वचा�लत�मोड�पर��मशन�उड़ान�भर�सकता�है।�

�वचार�कर� !

· �वदे शी�र�ा�उ�पादन�

https://economictimes.indiatimes.com/news/defence/drdo-successfully-carries-out-test-flight-of-
rustom-2-drone/articleshow/63068375.cms

QUESTION 96.
�व��पया�वरण��दवस� ( WED ) 2018 के �संबंध�म���न�न�ल�खत�कथन��पर��वचार�क��जए ?

1. भारत, 2018 �व��पया�वरण��दवस�का�वै��क�मेज़बान�है�जो� 5 जून� , 2018 को�होगा।�

2. WED 2018 का�थीम�' �ला��टक���षण�को�परा�त�करना(बीट��ला��टक�पोलुशन ) ' है।�

ऊपर��दए�गए�कथन��म��से�कौन�सा /से�सही�है� ?

a) 1 और�2
b) के वल�1
c) के वल�2
d) इनम��से�कोई�नह��
Correct Answer: A
Your Answer:
Explanation

Solution (a)

1972 म��मानव�पया�वरण�पर��टॉकहोम�स�मेलन�के ��थम��दन�, यूएन�महासभा�ने��व��पया�वरण��दवस� [ WED] क���थापना�क��, �जसके


प�रणाम�व�प�मानव�अंतर��या�और�पया�वरण�के �एक�करण�पर�चचा���ई।�दो�साल�बाद� , 1974 म���थम��वद��वषय� "के वल�एक�पृ�वी " के
साथ�आयो�जत��कया�गया�था।�

भारत�2018 �व��पया�वरण��दवस�का�वै��क�हो�ट�है�जो� 5 जून� , 2018 को�होगा।�

इस�साल�के �सं�करण�के ��वषय�के ��प�म�� "बीट��ला��टक���षण" के �साथ�, ��नया�एकल�उपयोग��ला��टक���षण�से��नपटने�के ��लए�एक


साथ�आ�रही�है।�

�व��पया�वरण� 2018 का�थीम�"बीट��ला��टक���षण" , सरकार��, उ�ोग��, समुदाय��और����य��से�आ�ह�करता�है��क�वे�एक�साथ�आएं


और��टकाऊ��वक�प��का�पता�लगाएँ�तथा�हमारे�महासागर��को����षत�करने�वाले , मरीन�लाइफ�को�नुकसान�प�ँचाने�वाले�एवं�मानव��वा��य
के ��लए�खतरा��स�गल�यूज़��ला��टक�के �अ�य�धक�उपयोग�म��कमी�लाएं।�

�या�आप�जानते�है� ?

· 2017 के ��लए�थीम�' कने��टं ग�पीपल�टू �नेचर - इन�द��सट��एंड�ऑन�द�ल�ड , �ॉम�पोल�टू �द�इ�वेटर ’ था; तथा�मेजबान�दे श�कनाडा�था।�

https://timesofindia.indiatimes.com/home/environment/pollution/drive-against-plastic-pollution-
to-gain-momentum-from-india-the-global-host-of-world-environment-day-2018/
articleshow/62994154.cms

IASbaba
Web: http://ilp.iasbaba.com/ Score:
Email: ilp@iasbaba.com 4.00 / 200
Page 157
Exam Title : Test 21-Revision: Polity & ...
Email
Contact :

QUESTION 97.
हाल�ही�म���यांमार�ने�भारतीय�सीमा�के �नजद�क�एक�नद��के ��कनारे�बाँध�का��नमा�ण�करना�शु��कर��दया�है� , �जससे�म�णपुर�के �च�डेल��जले�म
पानी�क��कमी�का�डर�है।��न�न�ल�खत�रा�य��म��से�कौन�म�णपुर�के �साथ�सीमा�नह��बनाता�है� ?

a) असम�
b) नगाल�ड�
c) �मजोरम�
d) ��पुरा�
Correct Answer: D
Your Answer:
Explanation

Solution (d)

�वचार�कर� !

· म�णपुर�क���मुख�न�दय��और�झील��

QUESTION 98.
भारतीय�नौसेना�के �प��मी�नौसेना�कमान��ारा�आयो�जत�स�ब�ध�अ�यास�म����न�न�ल�खत�म��से�कौन�शा�मल�थे� ?

a) आ�सयान�
b) �ह�द�महासागर�के �तट�य�दे श�
c) �ब�सटे क�दे श�
d) भारत-�वयतनाम-मले�शया -थाईल�ड�
Correct Answer: B
Your Answer:
Explanation

Solution (b)

��तभा�गय��म���ह�द�महासागर�के �10 तट�य�दे श�मालद�व� , �यांमार� , इंडोने�शया� , �ीलंका� , बां�लादे श� , ओमान�, कतर�, संयु��अरब
अमीरात�, मले�शया�और�मॉरीशस�शा�मल��ए।�

पहल�के �एक��ह�से�के ��प�म��छोटे �दे श��के �नौसेना�अ�धका�रय��और�कै डेट��को�भारत�के ��वमान�वाहक�बेड़े�आईएनएस��व�मा�द�य�पर�बड़े


नौसेना��का��वागत��कया�गया।�

IASbaba
Web: http://ilp.iasbaba.com/ Score:
Email: ilp@iasbaba.com 4.00 / 200
Page 158
Exam Title : Test 21-Revision: Polity & ...
Email
Contact :

�वचार�कर� !

· भारतीय�नौसै�नक�अ�यास�

· सै�य�अ�यास�के �संचालन�का�मह�व�

http://www.livemint.com/Opinion/A2oieg9GPqS7R39VecSOOK/Building-maritime-capacity-in-
SouthEast-Asia.html

QUESTION 99.
�न�न�ल�खत�म��से�कौन�सा /से�रा�ीय�बचत��माणप��क���वशेषताएं�ह�� ?

1. संपा���क (कोलैटरल ) के ��प�म��रखा�जा�सकता�है।�

2. अ�धकतम��नवेश� 5 लाख��. है।�

3. ��ट, इसम���नवेश�नह��कर�सकते�ह�।�

नीचे��दए�गए�कू ट�से�सही�उ�र�चु�नए ?

a) 1 और�2
b) 1 और�3
c) 2 और�3
d) 1,2 और�3
Correct Answer: B
Your Answer:
Explanation

Solution (b)

रा�ीय�बचत��माण�प��(एनएससी), भारत�सरकार��ारा�जारी�एक��नवेश�योजना�है।�इसम���न�न�ल�खत��वशेषताएं�ह� :

· यह�योजना��वशेष��प�से�सरकारी�कम�चा�रय�� , �वसा�यय��और�अ�य�वेतनभोगी�वग��के ��लए�तैयार�क��गई�है� , जो�आयकर�का�मू�यांकन


कर�रह��ह�।�

· �नवेश�के ��लए�कोई�अ�धकतम�सीमा�नह��है।�

· �ोत�पर�कोई�कर�कटौती�नह��है।�

· ब�क��से�ऋण�पाने�के ��लए��माण�प��को�संपा���क�सुर�ा�के ��प�म��रखा�जा�सकता�है।�

· ��ट�और�एचयूएफ��नवेश�नह��कर�सकते�ह�।�

सरकार��व�भ��अ�ध�नयम��के �तहत�आने�वाले��व�भ��लघु�बचत�योजना��को�एक�ही�अ�ध�नयम� , सरकारी�बचत�ब�क� (जीएसबी) अ�ध�नयम


, 1873 म��मज��करने�क��योजना�बना�रही�है।�

�वचार�कर� !

· लघु�बचत�योजनाएं�

· सरकारी�बचत�ब�क� (जीएसबी) अ�ध�नयम�, 1873

http://www.livemint.com/Money/v7KOGkxFAFMYnuiq1RawfN/Government-proposes-
amendments-in-small-savings-schemes.html

IASbaba
Web: http://ilp.iasbaba.com/ Score:
Email: ilp@iasbaba.com 4.00 / 200
Page 159
Exam Title : Test 21-Revision: Polity & ...
Email
Contact :

QUESTION 100.
�चट�फं ड�के �संदभ��म���न�न�ल�खत�कथन��पर��वचार�क��जए :

1. यह�भारत�म���च�लत�एक�बचत�योजना�है।�

2. यह�पूरी�तरह�से�अ�नय�मत�है।�

3. एक�लेन -दे न (�ांजे�शन ) , �जसम��कु छ�एक�को�, पर�तु�सभी�को�नह�� , भ�व�य�क��सद�यता�का�भुगतान�करने�के ��लए��बना��कसी�दे यता�के


पुर�कार�रा�श��ा�त�होती�है , तो�यह�एक��चटफं ड�नह��है।�

ऊपर��दए�गए�कथन��म��से�कौन�सा /से�सही�है� ?

a) 1 और�2
b) 1 और�3
c) 2 और�3
d) 1,2 और�3
Correct Answer: B
Your Answer:
Explanation

Solution (b)

�चट�फं ड, भारत�म���च�लत�एक��कार�क��बचत�योजना�है।�

�चट�फं ड��वसाय�को�क� ��य�अ�ध�नयम, �चट�फं ड�अ�ध�नयम�, 1982 तथा�इस�अ�ध�नयम�के �तहत�इस�उ�े �य�के ��लए��व�भ��रा�य
सरकार��के ��ारा�बनाए�गए��नयम��के �अंतग�त��व�नय�मत��कया�जाता�है।�

एक�लेनदे न (�ांजे�शन ), �चट�नह��है� , अगर�ऐसे�लेनदे न�म� -

· कु छ�एक�स�स�ाइबर��को�, पर�तु�सभी�नह�� , भ�व�य�क��सद�यता�का�भुगतान�करने�के ��लए��बना��कसी�दे यता�के �पुर�कार�रा�श��ा�त�होती


है� ; या�

· सभी�स�स�ाइबर��को�भ�व�य�क��सद�यता�का�भुगतान�करने�के ��लए�दे यता�के �साथ�बारीबारी�से��चट�रा�श��मलती�है।�

· काया��मक��प�से� , �चट�फं ड�को�सब-हेड��मसले�नयस�नॉन -ब��क�ग�कं पनी�(एमएनबीसी) के �तहत�आरबीआई��ारा�गैर -ब��क�ग��व�ीय


कं प�नय��क��प�रभाषा�म��शा�मल��कया�गया�है।�ले�कन�आरबीआई�ने�उनके ��लए�कोई�अलग��नयामक�ढांचा�नह��रखा�है।�

· अब�, �चट�फं ड�कं पनी�को�र�ज��ार�कं प�नय��से��नगमन�का��माण�प�� (सीआईएन) �ा�त�करना�होगा�और�इसके �बाद�संबं�धत�रा�य�के ��चट


फं ड��वभाग�के �पंजीकरण�के ��लए�आवेदन�करना�होगा।�

�या�आप�जानते�है� ?

�चट�फं ड�अ�ध�नयम�म��संशोधन� - हाल�ही�म�� ; कै �बनेट�ने��चट�फं ड् स�ए�ट� , 1982 म��संशोधन�क��मंजूरी�दे �द��है।�एक�संशोधन�के �अनुसार


अ�ध�नयम�म���चट��वसाय�के ��लए�"�े टर�नट��फ�ड" श�द��का�इ�तेमाल��कया�गया�है� , �जससे�यह�अंत�न��हत��कृ �त�को�दशा�ता�है� , और
इसके �काय�कलाप�को� ' �ाइज��चट�' से�अलग�कर�सकता�है , जो�एक�अ�य�कानून�के �तहत���तबं�धत�है।�

�वचार�कर� !

· गैर -ब��क�ग��व�ीय�कं प�नयां�

http://www.livemint.com/Money/aasZcBKy9PcW3jK0ZOi6dN/New-rules-for-chit-funds-and-
deposit-schemes.html

IASbaba
Web: http://ilp.iasbaba.com/ Score:
Email: ilp@iasbaba.com 4.00 / 200
Page 160

S-ar putea să vă placă și